Sie sind auf Seite 1von 66

PART FOUR

I.
DEFINITION, NATURE AND PURPOSE
A. NATURE AND VIEWS
PCGG VS DESIERTO
Facts:
On October 8, 1992, then President Fidel V. Ramos
issued Administrative Order No. 13 creating the
Presidential Ad Hoc Fact-Finding Committee on Behest
Loans (Committee) which was tasked to inventory all
behest loans, determine the parties involved and
recommend whatever appropriate actions to be pursued
thereby.
On November 9, 1992, President Ramos issued
Memorandum Order No. 61 expanding the functions of
the Committee to include the inventory and review of all
non-performing loans, whether behest or non-behest.
The Memorandum set the following criteria to show the
earmarks of a "behest loan," to wit: "a) it is
undercollaterized; b) the borrower corporation is
undercapitalized; c) a direct or indirect endorsement by
high government officials like presence of marginal
notes; d) the stockholders, officers or agents of the
borrower corporation are identified as cronies; e) a
deviation of use of loan proceeds from the purpose
intended; f) the use of corporate layering; g) the nonfeasibility of the project for which financing is being
sought; and, h) the extraordinary speed in which the loan
release was made."
Among the accounts referred to the Committee's
Technical Working Group (TWG) were the loan
transactions between NOCOSII and PNB.
After it had examined and studied all the documents
relative to the said loan transactions, the Committee
classified the loans obtained by NOCOSII from PNB as
behest because of NOCOSII's insufficient capital and
inadequate collaterals. Specifically, the Committee's
investigation revealed that in 1975, NOCOSII obtained
loans by way of Stand-By Letters of Credit from the
PNB; that NOCOSII was able to get 155% loan value
from the offered collateral or an excess of 85% from the
required percentage limit; that the plant site offered as
one of the collaterals was a public land contrary to the
General Banking Act; that by virtue of the marginal note
of then President Marcos in the letter of Cajelo,
NOCOSII was allowed to use the public land as plant
site and to dispense with the mortgage requirement of
PNB; that NOCOSII's paid-up capital at the time of the
approval of the guaranty was only P2,500,000.00 or only
about 6% of its obligation.
Based on the Sworn Statement of PCGG consultant
Orlando Salvador, petitioner filed with the Office of the
Ombudsman
the
criminal
complaint
against
respondents. Petitioner alleges that respondents violated

the following provisions of Section 3 (e) and (g) of R.A.


No. 3019.
The respondents failed to submit any responsive
pleading before the Ombudsman, prompting Graft
Investigator Officer (GIO) I Melinda S. Diaz-Salcedo to
resolve the case based on the available evidence. In a
Resolution dated January 12, 1998 in OMB-0-95-0890,
GIO Diaz-Salcedo recommended the dismissal of the
case on the ground of insufficiency of evidence or lack of
probable cause against the respondents and for
prescription of the offense. Ombudsman Desierto
approved the recommendation on May 21, 1999.
Petitioner filed a Motion for Reconsideration but it was
denied by GIO Diaz-Salcedo in the Order dated July 9,
1999, which was approved by Ombudsman Desierto on
July 23, 1999.
Issue:
Whether respondents violated the following provisions of
Sec 3 (e) and (g), specifically corrupt practices of public
official, of Republic Act No. 3019 or the Anti-Graft and
Corrupt Practices Act?
Held:
On the issue of whether the Ombudsman committed
grave abuse of discretion in finding that no probable
cause exists against respondents, it must be stressed
that the Ombudsman is empowered to determine
whether there exists reasonable ground to believe that a
crime has been committed and that the accused is
probably guilty thereof and, thereafter, to file the
corresponding information with the appropriate courts.
Settled is the rule that the Supreme Court will not
ordinarily interfere with the Ombudsman's exercise of his
investigatory and prosecutory powers without good and
compelling reasons to indicate otherwise. Said exercise
of powers is based upon his constitutional mandate and
the courts will not interfere in its exercise. The rule is
based not only upon respect for the investigatory and
prosecutory powers granted by the Constitution to the
Office of the Ombudsman, but upon practicality as well.
Otherwise, innumerable petitions seeking dismissal of
investigatory
proceedings
conducted
by
the
Ombudsman will grievously hamper the functions of the
office and the courts, in much the same way that courts
will be swamped if they had to review the exercise of
discretion on the part of public prosecutors each time
they decided to file an information or dismiss a complaint
by a private complainant.
While there are certain instances when this Court may
intervene in the prosecution of cases, such as, (1) when
necessary to afford adequate protection to the
constitutional rights of the accused; (2) when necessary
for the orderly administration of justice or to avoid
oppression or multiplicity of actions; (3) when there is a
prejudicial question which is sub-judice; (4) when the
acts of the officer are without or in excess of authority;

(5) where the prosecution is under an invalid law,


ordinance or regulation; (6) when double jeopardy is
clearly apparent; (7) where the court has no jurisdiction
over the offense; (8) where it is a case of persecution
rather than prosecution; (9) where the charges are
manifestly false and motivated by the lust for vengeance;
and (10) when there is clearly no prima facie case
against the accused and a motion to quash on that
ground has been denied, none apply here.

administrative matters, and this had been the practice


under the 1973 and 1987 Constitutions. The prosecution
by the COMELEC of violations of election laws is an
administrative power.
3. The exercise by the COMELEC of its quasi-judicial
powers is subject to Sec.3, Art.IX-C which expressly
requires that all election cases, including preproclamation controversies, shall be decided by the
COMELEC in division, and the motion for reconsid

After examination of the records and the evidence


presented by petitioner, the Court finds no cogent reason
to disturb the findings of the Ombudsman.

DOJ-NPS MANUAL PART III


SECTION 1. Concept of preliminary investigation A preliminary investigation is an inquiry or proceeding to
determine whether there is sufficient ground to engender a
well founded belief that a crime cognizable by the Regional
Trial Court has been committed and that the respondent is
probably guilty thereof and should be held for trial.

No grave abuse of discretion can be attributed to the


Ombudsman. Grave abuse of discretion implies a
capricious and whimsical exercise of judgment
tantamount to lack of jurisdiction. The exercise of power
must have been done in an arbitrary or despotic manner
by reason of passion or personal hostility. It must be so
patent and gross as to amount to an evasion of positive
duty or a virtual refusal to perform the duty enjoined or to
act at all in contemplation of law.
The herein assailed Orders being supported by
substantial evidence, there is no basis for the Court to
exercise its supervisory powers over the ruling of the
Ombudsman. As long as substantial evidence supports
the Ombudsman's ruling, that decision will not be
overturned.
WHEREFORE, the petition is DISMISSED. Except as to
prescription, the assailed Resolution dated May 21, 1999
and Order dated July 2
BAYATAN V COMELEC
Facts: Reynato Baytan registered as a voter in two
precincts and the COMELEC En Banc affirmed the
recommendation of its Law Department to file
information of double registration in violation of the
Election Code. Baytan filed with the Supreme Court a
petition for certiorari on the grounds, among others, that
there was no probable cause and that election cases must
first be heard and decided by a Division before the
COMELEC En Banc can assume jurisdiction.
Held: 1. It is well- settled that the finding of probable
cause in the prosecution of election offenses rests in the
sound discretion of the COMELEC. Generally, the Court
will not interfere with such finding of the COMELEC,
absent a clear showing of grave abuse of discretion. This
principle emanates from the exclusive power of the
COMELEC to conduct preliminary investigation of all
election investigation of all election offenses and to
prosecute the same.
2. Under Sec. 2, Art. IX-C of the Constitution, the
COMELEC exercises both administrative and quasijudicial powers. The administrative powers are found in
Sec 2. (1), (3) to (9) of Art IX-C. The Constitution does
not provide on whether these administrative powers
shall be exercised by the COMELEC en banc or in
division. The COMELEC en banc therefore can act on

A preliminary investigation is essentially a judicial


inquiry since there is the opportunity to be heard, the
production and weighing of evidence, and a decision rendered
on the basis of such evidence. In this sense, the investigating
prosecutor is a quasi-judicial officer.
SEC. 2. Purpose of preliminary investigation. - A
preliminary investigation is intended:
a)

to secure the innocent against hasty,


malicious and oppressive prosecution and to
protect him from an open and public
accusation of a crime and from the trouble,
expense and anxiety of a public trial and
b)
to protect the State from having to conduct
useless and expensive trials.\
SEC. 3. Nature of preliminary investigation. - The
conduct of a preliminary investigation is a substantive right
which the accused may invoke prior to or at least at the time of
plea, the deprivation of which would be a denial of his right to
due process.
SEC. 4. Effect of amendment of information. - In case
an information is amended, a new preliminary investigation
shall be conducted if the amended charge is not related to the
crime originally charged; if there is a change in the nature of
the crime charged; or if the information on its face is null and
void for lack of authority to file the same.
SEC. 5. where right of preliminary investigation may
he invoked. - The right to a preliminary investigation may be
invoked only in cases cognizable by the Regional Trial Court.
The right is not available in cases triable by inferior courts.
SEC. 6. Officers Authorized to Conduct Preliminary

Investigation. - The following may conduct a preliminary


investigation;
a)
b)
c)
d)

Provincial or City Prosecutors and their


assistants;
Judges of Municipal Trial Courts and
Municipal Circuit Trial Courts;
National and Regional State Prosecutors;
and
Other officers as may be authorized by law.

Their authority to conduct preliminary investigation


shall include all crimes cognizable by the proper court in their
respective territorial jurisdiction.
SEC.
7.
Commencement
of
Preliminary
Investigation.- A preliminary investigation proceeding is
commenced:
a)

b)

c)

d)
e)

by the filing of a complaint by the offended


party or any competent person8 directly with
the Office of the Investigating Prosecutor or
Judge;
by referral from or upon request of the law
enforcement agency that investigated a
criminal incident;
upon request of a person arrested or detained
pursuant to a warrantless arrest who
executes a waiver of the provisions of
Article 125 of the Revised Penal Code, as
amended;
by order or upon directive of the court or
other competent authority; or
for election offenses, upon the initiative of
the Commission on Elections, or upon
written complaint by any citizen, candidate,
registered political party, coalition of
registered parties or any accredited citizen
arm of the Commission on Elections.

SEC. 8. Complaint. - For purposes of preliminary


investigation, the complaint filed with the prosecutor's office
shall, as far as practicable, be accompanied or covered by an
Information Sheet and shall state, among others a)

b)
c)

the full and complete names and exact


home, office or postal addresses of the
complainant and his witnesses;
The full and complete name and exact home,
office or postal address of the respondent;
The offense charged and the place and exact

d)

date and time of its commission; and


Whether or not there exists a related case
and, if so, the docket number of said case
and the name of the Investigating Prosecutor
thereof.

SEC. 9. Supporting affidavits.- The complaint shall be


accompanied by the affidavits of the complainant and his
witnesses, as well as other supporting proofs or documents, if
any. The affidavits shall be sworn to before a Provincial, City
or State Prosecutor, or other government official authorized to
administer oaths or, in their absence or unavailability, a notary
public, who must certify that he personally examined the
affiants and that he is satisfied that they voluntarily executed
and understood their affidavits.
When the preliminary investigation is commenced by
referral from or upon request of the law enforcement agency
that investigated the incident, the affidavits of the complainant
and his witnesses to be submitted by the said agency shall
consist of the original or duplicate original or certified
machine copies thereof.
SEC. 10. Number of copies of affidavits. - The
complaint and supporting affidavits shall be in such number of
copies as there are respondents, plus four (4) copies for the
court/official file.
Where a complaint charges multiple offenses which
cannot be the subject of one indictment or information, the
complainant may be required to submit such additional copies
of the complaint and supporting affidavits as there are offenses
charged in the complaint.
SEC. 11. Barangay certification . - If the offense
charged is punishable by imprisonment not exceeding one (1)
year or a fine not exceeding Five Thousand Pesos (5,000.00)
and the parties to the case are all residents of the same city or
municipality, the complaint shall be accompanied by the
certification required under Section 412 (a) of R.A. Act
No.7160, "The Local Government Code of 1991.
SEC. 12. Lack of harangay certification.- The
absence of a barangay certification shall not be organizations
under the party-list system or a ground for the dismissal of the
complaint. The Investigating Prosecutor shall, however, make
the corresponding referral of the complaint to the proper
Lupong Tagapamayapa for appropriate action pursuant to the
provisions of Chapter 7, Book III of R.A. No.7160. In
connection therewith, the complainant may be summoned for
the purpose of delivering the referral to the Chairman of the

appropriate barangay and to secure the necessary certification


within thirty (30) days.
In any of the following cases. the Investigating
Prosecutor shall proceed to take cognizance of the complaint
for purposes of preliminary investigation even if there is no
Barangay Certification:
a)
b)

c)

where the respondent is under detention; or


where the respondent has been deprived of
personal liberty calling for habeas corpus
proceedings; or
where the case may be barred by the Statute
of Limitations.

SEC. 13. Initial/ action on the Complaint.- Within ten


(10) days after the filing of the complaint, the Investigating
Prosecutor shall either dismiss the same if he finds no ground
to continue with the inquiry, or issue a subpoena to the
respondent, attaching thereto a copy of the complaint together
with the affidavits of witnesses and other supporting
documents.
9
SEC. 14. Dismissal of complaint. - The following,
among others, shall constitute sufficient basis for the outright
dismissal of a complaint:
a)

b)

c)

d)

e)

that the offense charged in the complaint


was committed outside the territorial
jurisdiction of the Office of the Investigating
Officer;
that, at the time of the filing of the
complaint, the offense charged therein had
already prescribed;
that the complainant is not authorized under
the provisions of pertinent laws to file the
complaint;
that the acts and/or omissions alleged in the
complaint and/or the supporting affidavits
do not sufficiently show that a criminal
offense or violation of a penal law has been
committed; or
that the complaint and the supporting
affidavits are unsigned and/or have not been
duly subscribed and sworn to as prescribed
under the Rules on Criminal Procedure.

SEC. 15. Personal service of documents by


investigating prosecutor. -Whenever circumstances warrant
and to prevent the loss of documents in the course of the
service of a subpoena through ordinary modes, the

Investigating Prosecutor may require the respondent or other


parties to appear before him on a designated date, time and
place and then and there personally furnish them with copies
of the complaint, supporting affidavits and other documents.
At the said or any other setting, the respondent shall
have the right to examine all other evidence submitted by the
complainant.
Failure on the part of the respondent or his
counsel/representative to appear before the Investigating
Prosecutor to obtain copies of the complaint, supporting
affidavits and other documents despite receipt of notice or
subpoena shall be considered a waiver or forfeiture of
respondent9s right to be furnished copies of the complaint,
supporting affidavits and other documents, as well as to
examine all other evidence submitted by the complainant.
For the purposes specified in the first paragraph hereof,
the Investigating Prosecutor shall not require the appearance
before him of the respondent or other parties who are residing
in distant places. In such cases, the Investigating Prosecutor
shall issue and send the subpoena, together with copies of the
complaint, supporting affidavit and other documents, by
registered special delivery mail with return card.
SEC. 16. Service of subpoena in preliminary
investigation. - To expedite the conduct of a preliminary
investigation, the following guidelines shall be observed in the
service of subpoenasa)

b)

Service
of
subpoena
and
all
papers/documents required to be attached a
thereto shall be b'~- personal service by
regular process servers. In their
Absence, the cooperation of the Provincial
City/Municipal Station Commanders of the
Philippine National Police (PNP) may be
requested for the purpose.
Under other circumstances, where personal
service cannot be effected but the respondent
cannot be considered as incapable of being
subpoenaed as when he continues to reside at
his known address but the return states that
he "has left his residence and his return is
uncertain" or words of similar import,
service of subpoena and its attachments shall
be effected by registered mail with return
card at respondent's known home/office
address. On the face of the envelope shall be
indicated
I. the name and return

address of the sender1 and


the
Typewritten/printed
phrase "First Notice
Made
on______________",
thus
instructing
the
postmaster/postal
employee
of
the
necessity of informing
the sender of the date the
first notice was made on
the addressee; and
ii.the

typewritten/printed

request: "If not claimed within


five (5) days from first
notice, please return to
sender."
c)

Upon receipt of the unclaimed/returned


envelope, the Investigating Prosecutor may
then proceed to resolve the complaint on the
basis of the evidence presented by the
complainant.
SEC. 17. where Respondent cannot he subpoenaed If a respondent cannot be subpoenaed, as, for instance, he
transferred residence without leaving any forwarding address,
the Investigating Prosecutor shall base his resolution on the
evidence presented by the complainant.
SEC. 18. Counter-Affidavits.- In cases where the
respondent is subpoenaed, he shall within ten (10) days from
receipt of the complaint and other documents, submit his
counter-affidavit and other supporting documents which shall
be sworn to and certified as prescribed in the second sentence
of par. 1 of Section 9 this Part, copies of which shall be
furnished by the respondent to the complainant.
Only a counter-affidavit subscribed and sworn to by
the respondent before the Public Prosecutor can dispute or put
at issue the allegations in the complaint. A memorandum,
manifestation or motion to dismiss signed by the counsel
cannot take the place of a counter-affidavit. Thus, a respondent
relying on the manifestation, memorandum or motion to
dismiss of his counsel is deemed to have not controverted
complainant's evidence.
However, if such memorandum, manifestation or motion to
dismiss is verified by the respondent himself, the same may be

considered a counter-affidavit.
SEC. 19. Motion for dismissal of bill of particulars.The filing of a motion for the dismissal of the complaint or for
the submission of a bill of particulars shall not suspend or
interrupt the running of the period for the submission of
counter-affidavits and other supporting documents.
All the grounds for the dismissal of the complaint, as
well as objections to the sufficiency thereof, shall be alleged or
incorporated in the counter-affidavit and shall be resolved by
the Investigating Prosecutor jointly on the merits of the case.
The Investigating Prosecutor may grant a motion to
dismiss filed by a respondent who is yet to file or has not filed
his counter-affidavit if the said motion is verified and
satisfactorily establishes, among others:
a)
the circumstances specified in subparagraphs (a), (b)9 (c) and (d) and (e) of
Section 14 of this Part;
b)
the fact that the complaint, or one
similar thereto or identical therewith, has
previously been filed with the Office and has
been fully adjudicated upon on the merits
after
due
preliminary
investigation
proceedings; or
c)
the extinction of respondentts
criminal liability by reason of death, pardon,
amnesty, repeal of the law under which
prosecution is sought, or other legal causes.
SEC. 20. Consolidation.- The following cases shall,
as far as practicable, be consolidated for preliminary
investigation purposes and assigned to and jointly heard by
one Investigating Officer:
a)
charges and counter-charges;
b)
cases arising from one and the
same incident or transaction or series of
incident or transactions; and
c)
cases involving common parties
and founded on factual and/or legal issues of
the same or similar character.
SEC. 21. Extension of time. - No motion or request
for extension of time to submit counter-affidavits shall be
allowed or granted by the Investigating Prosecutor except
when the interest of justice demands that the respondent be
given reasonable time or sufficient opportunity to:

a)

b)

c)

engage the services of counsel to


assist him in the preliminary
investigation proceedings;
examine or verify the existence,
authenticity or accuracy of
voluminous records, files, accounts
or other papers or documents
presented or submitted in support
of the complaint; or
undertake studies or research on
novel, complicated or technical
questions or issues of law and of
facts attendant to the case under
investigation.

Extensions of time to submit a counter-affidavit for


any of the reasons stated above shall not exceed ten (10) days.
Additional extensions may be authorized by the
Provincial/City Prosecutor concerned.
SEC. 22. Suspension of proceedings.- Upon motion of
a party, or when raised in a counter-affidavit, the Investigating
Prosecutor may suspend the preliminary investigation
proceedings if the existence of a prejudicial question is
satisfactorily established.
The existence of a prejudicial question shall,
however, not be a ground for the dismissal of the complaint.
SEC. 23. Concept of prejudicial question.- A
prejudicial question is one the resolution of which is a logical
antecedent of the issue involved in a case and the cognizance
of which pertains to another tribunal. It is based on a fact
distinct and separate from the crime charged but so intimately
connected with it that it determines the guilt or innocence of
the accused. To suspend the criminal action, it must not only
appear that said case involves facts intimately related to those
upon which the criminal prosecution would be based but also
that in the resolution of the issue or issues raised in the civil
case, the guilt or innocence of the accused would necessarily
be determined.
SEC. 24. Elements of prejudicial question. - The
essential elements of a prejudicial question are:
a)

b)

the civil action involves an issue similar or


intimately related to the issue raised in the
criminal action;
the resolution of such issue determines
whether or not the criminal action may
proceed ; and

c)

the cognizance of the said issue pertains to


another tribunal.

SEC. 25. Issuance of orders of suspension of


proceedings.- No resolution or order suspending the
preliminary investigation based on the existence of a
prejudicial question shall be issued by the Investigating
Prosecutor without the written approval of the Provincial/City
Prosecutor concerned or his duly designated assistant.
SEC. 26. Reply-affidavits and rejoinders.- The
Investigating Prosecutor shall not require or allow the filing or
submission of reply-affidavits and/or rejoinders, except where
new issues of fact or questions of law which are material and
substantial in nature are raised or invoked in the counteraffidavit or subsequent pleadings and there exists a need for
said issues or questions to be controverted or rebutted,
clarified or explained to enable the Investigating Prosecutor to
arrive at a fair and judicious resolution of the case. In such a
case, the period for the submission of reply affidavits or
rejoinders shall in no case exceed five (5) days unless a longer
period is authorized by the Provincial/City Prosecutor
concerned.
SEC. 27. Clanficatory questions. - The Investigating
Prosecutor may set a hearing to propound clarificatory
questions to the parties or their witnesses if he believes that
there are matters which need to be inquired into personally by
him. In said hearing, the parties shall be afforded the
opportunity to be present but without the right to examine or
cross-examine. If they so desire, they may submit written
questions to the Investigating Prosecutor who may propound
such questions to the parties or witnesses concerned.
The Investigating Prosecutor shall make a record of
the questions asked and answers given during the clarificatory
questioning which shall be signed by the parties concerned
and/or their respective counsel. Said notes shall form part of
the official records of the case. Parties who desire to file a
petition for review of the Investigating Officer's resolution
may, at their option, cite specific portions of the oral testimony
by reference to the transcript of stenographic notes. Said notes
shall only be transcribed in cases of appeal and shall be
obtained at the expense of the interested party.
28. Submission of case for resolution. - The
Investigating Prosecutor shall case submitted for resolution:
a)

when the respondent cannot be


subpoenaed or, if subpoenaed, does
not submit his counter-affidavit

within the reglementary period. In


such a case, the Investigating
Prosecutor shall base his resolution
on the evidence presented by the
complainant; or
b)

upon submission by the parties of


their respective affidavits and
supporting proof or documents, in
which event, he shall, upon the
evidence thus adduced, determine
whether or not there is sufficient
ground to hold the respondent for
trial

SEC. 29. Lack of probable cause.- If the Investigating


Prosecutor does not find sufficient basis for the prosecution of
the respondent, he shall prepare the resolution recommending
the dismissal of the complaint.
SEC. 30. Finding of probable cause. - If the
Investigating Prosecutor finds that probable cause exists, he
shall prepare the resolution and the corresponding information
or complaint in the appropriate cases.
Where the respondent is a public officer or employee
or a member of the Philippine National Police (PNP), the
Investigating Prosecutor shall also determine whether or not
the offense with which he is charged was committed in
relation to his office and, if so committed, such fact should be
alleged in the information to be filed with the Sandiganbayan
through the Ombudsman
SEC. 31. Reopening of investigation.- After a case
under preliminary investigation has been submitted for
resolution under the provisions of the preceding Section but
before promulgation of the resolution, the preliminary
investigation may be reopened for the purpose of receiving
new and/or additional evidence upon the prior authorization
given by the Provincial/City Prosecutor concerned or upon
motion of the interested party, Provided, That in the latter case,
it shall be subject to the following conditions:
a)
the motion is verified and a copy
thereof furnished the opposing party;
b)
the motion is accompanied with the
new and/or additional evidence; and
c)
the motion sufficiently and
satisfactorily shows valid and justifiable
reason for the failure of the movant to
submit the new and/or additional evidence

during the
proceedings.

preliminary

investigation

SEC. 32. Cases Transmitted by the Municipal Trial


Judge. - Upon receipt of the records of the case from the
Municipal Trial Court or Municipal Circuit Trial Court which
conducted the Preliminary Investigation, the Prosecution
Office shall review the case based on the existing records,
without requesting the parties to submit memorandum of
authorities, and may affirm, modify or reverse the finding of
the Municipal Trial Court judge. However, if the interest of
justice so requires, the prosecutor may conduct a full blown
reinvestigation giving the parties the opportunity to submit
additional evidence, and thereafter, resolve the case on the
basis of the totality of the evidence thus adduced.
SEC. 33. Memoranda. - The Investigating Prosecutor
shall not require nor allow the filing or submission by the
parties of memoranda unless the case involves difficult or
complicated questions of law or of fact. In any event, the filing
of memoranda by the parties shall be done simultaneously and
the period therefore shall not exceed ten (10) days, unless a
longer period is authorized by the Provincial/City Prosecutor
concerned.
SEC. 34. Period for resolving a case. - The
Investigating Prosecutor shall resolve the case within ten (10)
days from the time the case is deemed submitted for
resolution,1 unless otherwise provided herein or a longer
period is authorized by the Provincial/City Prosecutor
concerned.
SEC. 35. Form of resolution and number of copies.- The
resolution shall be written in the official language, personally
and directly prepared and signed by the Investigating
Prosecutor. It shall be prepared in as many copies as there are
parties, plus three (3) additional copies.

SEC. 36. Contents of the resolution. - A resolution

shall contain a caption and a body.

SEC. 37. Caption of resolution. - The caption of the

resolution shall indicate the:

more docket numbers, the caption shall also contain said


information.
a)names of all the complainants and all of
the respondents;
b) Case Number, otherwise known as the
Investigation Slip Number or 1.8. No.;
c) the offense charged;
d) the date of the filing of the complaint
with the office;
e) the date of the assignment of the case to
or receipt of the case record by the
Investigating Officer; and
f) the date the case was submitted for
resolution.

SEC. 41. Contents of body of resolution. - In general,


the body of resolution should contain:

SEC. 38. Names of parties. - The complete names of


all the complainants and respondents in the case shall be set
out in the caption of the resolution. It is not proper to use the
phrase "et. al." to refer to other complainants and respondents.

SEC. 42. Parts of a resolution. - As a rule, the body


of a resolution is made up of four parts, namely:

The name of the victim or injured party, not their


representative, shall appear in the caption. In cases referred to
the prosecution by the police where there is no identified
victim, as in prohibited drugs cases, the complainant shall be
the police station involved, followed by the name and
designation of the police officer representing the police
station. In homicide or murder cases, the name of the victim or
of the complainant shall be in the caption. The heirs or
relatives of the slain victim shall be indicated as "Legal heirs
of deceased (name or person killed)", represented by "(either
the surviving spouse, father or mother)".

a)
b)
c)

All material details that should be found in the


information prepared by the Investigating Prosecutor shall be
stated in the resolution.

a)

b)

In the case of a corporation or judicial entity, its


corporate name or identity shall be indicated and written as
follows. " 'X' Corporation, represented by its (position title),
(name of corporate officer)".
c)
SEC. 39. Case number. - The number of a case shall
indicate the year and month; it was filed and its entry number
in the log book of the office, e.g. 97 (year)A(month)-024(entry
number).
SEC. 40. Designation of offense charged. - For
offenses that are punishable under the Revised Penal Code, the
caption shall set forth the denomination of the offense and the
specific article and paragraph of the statute violated.
Where there is another charge or countercharge in the same
case having one case number or in case of a consolidated
resolution involving two or more criminal cases with two or

a brief summary of the facts of the case;


a concise statement of the issues involved;
and
the findings and recommendations of the
Investigating Prosecutor.

d)

Part 1 shall state the nature of the case as


disclosed in the evidence presented by the
complainant such as his affidavit-complaint,
the affidavit of witnesses and documentary
and physical evidence. The affidavits shall
be numbered in the order of the presentation
of the prosecution witnesses as disclosed in
the list of witnesses appearing in the
information.
As for the documentary
evidence, they shall be alphabetically
marked as they would be marked during the
pre-trial and trial stages of the case.
Part 2 shall contain the version of
complainant
of
the
incident. The
presentation of the complainant's case
should be concise and shall not be cluttered
with details that are not necessary to show
the elements of the offense.
Part 3 shall allege the respondent 1s version
of the incident. This must also be concise.
Part 4 shall contain the discussion, analysis
and evaluation by the prosecutor of the
evidence presented by the complainant and
the respondent, without relying on the
weakness of the defense of the respondent. It
shall also contain the conclusion of the
prosecutor.
The
complainant's
and
respondent's versions of the incident need
not be repeated in this part except to point
out excerpts relating to the existence or
absence of the elements of the crime.
Citations of pertinent laws and jurisprudence

should support the conclusions reached.


Where numerical values are important, the
number shall be written in words and
figures.
SEC. 43. How recommended hail is written. - The
bail recommended in the resolution shall be written in words
and figures.
SEC. 44. Recommended bail. - The bail
recommended in the resolution shall be stated in the
information, written in words and figures, and initialed by the
investigating prosecutor.
SEC. 45. Parties to be furnished with a copy of the
resolution. - The complete names and addresses of the
complainant and the respondent shall be set out at the end of
the resolution after the signature of the investigating
prosecutor and the head of the Prosecutor's Office concerned
under the phrase: "Copy furnished:".
If the parties are represented by counsel and the
latter's appearance is entered formally in the record, the
counsel, not the party, shall be given a copy of the resolution.
SEC. 46. Signature and initials of investigating
prosecutor. - The investigating prosecutor shall sign the
resolution and if the resolution consists of two or more pages,
the prosecutor shall initial all of said pages, excluding the
signature page.
SEC. 47. Records of the case. - The investigating
fiscal shall forward his resolution, together with the complete
records of the case, to the Provincial or City Prosecutor or
Chief State Prosecutor concerned within five (5) days from the
date of his resolution.
SEC. 48. Action of the Provincial or City Prosecutor
or Chief State Prosecutor on resolution. - The Provincial or
City Prosecutor or Chief State Prosecutor concerned shall act
on all resolutions within ten (10) days from receipt thereof by
either approving or disapproving the resolution or returning
the same to the investigating prosecutor for further appropriate
action. immediately after approving or disapproving the
resolution, the Provincial or City Prosecutor or Chief State
Prosecutor concerned shall transmit a copy of the resolution to
the parties.

or Chief State Prosecutor reverses the recommendation in the


resolution of the investigating prosecutor, the former may, by
himself, file the corresponding information or direct any other
assistant prosecutor or state prosecutor, as the case may be, to
do so without need of conducting another preliminary
investigation.
SEC. 50. Approval of pleading by head of
prosecution office. - A pleading prepared by the trial
prosecutor, including exparte motions, shall not be filed in
court without the prior written approval by the Provincial or
City Prosecutor or Chief State Prosecutor, as the case may be,
of said pleading.

SEC. 51. Motion for reinvestigation, where filed. Before the arraignment of the accused, a motion for
reinvestigation of the case may be filed with the
City/Provincial Prosecutor, Provided, That when the case has
been appealed to the Regional State Prosecutor or the
Department of Justice, such motion may be filed, respectively,
with the said offices. After arraignment, said motion may only
be filed with the judge hearing the case.
SEC. 52. Confidentiality of resolutions. - All
resolutions prepared by an Investigating Prosecutor after
preliminary investigation, whether his recommendation be for
the filing or dismissal of the case, shall be held in strict
confidence and shall not be made known to the parties, their
counsel and/or to any unauthorized person until the same shall
have been finally acted upon by the Provincial/City Prosecutor
or his duly authorized assistant and approved for promulgation
and release to the parties.
Violation of the foregoing shall subject the
Investigating Prosecutor or the employee of the office
concerned to severe disciplinary action.
SEC.
53.
Information/Complaint.The
information/complaint shall be personally and directly
prepared by the Investigating Prosecutor or such other
prosecutor designated for the purpose and signed by him or the
complainant, as the case may be. It shall state and contain, in
addition to the requirements of the Rules of Court on the
sufficiency of the allegations in an information or complaint,
the following:
a)

SEC. 49. Reversal by the Provincial or City


Prosecutor or Chief State Prosecutor of resolution of
investigating prosecutor. - If the Provincial or City Prosecutor

b)

the full name and aliases, if any, and address


of the accused;
the age and date of birth of the complainant
or the accused, if eighteen (18) years of age

c)

d)
e)
f)
g)
h)

or below;
the full names and addresses of the parents,
custodian or guardian of the minor
complainant or accused, as the case may be;
the place where the accused is actually
detained;
the full names and addresses of the
complainant and witnesses;
a detailed description of the recovered items,
if any;
the full name and address of the evidence
custodian; and
the bail recommended, if the charge is
bailable.

The Investigating Prosecutor shall certify under oath


that he or, as shown by the record, an authorized officer, had
personally examined the complainant and his witnesses; that
there is reasonable ground to believe that a crime has been
committed and that the accused is probably guilty thereof; that
the accused was informed of the complaint and of the evidence
submitted against him and that he was given an opportunity to
submit controverting evidence; and that he is filing the
complaint or information with the prior authority and approval
of the Provincial/City Prosecutor concerned.
SEC. 54. Documents to be attached to
information/complaint. - An information/complaint that is
filed in court shall, as far as practicable, be accompanied by a
copy of the resolution of the Investigating Prosecutor, the
complainant's affidavit, the sworn statements of the
prosecution's witnesses, the respondent's counter-affidavit and
the sworn statements of his witnesses and such other evidence
as may have been taken into account in arriving at a
determination of the existence of probable cause.
SEC. 55. Promulgation of resolution.- The result of
the preliminary investigation shall be promulgated by
furnishing the parties or their counsel a copy of the resolution
by:
a)
b)

c)

personal service;
registered mail with return card to
complainant, and by ordinary mail to
respondent, if the resolution is for
dismissal of the complaint; or
registered mail with return card to
respondent, and by ordinary mail to
complainant, if the resolution is for
indictment of the respondent.

the
the
the
the
the
the

SEC. 56. Motion for reconsideration. - A motion for


reconsideration may be filed within ten (10) days from receipt
of the resolution. The motion shall be verified, addressed to
the Provincial/City Prosecutor or the Chief State Prosecutor,
and accompanied by proof of service of a copy thereof on the
opposing party and must state clearly and distinctly the
grounds relied upon in support of the motion.
A motion for reconsideration is still part of due
process in the preliminary investigation. The denial thereof is
a reversible error as it constitutes a deprivation of the
respondent's right to a full preliminary investigation
preparatory to the filing of the information against him. The
court therefore may not proceed with the arraignment and trial
pending resolution of the motion for reconsideration.
SEC. 57. Inhibition. - A Prosecutor shall inhibit
himself from conducting a preliminary investigation in a case
wherein a)
b)
c)

he or his wife or child is interested as heir,


legatee, creditor or otherwise; or
he is related to either affinity or to counsel
he has been named counsel. party within the
6th degree of consanguinity or within the 4th
degree; or executor, administrator, guardian,
trustee or

A motion to disqualify or inhibit the Investigating


Prosecutor may be filed with the City/Provincial or Chief State
Prosecutor concerned for just or valid reasons
other than those mentioned above.
SEC. 58. Period to resolve cases under preliminary
investigation. - The following periods shall be observed in the
resolution of cases under preliminary investigation:
a)

b)

c)

The preliminary investigation of complaints


charging a capital offense shall be terminated
and resolved within ninety (90) days from
the date of assignment to the Investigating
Prosecutor.
The preliminary investigation of all other
complaints involving crimes cognizable by
the Regional Trial Courts shall be terminated
and resolved within sixty (60) days from the
date of assignment.
In cases of complaints involving crimes
cognizable by the Metropolitan Trial Courts,
Municipal Trial Courts and Municipal
Circuit Trial Courts, the preliminary

investigation - should the same be warranted


by the circumstances - shall be terminated
and resolved within sixty(60)
days
from the date of assignment to the
Investigating Prosecutor.
In all instances, the total period (from the date of
assignment to the time of actual resolution) that may be
consumed in the conduct of the formal preliminary
investigation shall not exceed the periods prescribed herein.
(PRELIMININARY INVESTIGATION), PART II
(INQUEST)
SECTION 1. Concept. - Inquest is an informal and
summary investigation conducted by a public prosecutor in
criminal cases involving persons arrested and detained without
the benefit of a warrant of arrest issued by the court for the
purpose of determining whether or not said persons should
remain under custody and correspondingly be charged in
court.
SEC. 2. Designation of In quest Officer. The City or
Provincial Prosecutor shall designate the Prosecutors assigned
to inquest duties and shall furnish the Philippine National
Police (PNP) a list of their names and their schedule of
assignments. If, however, there is only one Prosecutor in the
area, all inquest eases shall be referred to him for appropriate
action.
Unless otherwise directed by the City or Provincial
Prosecutor, those assigned to inquest duties shall discharge
their functions during the hours of their designated
assignments and only at the police stations/headquarters of the
PNP in order to expedite and facilitate the disposition of
inquest eases.
SEC. 3. Commencement and termination of inquest.The inquest proceedings shall be considered commenced upon
receipt by the Inquest Officer from the law enforcement
authorities of the complaint/referral documents which should
include:
a)
the affidavit of arrest;
b)
the investigation report;
c)
the statement of the complainant and
witnesses; and
d)
other supporting evidence gathered by
the police in the course of the latter's
investigation of the criminal incident
involving the arrested or detained
person.

The Inquest Officer shall, as far as practicable, cause


the affidavit of arrest and statements/affidavits of the
complainant and the witnesses to be subscribed and sworn to
before him by the arresting officer and the affiants.
The inquest proceedings must be terminated within the
period prescribed under the provisions of Article 125 of the
Revised Penal Code, as amended.
SEC. 4. Documents required in specific cases. - The
Inquest Officer shall, as far as practicable, require the
submission/presentation of the documents listed below, to wit:

Murder, Homicide and Parricide


a)
certified true/machine copy of the
certificate of death of the victim; and
b)
necropsy report and the certificate
of post-mortem examination, if readily
available.
Frustrated or Attempted Homicide, Murder, Parricide and
Physical Injuries
a)
medical
certificate
of
the
complaining witness showing the nature or
extent of the injury;
b)
certification or statement as to
duration of the treatment or medical
attendance; and
c)
certificate or statement as to
duration of incapacity for work.
Violation of the Dangerous Drugs Law (R.A. No.6425, as
amended)
a)
chemistry report or certificate of
laboratory examination duly signed by the
forensic chemist or other duly authorized
officer. If the foregoing documents are not
available, the Inquest Officer may
temporarily rely on the field test results on
the seized drug, as attested to by a PNP
Narcotics Command operative or other
competent person, in which event, the
Inquest Officer shall direct the arresting
officer to immediately forward the seized
drug to the crime laboratory for expert
testing and to submit to the prosecutor's
office the final forensic chemistry report

within five (5) days from the date of the


inquest;
b)
machine copy or photograph of the
buy-bust money; and
c)
affidavit of the poseur-buyer, if
any.

a)
photograph of the confiscated fish,
if readily available; and
b)
certification of the Bureau of
Fisheries and Aquatic Resources.
Violation of the Forestry Law (P.9. No.705)

Theft and Robbery, Violation of the Anti-Piracy and AntiHighway Robbery Law (P.D. No.532) and Violation of the
Anti-Fencing Law (P.D. No.1612)

a)
scale sheets containing the volume
and species of the forest products
confiscated, number of pieces and other
important details such as estimated value of
the products confiscated;
b)
certification of Department of
Environment
and
Natural
Resources/Bureau of Forest Management;
and
c)
seizure receipt.

a) a list/inventory of the articles and items


subject of the offense; and b) statement of
their respective values.
Rape, Seduction and Forcible Abduction with Rape
a)
the medico-legal report (living case
report), if the victim submitted herself for
medical or physical examination.
Violation of the Anti-Carnapping Law (R.A. No.6539)
a)
machine copy of the certificate of
motor vehicle registration;
b)
machine copy of the current
official receipt of payment of
theregistration fees of the subject
motor vehicle; and
c)
other evidence of ownership.
Violation of the Anti-Cattle Rustling Law (P.D. No.533)
a)
machine copy of the cattle
certificate of registration; and
b)
photograph of the cattle, if readily
available.

The submission of the foregoing documents shall not


be absolutely required if there are other forms of evidence
submitted which will sufficiently establish the facts sought to
be proved by the foregoing documents.
SEC. 5. Incomplete documents. - When the
documents presented are not complete to establish probable
cause, the Inquest Officer shall direct the law enforcement
agency to submit the required evidence within the period
prescribed under the provisions of Article 125 of the Revised
Penal Code, as amended. Otherwise, the Inquest Officer shall
order the release of the detained person and, where the inquest
is conducted outside of office hours, direct the law
enforcement agency concerned to file the case with the City or
Provincial Prosecutor for appropriate action.
SEC. 6. Presence of detained person. - The presence
of the detained person who is under custody shall be ensured
during the proceedings.

Violation of Illegal Gambling Law (P.D. No.1602)


a)
b)

gambling paraphernalia; and


cash money, if any.

Illegal Possession of Explosives (P.D. No.1866)


a)
chemistry report duly signed by the
forensic chemist; and
b)
photograph of the explosives, if
readily available.
Violation of the Fisheries Law (P.9. No.704)

However, the production of the detained person


before the Inquest Officer may be dispensed with in the
following cases:
a)
b)
c)
d)

if he is confined in a hospital;
if he is detained in a place under maximum
security;
if production of the detained person will
involve security risks; or
if the presence of the detained person is not
feasible by reason of age, health, sex and
other similar factors.

The absence of the detained person by reason of any


of the foregoing factors shall be noted by the Inquest Officer
and reflected in the record of the case.
SEC. 7. Charges and counter-charges.- All charges
and counter-charges arising from the same incident shall, as
far as practicable, be consolidated and inquested jointly to
avoid contradictory or inconsistent dispositions.
SEC. 8. Initial duty of Inquest Officer. - The Inquest
Officer shall first determine if the arrest of the detained person
was made in accordance with paragraphs
(a)
and (b) of Section 5, Rule 113 of the 1985
Rules on Criminal Procedure, as amended, which provide that
arrests without a warrant may be effected:

of the charge sheet or complaint, affidavits or sworn


statements of the complainant and his witnesses and other
supporting evidence.
SEC. 10. where arrest properly effected. - Should the
inquest Officer find that the arrest was properly effected, the
detained person shall be asked if he desires to avail himself of
a preliminary investigation and, if he does, he shall be made to
execute a waiver of the provisions of Article 125 of the
Revised Penal Code, as amended, with the assistance of a
lawyer and, in case of non-availability of a lawyer, a
responsible person of his choice. The preliminary investigation
may be conducted by the Inquest Officer himself or by any
other Assistant Prosecutor to whom the case may be assigned
by the City or Provincial Prosecutor, which investigation shall
be terminated within fifteen (15) days from its inception.

9
a)

b)

when, in the presence of the arresting officer,


the person to be arrested has committed, is
actually committing, or is attempting to
commit an offense; or
when an offense has in fact just been
committed, and the arresting officer has
personal knowledge of facts indicating that
the person to be arrested has committed it.

For this purpose, the Inquest Officer may summarily


examine the arresting officers on the circumstances
surrounding the arrest or apprehension of the detained person.
SEC. 9. where arrest not properly effected. - Should
the Inquest Officer find that the arrest was not made in
accordance with the Rules, he shall;
a)
b)
c)
d)

recommend the release of the person


arrested or detained;
note down the disposition on the referral
document;
prepare a brief memorandum indicating the
reasons for the action taken; and
forward the same, together with the record
of the case, to the City or Provincial
Prosecutor for appropriate action.

Where the recommendation for the release of the


detained person is approved by the City or Provincial
Prosecutor but the evidence on hand warrants the conduct of a
regular preliminary investigation, the order of release shall be
served on the officer having custody of said detainee and shall
direct the said officer to serve upon the detainee the subpoena
or notice of preliminary investigation, together with the copies

SEC. 11. Inquest preliminary investigation Inquest


Officer shall statements/affidavits of evidence submitted to
him.
proper.- Where the detained person does not opt for a
or otherwise refuses to execute the required waiver, the
proceed with the inquest by examining the sworn the
complainant and the witnesses and other supporting
If necessary, the Inquest Officer shall require the
presence of the complaining witnesses and subject them to an
informal and summary investigation or examination for
purposes of determining the existence of probable cause.
SEC. 12. Meaning of probable cause.- Probable cause
exists when the evidence submitted to the Inquest Officer
engenders a well-founded belief that a crime has been
committed and that the arrested Or detained person is probably
guilty thereof.
SEC. 13. Presence of probable cause.- If the Inquest
Officer finds that probable cause exists, he shall forthwith
prepare the corresponding complaint/information with the
recommendation that the same be filed in court. The
complaint/information shall indicate the offense committed
and the amount of bail recommended, if bailable.
Thereafter, the record of the case, together with
the prepared complaint/information, shall be forwarded to the
City or Provincial Prosecutor for appropriate action.
The complaint/information may be filed by the

Inquest Officer himself or by any other Assistant Prosecutor to


whom the case may be assigned by the City or Provincial
Prosecutor.

complaint/information shall be filed by the City or Provincial


Prosecutor or by any Assistant Prosecutor to whom the case
may be assigned.

SEC. 14. Contents of Information.- The information


shall, among others,
contain:

SEC. 16. Presence at crime scene. - Whenever a dead


body is found and there is reason to believe that the death
resulted from foul play, or from the unlawful acts or omissions
of other persons and such fact has been brought to his
attention, the Inquest Officer shall:

a)

b)
c)
d)
e)
f)
g)

h)

a certification by the filing Prosecutor that


he is filing the same in accordance with the
provisions of Section 7, Rule 112, Rules on
Criminal Procedure, in cases cognizable by
the Regional Trial Court;
the full name and aliases, if any, and address
of the accused;
the place where the accused is actually
detained;
the full names and addresses of the
complainant and witnesses;
a detailed description of the recovered items,
if any;
the full name and address of the evidence
custodian;
the age and date of birth of the complainant
or the accused, if eighteen (18) years of age
or below; and
the full names and addresses of the parents,
custodians or guardians of the minor
complainant or accused, as the case may be.

SEC. 15. Absence of probable cause.- If the Inquest


Officer finds no probable cause, he shall:
a)
b)
c)
d)

recommend the release of the arrested or


detained person;
note down his disposition on the referral
document;
prepare a brief memorandum indicating the
reasons for the action taken; and
forthwith forward the record of the case to
the City or Provincial Prosecutor for
appropriate action.

If the recommendation of the Inquest Officer for the


release of the arrested or detained person is approved, the
order of release shall be served on the officer having custody
of the said detainee.
Should the City or Provincial Prosecutor disapprove
the recommendation of release, the arrested or detained person
shall remain under custody, and the corresponding

a)
b)

c)

d)

e)

forthwith proceed to the crime scene or


place of discovery of the dead person;
cause an immediate autopsy to be conducted
by the appropriate medico-legal officer in
the locality or the PNP medico-legal division
or the NBI medico-legal office, as the case
may be;
direct the police investigator to cause the
taking of photographs of the crime scene or
place of discovery of the dead body;
supervise the investigation to be conducted
by the police authorities as well as the
recovery of all articles and pieces of
evidence found thereat and see to it that the
same are safeguarded and the chain of the
custody thereof properly recorded; and
submit a written report of his finding to the
City or Provincial Prosecutor for appropriate
action.

SEC. 17. Sandiganbayan cases.- Should any


complaint cognizable by the Sandiganbayan be referred to an
Inquest Officer for investigation, the latter shall, after
conducting the corresponding inquest proceeding, forthwith
forward the complete record to the City or Provincial
Prosecutor for appropriate action.
SEC. 18. Recovered articles.- The Inquest Officer
shall see to it that all articles recovered by the police at the
time of the arrest or apprehension of the detained person are
physically inventoried, checked and accounted for with the
issuance of corresponding receipts by the police
officer/investigator concerned.
The said articles must be properly deposited with the
police evidence custodian and not with the police investigator.
The Inquest Officer shall ensure that the items
recovered are duly safeguarded and the chain of custody is
properly recorded.

SEC. 19. Release of recovered articles.- The Inquest


Officer shall, with the prior approval of the City or Provincial
Prosecutor or his duly authorized representative, order the
release of recovered articles to their lawful owner or
possessor, subject to the conditions that:
a)
b)

there is a written request for their release;


the person requesting the release of said
articles is shown to be the lawful owner or
possessor thereof;
c)
the requesting party undertakes under oath
to produce said articles before the court
when so required;
d)
the requesting party, if he is a material
witness to the case, affirms or reaffirms his
statement concerning the case and
undertakes under oath to appear and testify
before the court when so required;
e)
the said articles are not the instruments, or
tools in the commission of the offense
charged nor the proceeds thereof; and
f)
photographs of said articles are first taken
and duly certified to by the
police evidence custodian as accurately representing the
evidence in his custody.
B. DEFINITION; WHEN REQUIRED
RULE 112, Section 1. Preliminary investigation defined;
when required. Preliminary investigation is an inquiry
or proceeding to determine whether there is sufficient
ground to engender a well-founded belief that a crime
has been committed and the respondent is probably
guilty thereof, and should be held for trial.
Except as provided in section 7 of this Rule, a
preliminary investigation is required to be conducted
before the filing of a complaint or information for an
offense where the penalty prescribed by law is at least
four (4) years, two (2) months and one (1) day without
regard to the fine. (1a)
DOJ NPS MANUAL PART III
SECTION 1. Concept of preliminary investigation A preliminary investigation is an inquiry or proceeding to
determine whether there is sufficient ground to engender a
well founded belief that a crime cognizable by the Regional
Trial Court has been committed and that the respondent is
probably guilty thereof and should be held for trial.
A preliminary investigation is essentially a judicial
inquiry since there is the opportunity to be heard, the
production and weighing of evidence, and a decision rendered
on the basis of such evidence. In this sense, the investigating
prosecutor is a quasi-judicial officer.

SEC. 2. Purpose of preliminary investigation. - A


preliminary investigation is intended:
a)

to secure the innocent against hasty,


malicious and oppressive prosecution and to
protect him from an open and public
accusation of a crime and from the trouble,
expense and anxiety of a public trial; and
b)
to protect the State from having to conduct
useless and expensive trials.
SEC. 3. Nature of preliminary investigation. - The
conduct of a preliminary investigation is a substantive right
which the accused may invoke prior to or at least at the time of
plea, the deprivation of which would be a denial of his right to
due process.
RJCL
Section 8. Procedure for Handling Children Exempted
from Criminal Liability. - If it is determined at the initial
contact that the child is 15 years of age or below, the
procedure provided in Section 20, Republic Act No. 9344
shall be observed as follows:
(a) The authority who had the initial contact with the child
shall immediately release the child to the custody of the
mother or father, or the appropriate guardian or
custodian, or in their absence, the nearest relative.
(b) The authority shall immediately notify the local social
welfare and development officer of the taking of the child
into custody.
(c) The local social welfare and development officer
shall, with the consent of the child and the person having
custody over the child, determine the appropriate
intervention programs for the child.
(d) If the child's parents, guardians or nearest relatives
cannot be located, or if they refuse to take custody, the
child may be released to any of the following: a duly
registered nongovernmental or religious organization; a
barangay official or a member of the Barangay Council
for the Protection of Children; a local social welfare and
development officer; or, when and where appropriate,
the Department of Social Welfare and Development.
(e) If the child has been found by the local social
welfare and development office to be abandoned,
neglected or abused by the parents, or if the parents and
the child do not consent to or do not comply with the
prevention program, the Department of Social Welfare
and Development or the Local Social Welfare and
Development Office shall file before the court a petition
for involuntary commitment pursuant to Presidential
Decree No. 603, otherwise known as "The Child and
Youth Welfare Code."
Section 13. Taking Custody of a Child Without a
Warrant. - The law enforcement officer or a private

person taking into custody a child in conflict with the law


without a warrant shall observe the provisions in
Sections 5, 8 and 9 of Rule 113 of the Revised Rules of
Criminal Procedure and shall forthwith deliver the child to
the nearest police station. The child shall be proceeded
against in accordance with Section 7 of Rule 112 of the
Rules of Criminal Procedure.
HASHIM V BONCAN
Nature:Petitionforcertiorariandmandamus
Facts:
Hashimwascaughtredhandedinpossessionof counterfeit
treasurycertificates.Acriminalactionwasfiledagainsthim.
A warrant for his arrest was issued on the strength of the
fiscalsswornstatementthathehad conductedapreliminary
investigationandthat hehad examinedthewitnessesunder
oath. Prior to his arraignment, his counsel filed a motion
asking that the fiscal furnish the clerk of court with the
testimony of the witnesses who testified at the preliminary
investigation,oranextractthereof,aswellasthealleged560
counterfeit treasurycertificateorinthealternative,thatthe
court
immediatelyconductaninvestigation.
Fiscals Position: There is no necessity for the court to
conductanotherpreliminaryinvestigationbecausethe office
ofthefiscalhasalreadyconductedoneinaccordancewiththe
law.
Supreme Court: No need for another preliminary
Investigation.
TherighttoPIisstatutoryandnotconstitutional.Itspurpose
is to secure the innocent against hasty, malicious, and
oppressive prosecutions, and to protect him from open and
public accusation of crime, from the trouble, expenses and
anxiety of public trial, and also to protect the State from
uselessandexpensiveprosecutions.
The new rules on PI were drafted in light of the courts
experiencewherePIsdragonforweeksandevenmonths,so
nowruleswerepromulgatedtomakeitassimpleandspeedy
asisconsistentwiththerightsoftheaccused.As thename
implies, the proceedings are only preliminary, the
investigationjudgeorprosecutoractsuponprobablecauseand
reasonablebeliefandnotuponproofbeyondreasonabledoubt.
It isnot anoccasionforfull and exhaustivedisplayofthe
parties evidence but only for the presentation of such
evidence as may engender a well grounded belief that an
offensehasbeencommittedand thattheaccusedisprobably
guiltythereof.
In this case, a PI was already conducted by the fiscal, the
resultofwhichbecamethebasisofaninformationwhich in
turnwasthebasisforthecourtsdecisiontoissueawarrant.
Toaskforanabstracttestimonyofsuch proceedingsforno
otherpurposethantoscrutinizetheevidencewhichconvinced
theFiscalandtheJudgethattherewasreasonablegroundto
proceedagainstthepetitioner,iscontrarythenatureofPIsasa
briefinvestigation.
TANDOC VS RESULTAN
FACTS: This controversy arose from a heated altercation and

physical assaults amongst neighbors. Based on the collated


complaints of both parties, in October 1980, at the house of
Pacita Tandoc, respondents Cancino, Arnulfo Payopay,
Conrado Payopay, Sr. and several others intruded the sarisari store and house of the former and an altercation ensued.
In the middle of the verbal joust, Arnulfo and Beda Acosta
picked up stones and hurled them unto Pacita, though, the
projectiles instead hit the latters helpers who sustained
physical injuries. Thereafter, Tandocs party filed complaints
against the intruders with the City Fiscal of San Carlos City,
Pangasinan, which sometime November 1980 found
probable cause that all the respondents committed trespass
to dwelling, Arnulfo serious physical injuries and Acosta
slight physical injuries.
Four days later, respondents filed complaints against Tandocs
party with the same fiscals office, however, the latter found
them merely as belated countercharges meriting dismissal,
except the trespass to dwelling charged against Pedro
Tandoc. Displeased with the fiscals resolution, in July
1981, Payopays party directly lodged their complaints with
City Court San Carlos (CCSC), where the criminal cases
initiated by the Tandocs against them are pending.
Subsequently, the CCSC issued several Orders which are the
subject of this Petition for Certiorari, whereby the said
court, after conducting preliminary examination of
Payopays complaints found reasonable ground to believe
that the offenses charged may have been committed by the
accused, herein petitioners. The Tandocs moved for
reconsideration and re-investigation of the complaints by the
city fiscal, insisting that the latter had already evaluated the
same and found no prima facie case.
ISSUE: W/N the CCSC had the power and authority to
conduct anew a preliminary examination of charges, which
already went thru a preliminary investigation (PI) by the
city fiscal who ordered their dismissal.
HELD: Petition is denied, re-investigation is not allowed in
this instance.
RATIO: The policy objective for the conduct of a PI is to
protect the accused from the inconvenience, expense and
burden of defending himself in a formal trial unless
reasonable probability of his guilt shall have been first
ascertained in a fairly summary proceeding by a competent
officer. It is also intended to protect the state from having to
conduct useless and expensive trials.
There are TWO (2) STAGES in a PI; FIRST, the
PRELIMINARY EXAMINATION of the complainant and
his witnesses prior to the arrest of the accused to determine
whether or not there is ground to issue a warrant of arrest;
SECOND, PRELIMINARY INVESTIGATION PROPER,
wherein the accused, after his arrest, is informed of the
complaint filed against him and is given access to the
testimonies and evidence presented, and he is also permitted
to introduce evidence in his favor. The purpose of this stage
of investigation is to determine whether or not the accused
should be released [and the complaint be dismissed or he
should be held for trial].
A PI is inquisitorial in nature and it is not a trial on the merits
of the case and has no purpose except that of determining
whether a crime has been committed and whether there is
probable cause to believe that the accused is guilty thereof,
and it does not place the person against whom it is taken in
jeopardy (as does not constitute a trial on the merits).

Under the Section 9, Rule 112, certain crimes require a


different approach in PI. The rationale for this is as follows.
xxx the withholding of the right of the PI from the accused
in cases triable by the inferior courts involving offenses with
lower penalties than those exclusively cognizable by CFIs,
could not be termed an unjust or unfair distinction. The loss
of time entailed in the conduct of PIs, with the consequent
extension of deprivation of the accused's liberty, in case he
fails to post bail, which at times outlasts the period of the
penalty provided by law for the offense, besides the mental
anguish suffered in protracted litigations, are eliminated
with the assurance of a speedy and expeditious trial for the
accused, upon his arraignment (without having to undergo
the second stage of the PI), and of a prompt verdict on his
guilt or innocence. On the other hand, the so-called first
stage of PI or the preliminary examination, conducted by the
duly authorized officer, as borne out by the examination and
sworn written statement of the complainants and their
witnesses, generally suffices to establish the existence of
reasonable ground to charge the accused with having
committed the offense complained of.
In the case at bar, the offenses charged against petitioners for
"Trespass to Dwelling", "Grave Threats" and "Physical
Injuries" were all within the jurisdiction of the CCSC.
Under the circumstances, the complaints could be filed
directly with the City Court which is empowered to conduct
a preliminary examination for purposes of issuance of
warrants of arrest, and thereafter to proceed with the trial of
the cases on the merits. The PI proper conducted by the
Office of the City Fiscal could have been dispensed with.
Neither did the earlier order of dismissal of the complaints
by the investigating fiscal bar the filing of said complaints
with the city court on the ground of double jeopardy.
As long as the offense charged has not prescribed, the city
court has the power and authority to conduct a preliminary
examination and proceed with the trial of the case properly
within its jurisdiction. The prescriptive period of a crime
depends upon the penalty imposed by law. The prescriptive
period of offenses punishable by arresto mayor is five (5)
years, while crimes punishable by correctional penalties
prescribe in ten (10) years.
DOROMAL VS SANDIGANBAYAN
FACTS:
Quintin S. Doromal, a former Commissioner of the
Presidential Commission on Good Government
(PCGG), for violation of the Anti-Graft and Corrupt
Practices Act (RA 3019), Sec. 3(h), in connection
with his shareholdings and position as president
and director of the Doromal International Trading
Corporation (DITC) which submitted bids to
supply P61 million worth of electronic, electrical,
automotive, mechanical and airconditioning
equipment to the Department of Education,
Culture and Sports (or DECS) and the National
Manpower and Youth Council (or NMYC).
An information was then filed by the
Tanodbayan against Doromal for the said
violation and a preliminary investigation was
conducted.
The petitioner then filed a petition for certiorari

and prohibition questioning the jurisdiction of the


Tanodbayan to file the information without the
approval of the Ombudsman.
The Supreme Court held that the incumbent
Tanodbayan (called Special Prosecutor under the
1987 Constitution and who is supposed to retain
powers and duties NOT GIVEN to the
Ombudsman) is clearly without authority to
conduct preliminary investigations and to direct
the filing of criminal cases with the
Sandiganbayan, except upon orders of the
Ombudsman. Subsequently annulling the
information filed by the Tanodbayan.
A new information, duly approved by the
Ombudsman, was filed in the Sandiganbayan,
alleging that the Doromal, a public officer, being
then a Commissioner of the Presidential
Commission on Good Government, did then and
there wilfully and unlawfully, participate in a
business through the Doromal International
Trading Corporation, a family corporation of
which he is the President, and which company
participated in the biddings conducted by the
Department of Education, Culture and Sports and
the National Manpower & Youth Council, which
act or participation is prohibited by law and the
constitution.
The petitioner filed a motion to quash the
information on the ground that it was invalid
since there had been no preliminary investigation
for the new information that was filed against
him.
The motion was denied by Sandiganbayan
claiming that another preliminary investigation is
unnecessary because both old and new
informations involve the same subject matter.
ISSUES:
Whether or not the act of Doromal would
constitute a violation of the Constitution.
Whether or not preliminary investigation is
necessary even if both informations involve the
same subject matter.
Whether or not the information shall be effected
as invalid due to the absence of preliminary
investigation.
HELD:
Yes, as to the first and second issuses. No, as to
the third issue. Petition was granted by the
Supreme Court.
RATIO:
(1) The presence of a signed document bearing
the signature of Doromal as part of the
application to bid shows that he can rightfully be
charged with having participated in a business
which act is absolutely prohibited by Section 13
of Article VII of the Constitution" because "the
DITC remained a family corporation in which
Doromal has at least an indirect interest."
Section 13, Article VII of the 1987 Constitution
provides that "the President, Vice-President, the
members of the Cabinet and their deputies or

assistants shall not... during (their)


tenure, ...directly or indirectly... participate in any
business.
(2) The right of the accused to a preliminary
investigation is "a substantial one." Its denial over
his opposition is a "prejudicial error, in that it
subjects the accused to the loss of life, liberty, or
property without due process of law" provided by
the Constitution.
Since the first information was annulled, the
preliminary investigation conducted at that time
shall also be considered as void. Due to that fact,
a new preliminary investigation must be
conducted.
(3) The absence of preliminary investigation does
not affect the court's jurisdiction over the case.
Nor do they impair the validity of the information
or otherwise render it defective; but, if there were
no preliminary investigations and the defendants,
before entering their plea, invite the attention of
the court to their absence, the court, instead of
dismissing the information should conduct such
investigation, order the fiscal to conduct it or
remand the case to the inferior court so that the
preliminary investigation may be conducted.
WHEREFORE, the petition for certiorari and
prohibition is granted. The Sandiganbayan shall
immediately remand Criminal Case No. 12893 to
the Office of the Ombudsman for preliminary
investigation and shall hold in abeyance the
proceedings before it pending the result of such
investigation.
COJUANGCO VS SANDIGANBAYAN
FACTS: The Republic of the Philippines (Republic) filed
before the Sandiganbayan a "Complaint for
Reconveyance, Reversion, Accounting, Restitution and
Damages," of the alleged ill-gotten wealth of the
Marcoses which have been invested in the Philippine
Long Distance Telecommunication Corporation (PLDT).
Ramon and Imelda Cojuangco (Spouses Cojuangco)
were subsequently impleaded. The Sandiganbayan
dismissed the complaint with respect to the recovery of
the PLDT shares. The Republic appealed to the
Supreme Court, and the same issued a favorable ruling.
The Republic thereafter filed with the Sandiganbayan a
Motion for the Issuance of a Writ of Execution, praying
for the cancellation of the shares of stock registered in
the name of Prime Holdings and the annotation of the
change of ownership on PTICs Stock and Transfer
Book. The Republic further prayed for the issuance of an
order for PTIC to account for all cash and stock
dividends declared by PLDT in favor of PTIC from 1986
up to the present including compounded interests. The
Sandiganbayan granted the same, except its prayer for
accounting of dividends.
The Republic moved for reconsideration with respect to
the denial of accounting of dividends, which the
Sandiganbayan granted. The Cojuangcos protested,

alleging that the SCs decision did not include in its


dispositive portion the grant of dividends and interests
accruing to the shares adjudicated in favor of the
Republic.
ISSUE: Whether or not the Republic is entitled to the
dividends and interests accruing to the shares despite its
non-inclusion in the dispositive portion of the decision
HELD: The Cojuangcos insist on a literal reading of the
dispositive portion of the SCs Decision, excluding the
dividends, interests, and earnings accruing to the shares
of stock from being accounted for and remitted.
The SC, in directing the reconveyance to the Republic of
the 111,415 shares of PLDT stock owned by PTIC in the
name of Prime Holdings, declared the Republic as the
owner of said shares and, necessarily, the dividends and
interests accruing thereto.
Ownership is a relation in law by virtue of which a thing
pertaining to one person is completely subjected to his
will in everything not prohibited by law or the
concurrence with the rights of another. Its traditional
elements or attributes include jus utendi or the right to
receive from the thing that it produces.
Contrary to the Cojuangcos contention, while the
general rule is that the portion of a decision that
becomes the subject of execution is that ordained or
decreed in the dispositive part thereof, there are
recognized exceptions to this rule, viz: (a) where there is
ambiguity or uncertainty, the body of the opinion may be
referred to for purposes of construing the judgment,
because the dispositive part of a decision must find
support from the decisions ratio decidendi; and (b)
where extensive and explicit discussion and settlement
of the issue is found in the body of the decision.
In the Decision, although the inclusion of the dividends,
interests, and earnings of the 111,415 PTIC shares as
belonging to the Republic was not mentioned in the
dispositive portion of the Courts Decision, it is clear from
its body that what was being adjudicated in favor of the
Republic was the whole block of shares and the fruits
thereof, said shares having been found to be part of the
Marcoses illgotten wealth, and therefore, public money.
WEBB VS DELEON
FACTS:
On June 19, 1994, the National Bureau of
Investigation filed with the DOJ a letter-complaint
charging petitioners Hubert Webb, Michael
Gatchalian, Antonio J. Lejano and 6 other persons
with the crime of Rape and Homicide of Carmela
N. Vizconde, her mother Estrellita NicolasVizconde, and her sister Anne Marie Jennifer in
their home at Number 80 W. Vinzons, St., BF
Homes, Paranaque, Metro Manila on June 30,

1991.
Forthwith, the DOJ formed a panel of prosecutors
headed by Asst Chief State Prosecutor Jovencio
R. Zuno to conduct the preliminary investigation.
Petitioners: fault the DOJ Panel for its finding
of probable cause. They assail the credibility of
Jessica Alfaro as inherently weak and
uncorroborated due to her inconsistencies
between her April 28, 1995 and May 22, 1995
sown statements. They criticize the procedure
followed by the DOJ Panel when it did not
examine witnesses to clarify the alleged
inconsistencies.
charge that respondent Judge Raul de Leon
and respondent Judge Amelita Tolentino issued
warrants of arrest against them without
conducting the required preliminary examination.
Complain about the denial of their
constitutional right to due process and violation
of their right to an impartial investigation. They
also assail the prejudicial publicity that attended
their preliminary investigation.
ISSUES:
(1) Did the DOJ Panel gravely abuse its discretion
in holding that there is probable cause to charge
accused with crime of rape and homicide?
(2) Did respondent judges de Leon and Tolentino
gravely abuse their discretion when they failed to
conduct a preliminary examination before issuing
warrants of arrest against the accused?
(3) Did the DOJ Panel deny them their
constitutional right to due process during their
preliminary investigation?
(4) Did the DOJ Panel unlawfully intrude into
judicial prerogative when it failed to charge
Jessica Alfaro in the information as an accused?
HELD:
(1) NO. Valid determination -- A probable cause
needs only to rest on evidence showing that
more likely than not, a crime has been committed
and was committed by the suspects. Probable
cause need not be based on clear and convincing
evidence of guilt, neither on evidence
establishing guilt beyond reasonable doubt and
definitely, not on evidence establishing absolute
certainty of guilt.

(2) NO. Valid arrest -- In arrest cases, there must


be a probable cause that a crime has been
committed and that the person arrested
committed it.
Section 6 of Rule 112 provides that upon filing
of an information, the RTC may issue a warrant
for the accused.
Clearly then, our laws repudiate the submission
that respondent judges should have conducted
searching examination of witnesses before
issuing warrants of arrest against them.
(3) NO. There is no merit in this contention
because petitioners were given all the
opportunities to be heard.
The DOJ Panel precisely requested the parties to
adduce more evidence in their behalf and for the
panel to study the evidence submitted more fully.
(4) NO.
Petitioner's argument lacks appeal for it lies on
the faulty assumption that the decision whom to
prosecute is a judicial function, the sole
prerogative of courts and beyond executive and
legislative interference.
In truth, the prosecution of crimes appertains to
the executive department whose principal power
and responsibility is to see that our laws are
faithfully executed. A necessary component of
this right is to prosecute their violators.
C. PERSON AUTHORIZED TO CONDUCT
RULE 112 Section 2. Officers authorized to conduct
preliminary investigations.
The following may conduct preliminary investigations:
(a) Provincial or City Prosecutors and their assistants;
(b) Judges of the Municipal Trial Courts and Municipal
Circuit Trial Courts;
(c) National and Regional State Prosecutors; and
(d) Other officers as may be authorized by law.

Their authority to conduct preliminary investigations shall


include all crimes cognizable by the proper court in their
respective territorial jurisdictions.
SEC. 6. Officers Authorized to Conduct Preliminary
Investigation. - The following may conduct a preliminary
investigation;2
a)Provincial or City Prosecutors and their assistants;
b)
Judges of Municipal Trial Courts and
Municipal Circuit Trial Courts;
c)
National and Regional State Prosecutors;
and
d)
Other officers as may be authorized by law.3
Their
authority
to
conduct
preliminary
investigation shall include all crimes cognizable
by the proper court in their respective territorial
jurisdiction.
VELASCO VS CASACLANG
Before the court is a petition for Certiorari and
Prohibition to annul the Order dated June 29, 1993 of
the Deputy Ombudsman for AFP in OMB Case No. 0-900296, denying petitioners motion to quash, and the
Order dated July 13, 1993, denying petitioners Motion for
Reconsideration, both of which Orders are attacked for
having been issued with grave abuse of discretion.
The antecedent facts that matter can be culled, as
follows:
By virtue of Assignment Order No. 89-846 dated
June 5, 1989 of the Commission on Audit, COA Audit
Examiners Priscilla G. Cruz and Virginia G. Pantoja
conducted a special audit of selected transactions of the
Armed Forces of the Philippines (AFP) Logistics
Command, covering the period from January 1988 to
May 1989.
In their Memorandum Report, dated March 13,
1990, to the Chairman of the Commission on Audit, the
said COA audit examiners reported their findings, as
follows:
xxx
6. The propriety of the procurement of 28,432 pieces of
meat can (stainless steel) amounting to P3,502,432
were [sic] of doubtful validity. The transaction cycle from
preparation of purchase orders to acceptance of
delivered items were [sic] all completed in just one day December 29, 1988 and the three winning bidders have
[sic] common incorporators. Furthermore, while stainless
can are [sic] for distribution to CAFGU at P126 per unit,
previous orders of P5 million were for aluminum can at
2 Par. 1, Sec. 2, Rule 112, supra.
3 The Special Prosecution Officers and Graft

Investigation Officers incases cognizable by


the Office of the Ombudsman and the
COMELEC officials in
cases involving
violations of the Election Code, PCGG Officers

P89/can intended to regular military units. Under the


case, CAFGU expense was more than P1 million.
On December 15, 1989, the approved Procurement
Directive No. QM-0156-88 was issued for the purchase
of 28,432 pieces of Meat Can, Austenitic Steel
(Stainless Steel), local manufacture and brand new for
the use of CAFGU. On December 27, 1988, the public
bidding was held at Camp Aguinaldo with nine
participating bidders. The sequence of events cast doubt
on the propriety of the claim. Consider:
a) December 21, 1988 - the bids were opened.
b) December 27, 1988 - the date the Bidder Tender
Sheets were submitted by the participating bidders.
The stamped date showed that the bids were opened six
days before the submission of the bids.
The result of the bidding showed that four suppliers had
the same lowest bid for P126 per unit of meat can,
hence the required 28,432 pieces of meat can were
divided between the four winning bidders at P895,608
per supplier.
c)
The transaction cycle of
procuring meat cans from four
suppliers were completed in
just one day - December 29,
1988, the last working day of
the year.
The fourth meat can supplier, Michelle-V Mgf. and
Trading is not registered with the SEC.
Tabulation also show that the capital of Mitrick is only
P500,000 whereas the PO issued to it amounted to
P895,800.
While the foregoing data showed that the procurement
of 28,432 pieces of stainless steel meat can was
urgently needed as indicated in the speedy processing
of documents, records however, showed that 24,640
meat cans were issued to various Military Support
Points (MSPs) only on May 9, 1989 or four months after
delivery, thus belying its urgency.
Analysis also shows that two more Purchase Orders
were issued to Trojan Manufacturing and Marketing Inc.
for the procurement of 51,568 and 7,485 meat cans on
October 7, 1988 and December 16, 1987, respectively.
Both Pos were for aluminum meat cans with AFP
marking at P89 per unit totaling to P4,589,552 and
P666,165, respectively. The provision of stainless meat
cans for CAGFU instead of aluminum meat cans
intended for regular military service units resulted to
additional expenses of P1 million arrived as follows:
Unit Cost - Stainless Can - P 126
Unit Cost - Aluminum Can - 89
Price Difference 37
Quantity Ordered [x] 28,432
Total Additional Expenses for AFP - P 1,051,984
On May 6, 1993, the same audit examiners filed
with the Office of the Ombudsman a Joint AffidavitComplaint deploring the aforesaid transactions dubbed
as anomalous and highly irregular.
On May 27, 1993, respondent Deputy
Ombudsman for the Military issued an Order in OMB
Case No. 0-90-0296, entitled Commission on Audit, et
al. vs. BGen. Buenaventura Tabo, et al., to wit:
It appearing from the affidavit-complaint dated 6 May

1993 filed by complainant/s to be sufficient in form and


substance, respondent/s is/are hereby directed to file
within ten (10) days from receipt hereof his/her/their
counter-affidavit/s and other controverting evidence with
proof of service upon complainant/s who may file
his/her/their reply/ies within ten (10) days from receipt of
the same. Failure of the respondent/s to do so shall be
construed as a waiver of his/her/their right/s to be heard
and the preliminary investigation of this case shall
proceed accordingly, thereafter, the same shall be
deemed submitted for resolution. (Underscoring,
supplied)
SO ORDERED.
On June 6, 1993, petitioner Laura Velasco
presented to respondent Deputy Ombudsman a motion
to direct the complainants to particularize the offenses
charged in OMB Case No. 0-90-0296.
On June 17, 1993, the respondent Deputy
Ombudsman, without a word from the COA, issued an
Order stating that petitioner was being charged with a
violation of Section 3, subparagraphs (e) and (g) of R.A.
3019.
On June 25, 1993, petitioner interposed a Motion
To Quash theorizing that the complaint and its annexes
did not charge an offense. But on June 29, 1993, without
any opposition from COA, the respondent Deputy
Ombudsman denied petitioners Motion to Quash,
ratiocinating that Administrative Order No. 07, Section 4
(d) of the Ombudsman, does not allow such a motion.
With the denial of her unopposed Motion for
Reconsideration, petitioner found her way to this Court
via the instant Petition; contending, that:
I.
RESPONDENT OMBUDSMAN IS ACTING WITHOUT
OR IN EXCESS OF JURISDICTION OR WITH
GRAVE ABUSE OF DISCRETION IN HOLDING
THAT PETITIONER IS CHARGED UNDER
SECTION 3 (e) and (g) OF R.A. 3019 AND IN
DENYING THE MOTION TO QUASH;
II.
PETITIONER HAS NO PLAIN, SPEEDY AND
ADEQUATE REMEDY IN THE ORDINARY
COURSE OF LAW EXCEPT THE PRESENT
PETITION.
The Petition is not impressed with merit.
In her Replypetitioner Laura Velasco questioned
the authority of respondent Deputy Ombudsman to
conduct preliminary investigation, arguing that pursuant
to Section 11, par. (4), subpar. (a) of R.A. 6770, the
Office of the Special Prosecutor is vested with the power
and authority to conduct preliminary investigation and to
prosecute criminal cases falling within the jurisdiction of
the Sandiganbayan and Section 17 of P.D. 1630
provides that the Office of Tanodbayan (now Office of
the Special Prosecutor) has the exclusive authority to
conduct preliminary investigation in all cases cognizable
by the Sandiganbayan.
Under Section 2, Rule 112 of the 1985 Rules of
Criminal Procedure, as amended:
The following may conduct preliminary investigation:
(a) Provincial or city fiscals and their assistants;
(b) Judges of the Municipal Trial Courts and Municipal

Circuit Trial Courts;


(c) National and Regional state prosecutors; and
(d) Such other officers as may be authorized by law.
On the other hand, Section 15, paragraph (1) of
R.A. 6770, otherwise known as The Ombudsman Act ,
provides:
The Office of the Ombudsman shall have the following
powers, functions, and duties:
(1) Investigate and prosecute on its own or on complaint
by any person, any act or omission of any public officer
or employee, office, or agency, when such act or
omission appears to be illegal, unjust, improper or
inefficient. It has a primary jurisdiction over cases
cognizable by the Sandiganbayan and, in the exercise of
this primary jurisdiction, it may take over, at any stage,
from any investigatory agency of Government, the
investigation of such cases.
In light of the aforequoted provisions of law in
point, it is beyond cavil that the Ombudsman and his
Deputies are, within legal contemplation, other officers
authorized by law to conduct preliminary investigation.
In Enrique Zaldivar vs. Sandiganbayan, [if this
Court succinctly ruled, that :
Under the 1987 Constitution, the Ombudsman (as
distinguished from the incumbent Tanodbayan) is
charged with the duty to:
Investigate on its own, or on complaint by any person,
any act or omission of any public official, employee,
office or agency, when such act or omission appears to
be illegal, unjust improper, or inefficient. (Sec. 13, par. 1)
The Constitution likewise provides that:
The existing Tanodbayan shall hereafter be known as
the Office of the Special Prosecutor. It shall continue to
function and exercise its power as now or hereafter may
be provided by law, except those conferred on the Office
of the Ombudsman created under this Constitution.
(Article XI, Section 7) [Underscoring, supplied].
Now then, in as much as the aforementioned duty is
given to the Ombudsman, the incumbent Tanodbayan
(called Special Prosecutor under the 1987 Constitution
and who is supposed to retain powers and duties NOT
GIVEN to the Ombudsman) is clearly without authority to
conduct preliminary investigations and to direct the filing
of criminal cases with the Sandiganbayan, except upon
orders of the Ombudsman. This right to do so was lost
effective February 2, 1987. From that time, he has been
divested of such authority.
Under the present Constitution, the Special
Prosecutor...is a mere subordinate of the Tanodbayan
(Ombudsman) and can investigate and prosecute cases
only upon the latters authority or order... Even his
original power to issue subpoena, which he still claims
under Section 10 (d) of PD 1630, is now deemed
transferred to the Ombudsman, who may, however,
retain it in the Special Prosecutor in connection with the
cases he is ordered to investigate. [Underscoring,
supplied]
So also, Section 3 of Administrative Order No. 07,
otherwise known as The Rules of Procedure of the
Office of the Ombudsman, published in the May 1, 1990
issue of Manila Bulletin, states that:
Preliminary investigation may be conducted by any of

the following:
1) Ombudsman Investigators;
2) Special Prosecuting Officers;
3) Deputized Prosecutors;
4) Investigating Officials authorized by law to conduct
preliminary investigation;
5) Lawyers in the government service, so designated by
the Ombudsman. (Underscoring, supplied)
Neither can we discern any tenability in petitioners
reliance on Section 17 of P.D. 1630. Said law, which took
effect on July 18, 1979, was deemed abrogated by
Section 7, Article XI of the 1987 Philippine Constitution,
which reads:
The existing Tanodbayan shall hereafter be known as
the Office of the Special Prosecutor. It shall continue to
function and exercise its powers as now and hereafter
may be provided by law, except those conferred on the
Office of the Ombudsman created under this
Constitution.
(Underscoring supplied)
The powers, functions and duties of the Office of
the Ombudsman are clearly provided in Section 13,
Article XI of the 1987 Charter, as follows:
(1) [to] investigate on its own, or on complaint by any
person, any act or omission of any public official,
employee, office or agency, when such act or omission
appears to be illegal, unjust, improper, or inefficient.
(2) [to] direct, upon complaint or at its own instance, any
public official or employee of the Government, or any
subdivision, agency or instrumentality thereof, as well as
of any government-owned or controlled corporation with
original charter, to perform and expedite any act or duty
required by law, or to stop, prevent, and correct any
abuse or impropriety in the performance of duties.
xxx
In line with the aforestated constitutional
provisions, then President Corazon C. Aquino signed
Executive Order No. 24 limiting the Special Prosecutors
authority, thus:
Section 2 - The Office of the Special Prosecutor shall
exercise powers presently exercised by the Tanodbayan
except those conferred on the Office of the Ombudsman
under the Constitution. (Underscoring, supplied)
Then, too, while Section 17 of P.D. No. 1630
provides, that:
The Office of Tanodbayan (now, Office of the Special
Prosecutor) shall have the exclusive authority to conduct
preliminary investigation of all cases cognizable by the
Sandiganbayan...;
Section 11, subparagraph 4 (c) of R.A. No. 6770, states,
that:
The Office of the Special Prosecutor shall, under the
supervision and control and upon the authority of the
Ombudsman, have the following powers:
(a) To conduct preliminary investigation and prosecute
criminal cases within the jurisdiction of the
Sandiganbayan.
xxx
Unmistakably, the exclusive authority of the Office
of the Tanodbayan (now Office of the Special
Prosecutor) under P. D. No. 1630 to conduct preliminary
investigation was not included in Section 11,

subparagraph 4 (a) of R.A. No. 6770. Consequently, the


irresistible conclusion that can be drawn unerringly from
the aforementioned statutory amendments is that the
exclusive authority of the Office of Special Prosecutor to
conduct preliminary investigation has become a thing of
the past, and the Office of the Ombudsman has the
power to investigate and to conduct preliminary
investigation.
Corollary to the investigative power of the Office of
the Ombudsman is the authority to lay down its own
rules of procedure, as gleanable from the following
provisions of Section 13, subparagraph (8), Article XI of
the 1987 Constitution:
The Office of the Ombudsman shall have the ... power ...
[to]:
(8) Promulgate its rules of procedure and exercise such
other powers or perform such functions or duties as may
be provided by law.
and Section 18 of R.A. No. 6770:
Rules of Procedure - (1) The Office of the Ombudsman
shall promulgate its rules of procedure for the effective
exercise or performance of its powers, functions and
duties.
All things studiedly considered, we are therefore
of the opinion, and so rule, that the respondent Deputy
Ombudsman has the power and authority to conduct
preliminary investigation under the attendant facts and
circumstances.
Similarly untenable is petitioners contention that
respondent Deputy Ombudsman could not, on his own
initiative, determine what offense or offenses subject
Joint Affidavit-Complaint and its annexes charge.
With the vast powers vested in him by law,
respondent Deputy Ombudsman is definitely with
authority and competence to look into and find out motu
proprio the nature of the accusation embodied in the
said Joint Affidavit-Complaint endorsed to him for the
indictment of the herein petitioner and her corespondents under Rep. Act 3019 otherwise known as
The Anti-Graft and Corrupt Practices Act.
In accordance with Section 15, subparagraph (10)
of R.A. No. 6770, the powers, functions and duties of the
Ombudsman may be delegated to his Deputies, to
ensure effective exercise of his powers. Embraced in the
broad powers of the Ombudsman is the discretionary
power to define, supervise and control the methodology
and procedure his office may adopt in connection with its
investigative power. Comformably, the respondent
Deputy Ombudsman has the authority to decide what
offense or offenses to charge on the basis of the
evidence before him.
Section 1, Rule 112 of the 1985 Rules on Criminal
Procedure,
as
amended,
defines
preliminary
investigation as an inquiry or proceeding for the purpose
of determining whether there is sufficient ground to
engender a well-grounded belief that a crime cognizable
by the Regional Trial Court has been committed and that
the respondent is probably guilty thereof, and should be
held for trial.
To be sure, the respondent Deputy Ombudsman
has direct supervision and control over the preliminary
investigation conducted by him. Absent any grave abuse

of discretion tainting it, his action is not subject to judicial


review.
The propriety of endowing the Ombudsman with
wide latitude of ministerial and discretionary powers
emanates from the vitality and importance of his
constitutional duty and function - to protect the people
from inefficiency, red tape, mismanagement, fraud, and
corruption in the government.
It bears stressing that the determination of the
existence or absence of a sustainable basis and ground
for holding the respondent (petitioner here) for trial is
within the primary jurisdiction of the Ombudsman to
undertake.
Taking into account the same Affidavit-Complaint
dated May 6, 1993, together with the auditors
Memorandum Report and findings sent to him by the
Commission on Audit, the respondent Deputy
Ombudsman cannot be faulted for conducting the
requisite preliminary investigation against petitioner
Laura Velasco and the other respondents in OMB Case
No. 0-90-0296. Sufficient basis therefor has been duly
laid.
Neither is the Court persuaded by petitioners
imputation to respondent Deputy Ombudsman of grave
abuse of discretion in denying her motion to quash and
motion for reconsideration.
Section 4 (d) of Administrative Order No. 07,
reads:
Procedure - The preliminary investigation of cases
falling under the jurisdiction of the Sandiganbayan and
Regional Trial Courts shall be conducted in the manner
prescribed in Section 3, Rule 112 of the Rules of Court,
subject to the following provisions:
xxx
(d) No motion to dismiss shall be allowed except for lack
of jurisdiction.
While Section 3, Rule 112 of the 1985 Rules on
Criminal Procedure, as amended, provides:
Procedure - Except as provided for in Section 7 hereof,
no complaint or information for an offense cognizable by
the Regional Trial Court shall be filed without a
preliminary investigation having been first conducted in
the following manner:
xxx
(b) Within ten (10) days after the filing of the complaint,
the investigating officer shall either dismiss the same if
he finds no ground to continue with the inquiry, or issue
a subpoena to the respondent, attaching thereto a copy
of the complaint, affidavits and other supporting
documents. Within ten (10) days from receipt thereof,
the respondent shall submit counter-affidavits and other
supporting documents. He shall have the right to
examine all other evidence submitted by the
complainant.
xxx
(f) Thereafter, the investigation shall be deemed
concluded, and the investigating officer shall resolve the
case within ten (10) days therefrom. Upon the evidence
thus adduced, the investigating officer shall determine
whether or not there is sufficient ground to hold the
respondent for trial.
Verily, the respondent Deputy Ombudsman erred

not in denying the motion to quash and motion for


reconsideration interposed by petitioner in the said case.
He acted thereupon according to applicable provisions of
the Revised Rules of Court and Administrative Order No.
07 of the Ombudsman. Section 4 (d) of said
administrative order disallows a motion to quash (or
dismiss) except on the ground of lack of jurisdiction.
Here, no absence of jurisdiction is perceived.
Following Section 3 of Rule 112 supra, petitioner
was ordered to submit her counter-affidavit within ten
(10) days from service of the subpoena upon her.
The constitutional prescript of promptness of
action patently foremost in his mind, respondent Deputy
Ombudsman must have been impelled to afford the case
against petitioner speediest resolution possible. To the
fore is Section 12, Article XI of the 1987 Philippine
Constitution, to wit:
The Ombudsman and his Deputies, as protectors of the
people, shall act promptly on complaints filed in any
form or manner against public officials or employees of
the government, or any subdivision, agency or
instrumentality thereof, including government owned and
controlled corporations, and shall, in appropriate cases,
notify the complainants of the action taken and the result
thereof. (Underscoring, supplied)
The aforecited constitutional provision directs the
Ombudsman and his Deputies to act promptly on
complaints. The forms of the complaint or manner of
bringing the same should not deter action thereon with
dispatch and sense of urgency.
It is noteworthy and significant, however, that
notwithstanding several proddings and warnings,
petitioner preoccupied herself with the filing of a motion
to quash the Complaint in lieu of her counter-affidavit, so
much so that respondent Deputy Ombudsman had no
alternative but to declare the waiver of petitioners right to
be heard and to preliminary investigation.
What is more, petitioner had a plain, speedy and
adequate remedy by the simple expedient of sending in
her counter-affidavit. Irrefutably, a counter-affidavit would
have been a plain, speedy and adequate remedy since
petitioner could have sought in that pleading the quashal
of the complaint against her, and could have also stated
therein her theory and protestation of innocence. Such
an approach would have been a more expeditious and
effective remedy as within ten (10) days from submission
of petitioners counter-affidavit, the respondent Deputy
Ombudsman would have been duty bound to determine
whether or not there is sufficient ground to hold the
respondent (petitioner in this case) for trial.
WHEREFORE, for want of merit, the Petition at bar is
hereby DISMISSED, and the assailed Orders, dated
June 29, 1993, and July 13, 1993, respectively, in OMB
Case No. 0-90-0296 UPHELD.
BALGOS VS SANDIGANBAYAN
FACTS
- Balgos et al were charged with violation of Section 3(c) of RA 3019, otherwise known
as the Anti-Graft and Corrupt Practice Act, as amended, in an information that was
filed with the Sandiganbayan by the Special Prosecutor which was approved by the
Deputy Tanodbayan, after a preliminary investigation.
- Lim, the plaintiff and prevailing party in Civil Case No. 4047 filed a complaint for
rescission of the sale of the car by Juanito Ang to private respondent Leticia AcostaAng for being allegedly in fraud of creditors. The said complaint was filed with the RTC

of Nueva Vizcaya. On the same day, petitioners filed a motion for reinvestigation in the
Tanodbayan. The same was granted.
- The Tanodbayan ordered to dismiss the case for lack of merit and to withdraw the
Information filed in Criminal Case No. 11414 as soon as possible in the interest of
justice.
- Tanodbayan filed with the Sandiganbayan a motion to withdraw the information
against petitioners. This was denied.
- BAlgos et al filed a motion to suspend proceedings in the criminal case against them
on the ground of the existence of a prejudicial question in Civil Case No. 5307. This
was likewise denied by the Sandiganbayan.
ISSUE
WON the denial by the Sandiganbayan of the motion to withdraw the information and
of another motion to suspend proceedings on the ground of a prejudicial question in a
pending civil action constitute a grave abuse of discretion.
HELD
NO.
- While the public prosecutor has the sole direction and control in the prosecution of
offenses, once the complaint or information is filed in court, the court thereby acquires
jurisdiction over the case and all subsequent actions that may be taken by the public
prosecutor in relation to the disposition of the case must be subject to the approval of
the said court. Before a re-investigation of the case may be conducted by the public
prosecutor, the permission or consent of the court must be secured. And if after such
reinvestigation the prosecution finds a cogent basis to withdraw the information or
otherwise cause the dismissal of the case, such proposed course of action must be
addressed to the sound discretion of the court. - The only instance when the appellate
court should stay the hand of the trial court in such cases is when it is shown that the
trial court acted without jurisdiction or in excess of its jurisdiction or otherwise
committed a grave abuse of discretion amounting to such lack or excess of jurisdiction.
- Petitioners are public officers charged with having violated Section 3(c) of RA 3019,
for evident bad faith and manifest partiality in enforcing the writ of execution in Civil
Case No. 4047 against a Mustang car registered in the name of Leticia Acosta-Ang
(complainant) who is not the judgment debtor thereby causing undue injury to said
complainant and giving unwarranted benefits to the judgment creditor in said case.
- Upon reinvestigation of the criminal case by the Tanodbayan, he found evidence
tending to show that the sale of said car to the complainant by Juanito Ang, the
judgment debtor, was a sham intended to defraud his creditors; that the deed of
absolute sale which ostensibly was executed before a notary public appeared to be
fictitious inasmuch as the entry of the document in the notarial register of said notary
public on said date referred to a catering contract of other parties; that the certificate of
registration of the car was issued to complainant only on June 13, 1984 which showed
that the document of sale was actually executed only on or about the same date, that
is, seven days after Juanito Ang received copy of the adverse decision in Civil Case
No. 4047; and that upon the execution of the judgment, the car was found in the
possession of Alvin, the son of Juanito Ang, who admitted that the car belonged to his
father by showing the receipt of its repair in the name of Juanito Ang. This is the basis
of the motion for withdrawal of the information of the Tanodbayan.
- The respondents are aware that the complainant is not a party to the civil case filed
by the creditor against spouses Juanito and Lydia Ang and that a writ of execution
cannot be implemented validly against one who is not a party to the action. All these,
coupled with the under haste in which the levy on the Mustang car was made without
first ascertaining the true owner thereof demonstrate quite convincingly the evident
bad faith and manifest partiality of the respondents, thereby giving unwarranted
benefits to the judgment creditor to the damage and prejudice of the complainant.
- Although at the reinvestigation, the Tanodbayan was persuaded that in fact the sale
of the car to Leticia Ang was fraudulent, this did not necessarily clear petitioners of the
aforesaid Anti-Graft charge against them. Still the burden is on the petitioners to
establish that they acted in good faith in proceeding with the execution on the car even
they were presented evidence tending to show it did not belong to Juanito Ang
anymore.
- The denial of the motion to suspend the criminal proceedings on the ground of the
pendency of a prejudicial question in Civil Case No. 5307 is well taken. The doctrine of
prejudicial question comes into play usually in a situation where a civil action and a
criminal action are both pending and there exists in the former an issue which must be
preemptively resolved before the criminal action may proceed, because whatsoever
the issue raised in the civil action is resolved would be determinative juris et jure of the
guilt or innocence of the accused in the criminal case.
- The pending civil case for the annulment of the sale of the car to Leticia Ang is not
determinative of the guilt or innocence of the petitioners for the acts allegedly
committed by them in seizing the car. Even if in the civil action it is ultimately resolved
that the sale was null and void, it does not necessarily follow that the seizure of the car
was rightfully undertaken. The car was registered in the name of Leticia Ang six
months before the seizure. Until the nullity of the sale is declared by the courts, the
same is presumptively valid. Thus, petitioners must demonstrate that the seizure was
not attended by manifest bad faith in order to clear themselves of the charge in the
criminal action.
Dispositive The petition is DENIED for lack of merit and the restraining order dated

June 6, 1989 is hereby lifted. No costs.

D. SCOPE
PADERANGGA VS DRILON

Facts: On 16 October 1986, an information for multiple


murder was filed in the Regional Trial Court, Gingoog
City, against Felipe Galarion, Manuel Sabit, Cesar Sabit,
Julito Ampo, Eddie Torion, John Doe, Peter Doe and
Richard Doe, for the deaths on 1 May 1984 of Renato
Bucag, his wife Melchora Bucag, and their son Renato
Bucag II. Venue was, however, transferred to Cagayan
de Oro City per Administrative Matter 87- 2-244. Only
Felipe Galarion was tried and found guilty as charged.
The rest of the accused remained at large. Felipe
Galarion, however, escaped from detention and has not
been apprehended since then. In an amended information
filed on 6 October 1988, Felizardo Roxas, alias "Ely
Roxas," "Fely Roxas" and "Lolong Roxas," was included
as a co-accused. Roxas retained Atty. Miguel P.
Paderanga as his counsel. As counsel for Roxas,
Paderanga filed, among others, an Omnibus Motion to
dismiss, to Quash the Warrant of Arrest and to Nullify
the Arraignment on 14 October 1988. The trial court in
an order dated 9 January 1989, denied the omnibus
motion but directed the City Prosecutor "to conduct
another preliminary investigation or reinvestigation in
order to grant the accused all the opportunity to adduce
whatever evidence he has in support of his defense." In
the course of the preliminary investigation, through a
signed affidavit, Felizardo Roxas implicated Atty.
Paderanga in the commission of the crime charged. The
City Prosecutor of Cagayan de Oro City inhibited
himself from further conducting the preliminary
investigation against Paderanga at the instance of the
latter's counsel, per his resolution dated 7 July 1989. In
his first indorsement to the Department of Justice, dated
24 July 1989, said city prosecutor requested the
Department of Justice to designate a state prosecutor to
continue the preliminary investigation against
Paderanga. In a resolution dated 6 September 1989, the
State Prosecutor Henrick F. Gingoyon, who was
designated to continue with the conduct of the
preliminary investigation against Paderanga, directed the
amendment of the previously amended information to
include and implead Paderanga as one of the accused
therein. Paderanga moved for reconsideration,
contending that the preliminary investigation was not yet
completed when said resolution was promulgated, and
that he was deprived of his right to present a
corresponding counter-affidavit and additional evidence
crucial to the determination of his alleged "linkage" to
the crime charged. The motion was, however, denied by
Gingoyon in his order dated 29 January 1990. From the
aforesaid resolution and order, Paderanga filed a Petition
for Review with the Department of Justice. Thereafter,
he submitted a Supplemental Petition with
Memorandum, and then a Supplemental Memorandum

with Additional Exculpatory/Exonerating Evidence


Annexed, attaching thereto an affidavit of Roxas dated
20 June 1990 and purporting to be a retraction of his
affidavit of 30 March 1990 wherein he implicated
Paderanga. On 10 August 1990, the Department of
Justice, through Undersecretary Silvestre H. Bello III,
issued Resolution 648 dismissing the said petition for
review. His motion for reconsideration having been
likewise denied, Paderanga then filed the petition for
mandamus and prohibition before the Supreme Court.

rather than prosecution; (i) Where the charges are


manifestly false and motivated by the lust for vengeance;
and (j) When there is clearly no prima facie case against
the accused and a motion to quash on that ground has
been denied. A careful analysis of the circumstances
obtaining in the present case, however, will readily show
that the same does not fall under any of the aforesaid
exceptions.
II.

Issue: Whether there is no prima facie evidence, or


probable cause, or sufficient justification to hold
Paderangato a tedious and prolonged public trial.
Held: A preliminary investigation is defined as an
inquiry or proceeding for the purpose of determining
whether there is sufficient ground to engender a well
founded belief that a crime cognizable by the Regional
Trial Court has been committed and that the respondent
is probably guilty thereof, and should be held for trial.
The quantum of evidence now required in preliminary
investigation is such evidence sufficient to "engender a
well founded belief" as to the fact of the commission of
a crime and the respondent's probable guilt thereof. A
preliminary investigation is not the occasion for the full
and exhaustive display of the parties' evidence; it is for
the presentation of such evidence only as may engender
a well grounded belief that an offense has been
committed and that the accused is probably guilty
thereof. Preliminary investigation is generally
inquisitorial, and it is often the only means of
discovering the persons who may be reasonably charged
with a crime, to enable the fiscal to prepare his
complaint or information. It is not a trial of the case on
the merits and has no purpose except that of determining
whether a crime has been committed and whether there
is probable cause to believe that the accused is guilty
thereof, and it does not place the person against whom it
is taken in jeopardy. The institution of a criminal action
depends upon the sound discretion of the fiscal. He has
the quasi-judicial discretion to determine whether or not
a criminal case should be filed in court. Hence, the
general rule is that an injunction will not be granted to
restrain a criminal prosecution. The case of Brocka, et al.
vs. Enrile, et al. cites several exceptions to the rule, to
wit: (a) To afford adequate protection to the
constitutional rights of the accused; (b) When necessary
for the orderly administration of justice or to avoid
oppression or multiplicity of actions; (c) When there is a
prejudicial question which is sub-judice; (d) When the
acts of the officer are without or in excess of authority;
(e) Where the prosecution is under an invalid law,
ordinance or regulation; (f) When double jeopardy is
clearly apparent; (g) Where the court has no jurisdiction
over the offense; (h) Where it is a case of persecution

PROCEDURE
IN
CASE
WHERE
PRELIMINARY
INVESTIGATION
REQUIRED
A. IN CASE COGNIZIBLE BY RTC
1. CONDUCTED BY PROSECUTOR
RULE 112

Section 3. Procedure. The preliminary


investigation shall be conducted in the
following manner:
(a) The complaint shall state the address of the
respondent and shall be accompanied by the
affidavits of the complainant and his witnesses,
as well as other supporting documents to
establish probable cause. They shall be in such
number of copies as there are respondents,
plus two (2) copies for the official file. The
affidavits shall be subscribed and sworn to
before any prosecutor or government official
authorized to administer oath, or, in their
absence or unavailability, before a notary
public, each of who must certify that he
personally examined the affiants and that he is
satisfied that they voluntarily executed and
understood their affidavits.
(b) Within ten (10) days after the filing of the
complaint, the investigating officer shall either
dismiss it if he finds no ground to continue with
the investigation, or issue a subpoena to the
respondent attaching to it a copy of the
complaint and its supporting affidavits and
documents.
The respondent shall have the right to examine
the evidence submitted by the complainant
which he may not have been furnished and to
copy them at his expense. If the evidence is
voluminous, the complainant may be required
to specify those which he intends to present
against the respondent, and these shall be
made available for examination or copying by
the respondent at his expense.
Objects as evidence need not be furnished a
party but shall be made available for
examination, copying, or photographing at the

expense of the requesting party.


(c) Within ten (10) days from receipt of the
subpoena with the complaint and supporting
affidavits and documents, the respondent shall
submit his counter-affidavit and that of his
witnesses and other supporting documents
relied upon for his defense. The counteraffidavits shall be subscribed and sworn to and
certified as provided in paragraph (a) of this
section, with copies thereof furnished by him to
the complainant. The respondent shall not be
allowed to file a motion to dismiss in lieu of a
counter-affidavit.
(d) If the respondent cannot be subpoenaed, or
if subpoenaed, does not submit counteraffidavits within the ten (10) day period, the
investigating officer shall resolve the complaint
based on the evidence presented by the
complainant.
(e) The investigating officer may set a hearing
if there are facts and issues to be clarified from
a party or a witness. The parties can be
present at the hearing but without the right to
examine or cross-examine. They may,
however, submit to the investigating officer
questions which may be asked to the party or
witness concerned.
The hearing shall be held within ten (10) days
from submission of the counter-affidavits and
other documents or from the expiration of the
period for their submission. It shall be
terminated within five (5) days.
(f) Within ten (10) days after the investigation,
the investigating officer shall determine
whether or not there is sufficient ground to hold
the respondent for trial. (3a)
Section 4. Resolution of investigating
prosecutor and its review. If the investigating
prosecutor finds cause to hold the respondent
for trial, he shall prepare the resolution and
information. He shall certify under oath in the
information that he, or as shown by the record,
an authorized officer, has personally examined
the complainant and his witnesses; that there is
reasonable ground to believe that a crime has
been committed and that the accused is
probably guilty thereof; that the accused was
informed of the complaint and of the evidence
submitted against him; and that he was given
an opportunity to submit controverting
evidence. Otherwise, he shall recommend the
dismissal of the complaint.

Within five (5) days from his resolution, he shall


forward the record of the case to the provincial
or city prosecutor or chief state prosecutor, or
to the Ombudsman or his deputy in cases of
offenses cognizable by the Sandiganbayan in
the exercise of its original jurisdiction. They
shall act on the resolution within ten (10) days
from their receipt thereof and shall immediately
inform the parties of such action.
No complaint or information may be filed or
dismissed by an investigating prosecutor
without the prior written authority or approval of
the provincial or city prosecutor or chief state
prosecutor or the Ombudsman or his deputy.
Where
the
investigating
prosecutor
recommends the dismissal of the complaint but
his recommendation is disapproved by the
provincial or city prosecutor or chief state
prosecutor or the Ombudsman or his deputy on
the ground that a probable cause exists, the
latter may, by himself, file the information
against the respondent, or direct any other
assistant prosecutor or state prosecutor to do
so without conducting another preliminary
investigation.
If upon petition by a proper party under such
rules as the Department of Justice may
prescribe or motu proprio, the Secretary of
Justice reverses or modifies the resolution of
the provincial or city prosecutor or chief state
prosecutor, he shall direct the prosecutor
concerned either to file the corresponding
information
without
conducting
another
preliminary
Section 6. When warrant of arrest may issue.
(a) By the Regional Trial Court. Within ten
(10) days from the filing of the complaint or
information, the judge shall personally evaluate
the resolution of the prosecutor and its
supporting evidence. He may immediately
dismiss the case if the evidence on record
clearly fails to establish probable cause. If he
finds probable cause, he shall issue a warrant
of arrest, or a commitment order if the accused
has already been arrested pursuant to a
warrant issued by the judge who conducted the
preliminary investigation or when the complaint
or information was filed pursuant to section 7 of
this Rule. In case of doubt on the existence of
probable cause, the judge may order the
prosecutor to present additional evidence
within five (5) days from notice and the issue

must be resolved by the court within thirty (30)


days from the filing of the complaint of
information.
(b) By the Municipal Trial Court. When
required pursuant to the second paragraph of
section 1 of this Rule, the preliminary
investigation of cases falling under the original
jurisdiction of the Metropolitan Trial Court,
Municipal Trial Court in Cities, Municipal Trial
Court, or Municipal Circuit Trial Court may be
conducted by either the judge or the
prosecutor. When conducted by the prosecutor,
the procedure for the issuance of a warrant or
arrest by the judge shall be governed by
paragraph (a) of this section. When the
investigation is conducted by the judge himself,
he shall follow the procedure provided in
section 3 of this Rule. If the findings and
recommendations are affirmed by the
provincial or city prosecutor, or by the
Ombudsman or his deputy, and the
corresponding information is filed, he shall
issue a warrant of arrest. However, without
waiting for the conclusion of the investigation,
the judge may issue a warrant of arrest if he
finds after an examination in writing and under
oath of the complainant and his witnesses in
the form of searching question and answers,
that a probable cause exists and that there is a
necessity of placing the respondent under
immediate custody in order not to frustrate the
ends of justice.
(c) When warrant of arrest not necessary. A
warrant of arrest shall not issue if the accused
is already under detention pursuant to a
warrant issued by the municipal trial court in
accordance with paragraph (b) of this section,
or if the complaint or information was filed
pursuant to section 7 of this Rule or is for an
offense penalized by fine only. The court shall
then proceed in the exercise of its original
jurisdiction
Section 8. Records. (a) Records supporting
the information or complaint. An information
or complaint filed in court shall be supported by
the affidavits and counter-affidavits of the
parties and their witnesses, together with the
other supporting evidence and the resolution
on the case.
(b) Record of preliminary investigation. The
record of the preliminary investigation, whether
conducted by a judge or a fiscal, shall not form

part of the record of the case. However, the


court, on its own initiative or on motion of any
party, may order the production of the record or
any its part when necessary in the resolution of
the case or any incident therein, or when it is to
be introduced as an evidence in the case by
the requesting party.
RJCL Section 13. Taking Custody of a

Child Without a Warrant. - The law


enforcement officer or a private person
taking into custody a child in conflict with
the law without a warrant shall observe
the provisions in Sections 5, 8 and 9 of
Rule 113 of the Revised Rules of
Criminal Procedure and shall forthwith
deliver the child to the nearest police
station. The child shall be proceeded
against in accordance with Section 7 of
Rule 112 of the Rules of Criminal
Procedure.
RODIL VS GARCIA
FACTS
-Counsel for Reynaldo Rodil who was charged with murder, asks to recall witnesses
for the prosecution to enable such counsel to cross-examine them, on, to quote his
words, "clarificatory and amplificatory matters" which was denied by Municipal Judge
Segundo M. Garcia of Sta. Cruz, Marinduque.
-What is prayed for is not only that such order denying counsel's request to recall
government witnesses be set aside and nullified, but also that bail be granted
petitioner, a petition to that effect having been denied with a subsequent motion for
reconsideration still undecided.
-Respondents were required to comment and the Court likewise issued a temporary
restraining order. Such a comment was submitted on behalf of respondents by the
Solicitor General seeking the dismissal of the petition on the ground that the right to
cross-examine in a preliminary investigation is not a right granted an accused
and that the exercise of discretion by respondent Judge considering the
evidence of record sufficed to justify denial of the application for bail.
-An examination of the record, as well as the pertinent doctrines, makes evident that
the jurisdictional issue posed arises from the failure to accord petitioner a
hearing on his application for bail.
-A resolution of that question in the sense of respondent Judge affording petitioner his
day in court is equally decisive of the other issue, whether or not counsel for petitioner
could recall witnesses for the prosecution for the purpose of asking clarificatory
questions. That he could very well do when they testify to prove evidence of guilt is
strong. Under the present state of the law, it cannot be said that the right to
cross-examine is guaranteed an accused at the stage of preliminary
investigation.
ISSUE
WON counsel for petitioner could recall witnesses for the prosecution for the purpose
of asking clarificatory questions (that he could very well do when they testify to prove
evidence of guilt is strong)
HELD
YES. Counsel could recall the witnesses.
- The Judge issued the denial for bail on the basis of the motion of petitioner that
he be granted such right and the opposition filed by the First Assistant
Provincial Fiscal without conducting any hearing on such motion. Clearly, he
acted on the mistaken belief that the presentation of evidence by the
prosecution for the purpose of the issuance of the warrant of arrest, the
preliminary examination proper, suffices for the denial of the plea for bail. In the
latest case on the subject, People v. Sola, decided on March 17, 1981, this Court

relying on People v. San Diego, nullified an order of a municipal judge named


respondent in that case as he granted bail to the accused without hearing the
prosecution. The present case is much stronger; it is the accused himself, the
explicit beneficiary of the constitutional right, who was not heard.
- There was misapprehension on the part of respondent Judge of the import of the
ruling in Ocampo v. Bernabe citing that The regular trial is, to a limited extent at
least, anticipated. While the guilt or innocence of the accused is not to be
determined, the quantity and character of the proofs on this point are, for the
special purpose in hand, necessarily considered. Occasionally much time is thus
consumed, and the court's attention is correspondingly diverted from other business.
But these objections cannot avail against a positive constitutional command; if the
Constitution requires the court to determine for itself whether or not the proof is evident
or presumption great in a given case, all considerations of expediency or convenience,
however potent they might be at the common law, must give way.'" This is so because
the procedure to be followed in the hearing on an application for bail, while
summary in character, is not to be a mere sham or pretense. It must not be an
exercise in futility. The accused is not to be denied his day in court.
- While it could be said that that the refusal of the Justice of the Peace to allow the
defense to cross-examine the prosecution's witnesses presented prior to
petitioner's arrest, cannot be utilized as argument for the contention that the
prosecution should not have been allowed to cross-examine the defense
witnesses and that an accused is not entitled to cross-examine the witnesses
presented against him in the preliminary investigation before his arrest, this
being a matter that depends on the sound discretion of the Judge or
investigating officer concerned (People v. Ramilo, \Dequito v. Arellano, Bustos v.
Lucero) it could still be argued that the judge is not a ministerial officer reduced
to recording what takes place and what witnesses say in the examination. Above
all, his is the great responsibility of safeguarding the accused from groundless
or vindictive prosecution. If the justice of the peace is to ascertain, as he must,
whether a crime has been committed and, if so, whether there is probable cause that
the accused committed it, his authority cannot be confined as in a straight jacket to the
stiffness of medieval and outmoded technicalities of practice. It thus appears clearly
that in the exercise of his discretion respondent Judge could have granted the request
and thus avoided the necessity of a petition of this character having to be filed. The
interest of a more speedy and a more efficient administration of justice would be
best served if there is a greater awareness on the part of judges that in addition
to safeguarding the express rights of an accused person, a matter mandated by
the Constitution or the Rules of Court, they should likewise exercise their
discretion in such a way that the purpose of a preliminary investigation, the
avoidance of groundless or vindictive prosecutions, could be attained in as fair
and objective manner as possible.
Dispositive WHEREFORE, the writ of certiorari is granted. The order of respondent
Judge denying bail is set aside. He, or whoever is now the Municipal Judge of Santa
Cruz, Marinduque, must set forthwith the hearing on the application for bail of
petitioner, to be conducted in accordance with the requirements of the Constitution, the
Rules of Court, and this opinion. No costs.

ALLADO VS DIOKNO
FACTS
- Petitioners Diosdado Jose Allado and Roberto L. Mendoza, alumni of the College of
Law, University of the Philippines, are partners of the Law Firm of Salonga, Hernandez
and Allado. In the practice of their profession, and on the basis of an alleged
extrajudicial confession of a security guard (Umbal), they have been accused of the
heinous crime of kidnapping with murder of a German national named Van Twest by
the Presidential Anti-Crime Commission (PACC) and ordered arrested without bail by
respondent judge.
- Petitioners filed this petition and principally contended that respondent judge acted
with grave abuse of discretion and in excess of jurisdiction in "whimsically holding that
there is probable cause against petitioners without determining the admissibility of the
evidence against petitioners and without even stating the basis of his findings," and in
"relying on the Resolution of the Panel and their certification that probable cause exists
when the certification is flawed." Petitioners maintain that the records of the
preliminary investigation which respondent judge solely relied upon failed to establish
probable cause against them to justify the issuance of the warrant of arrest. Petitioners
likewise assail the prosecutors' "clear sign of bias and partiality."
- On the other hand, the Office of the Solicitor General argues that the determination of
probable cause is a function of the judge who is merely required to personally
appreciate certain facts to convince him that the accused probably committed the
crime charged.
ISSUE
WON the respondent judge committed grave abuse of discretion in the preliminary
inquiry which determines probable cause for the issuance of a warrant of arrest
HELD
- In the Order of respondent judge, it is expressly stated that "[t]his court after careful
evaluation of the evidence on record, believes and rules that probable cause exists;

and therefore, a warrant of arrest should be issued." However, we are unable to see
how respondent judge arrived at such ruling. We have painstakingly examined the
records and we cannot find any support for his conclusion. On the contrary, we discern
a number of reasons why we consider the evidence submitted to be insufficient for a
finding of probable cause against petitioners.
- The PACC relies heavily on the sworn statement of Security Guard Umbal who
supposedly confessed his participation in the alleged kidnapping and murder of Van
Twest. For one, there is serious doubt on Van Twest's reported death since the corpus
delicti has not been established, nor have his remains been recovered. Umbal claims
that Van Twest was completely burned into ashes with the use of gasoline and rubber
tires from around ten o'clock in the evening to six o'clock the next morning. This is
highly improbable, if not ridiculous. A human body cannot be pulverized into ashes by
simply burning it with the use of gasoline and rubber tires in an open field. Even
crematoria use entirely closed incinerators where the corpse is subjected to intense
heat. Thereafter, the remains undergo a process where the bones are completely
ground to dust.
- Strangely, if not awkwardly, after Van Twest's reported abduction which culminated in
his decimation by cremation, his counsel continued to represent him before judicial
and quasi-judicial proceedings. Hence, even Asst. Solicitor General Estoesta believes
that counsel of Van Twest doubted the latter's death.
- Verily, respondent judge committed grave abuse of discretion in issuing the warrant
for the arrest of petitioners it appearing that he did not personally examine the
evidence nor did he call for the complainant and his witnesses in the face of their
incredible accounts. Instead, he merely relied on the certification of the prosecutors
that probable cause existed. For, otherwise, he would have found out that the
evidence thus far presented was utterly insufficient to warrant the arrest of petitioners.
- In Soliven v. Makasiar, we said that the judge (a) shall personally evaluate the report
and the supporting documents submitted by the fiscal regarding the existence of
probable cause and, on the basis thereof, issue a warrant of arrest; or, (b) if on the
basis thereof he finds no probable cause, may disregard the fiscal's report and require
the submission of supporting affidavits of witnesses to aid him in arriving at a
conclusion on the existence of probable cause.
- In People v. Inting, we emphasized the important features of the constitutional
mandate: (a) The determination of probable cause is a function of the judge; it is not
for the provincial fiscal or prosecutor to ascertain. Only the judge and the judge alone
makes this determination; (b) The preliminary inquiry made by a prosecutor does not
bind the judge. It merely assists him in making the determination of probable cause.
The judge does not have to follow what the prosecutor presents to him. By itself, the
prosecutor's certification of probable cause is ineffectual. It is the report, the affidavits,
the transcript of stenographic notes (if any), and all other supporting documents
behind the prosecutor's certification which are material in assisting the judge in his
determination of probable cause; and, (c) Judges and prosecutors alike should
distinguish the preliminary inquiry which determines probable cause for the issuance
of a warrant of arrest from the preliminary investigation proper which ascertains
whether the offender should be held for trial or released. Even if the two inquiries be
conducted in the course of one and the same proceeding, there should be no
confusion about their objectives. The determination of probable cause for the warrant
is made by the judge. The preliminary investigation proper whether or not there is
reasonable ground to believe that the accused is guilty of the offense charged and
therefore, whether or not he should be subjected to the expense, rigors and
embarrassment of trial is a function of the prosecutor.
- ALLADO DOCTRINE: If upon the filing of the information in court, the trial judge, after
reviewing the information and the document attached thereto, finds that no probable
cause exists, he must either call for the complainant and the witnesses themselves or
simply dismiss the case. There is no reason to hold the accused for trial and further
expose him to an open and public accusation of the crime when no probable cause
exists.
Dispositive Petition granted

2. CONDUCTED BY MTC JUDGE


RULE 112 Section 3. Procedure.

The
preliminary investigation shall be conducted in
the following manner:
(a) The complaint shall state the address of the
respondent and shall be accompanied by the
affidavits of the complainant and his witnesses,
as well as other supporting documents to
establish probable cause. They shall be in such
number of copies as there are respondents,
plus two (2) copies for the official file. The
affidavits shall be subscribed and sworn to

before any prosecutor or government official


authorized to administer oath, or, in their
absence or unavailability, before a notary
public, each of who must certify that he
personally examined the affiants and that he is
satisfied that they voluntarily executed and
understood their affidavits.
(b) Within ten (10) days after the filing of the
complaint, the investigating officer shall either
dismiss it if he finds no ground to continue with
the investigation, or issue a subpoena to the
respondent attaching to it a copy of the
complaint and its supporting affidavits and
documents.
The respondent shall have the right to examine
the evidence submitted by the complainant
which he may not have been furnished and to
copy them at his expense. If the evidence is
voluminous, the complainant may be required
to specify those which he intends to present
against the respondent, and these shall be
made available for examination or copying by
the respondent at his expense.
Objects as evidence need not be furnished a
party but shall be made available for
examination, copying, or photographing at the
expense of the requesting party.
(c) Within ten (10) days from receipt of the
subpoena with the complaint and supporting
affidavits and documents, the respondent shall
submit his counter-affidavit and that of his
witnesses and other supporting documents
relied upon for his defense. The counteraffidavits shall be subscribed and sworn to and
certified as provided in paragraph (a) of this
section, with copies thereof furnished by him to
the complainant. The respondent shall not be
allowed to file a motion to dismiss in lieu of a
counter-affidavit.
(d) If the respondent cannot be subpoenaed, or
if subpoenaed, does not submit counteraffidavits within the ten (10) day period, the
investigating officer shall resolve the complaint
based on the evidence presented by the
complainant.
(e) The investigating officer may set a hearing
if there are facts and issues to be clarified from
a party or a witness. The parties can be
present at the hearing but without the right to
examine or cross-examine. They may,
however, submit to the investigating officer
questions which may be asked to the party or

witness concerned.
The hearing shall be held within ten (10) days
from submission of the counter-affidavits and
other documents or from the expiration of the
period for their submission. It shall be
terminated within five (5) days.
(f) Within ten (10) days after the investigation,
the investigating officer shall determine
whether or not there is sufficient ground to hold
the respondent for trial.
Section 5. Resolution of investigating judge
and its review. Within ten (10) days after the
preliminary investigation, the investigating
judge shall transmit the resolution of the case
to the provincial or city prosecutor, or to the
Ombudsman or his deputy in cases of offenses
cognizable by the Sandiganbayan in the
exercise of its original jurisdiction, for
appropriate action. The resolution shall state
the findings of facts and the law supporting his
action, together with the record of the case
which shall include: (a) the warrant, if the arrest
is by virtue of a warrant; (b) the affidavits,
counter-affidavits
and
other
supporting
evidence of the parties; (c) the undertaking or
bail of the accused and the order for his
release; (d) the transcripts of the proceedings
during the preliminary investigation; and (e) the
order of cancellation of his bail bond, if the
resolution is for the dismissal of the complaint.
Within thirty (30) days from receipt of the
records, the provincial or city prosecutor, or the
Ombudsman or his deputy, as the case may
be, shall review the resolution of the
investigating judge on the existence of
probable cause. Their ruling shall expressly
and clearly state the facts and the law on which
it is based and the parties shall be furnished
with copies thereof. They shall order the
release of an accused who is detained if no
probable cause is found against him. (5a)
Section 6. When warrant of arrest may issue.
(a) By the Regional Trial Court. Within ten
(10) days from the filing of the complaint or
information, the judge shall personally evaluate
the resolution of the prosecutor and its
supporting evidence. He may immediately
dismiss the case if the evidence on record
clearly fails to establish probable cause. If he
finds probable cause, he shall issue a warrant
of arrest, or a commitment order if the accused
has already been arrested pursuant to a

warrant issued by the judge who conducted the


preliminary investigation or when the complaint
or information was filed pursuant to section 7 of
this Rule. In case of doubt on the existence of
probable cause, the judge may order the
prosecutor to present additional evidence
within five (5) days from notice and the issue
must be resolved by the court within thirty (30)
days from the filing of the complaint of
information.
(b) By the Municipal Trial Court. When
required pursuant to the second paragraph of
section 1 of this Rule, the preliminary
investigation of cases falling under the original
jurisdiction of the Metropolitan Trial Court,
Municipal Trial Court in Cities, Municipal Trial
Court, or Municipal Circuit Trial Court may be
conducted by either the judge or the
prosecutor. When conducted by the prosecutor,
the procedure for the issuance of a warrant or
arrest by the judge shall be governed by
paragraph (a) of this section. When the
investigation is conducted by the judge himself,
he shall follow the procedure provided in
section 3 of this Rule. If the findings and
recommendations are affirmed by the
provincial or city prosecutor, or by the
Ombudsman or his deputy, and the
corresponding information is filed, he shall
issue a warrant of arrest. However, without
waiting for the conclusion of the investigation,
the judge may issue a warrant of arrest if he
finds after an examination in writing and under
oath of the complainant and his witnesses in
the form of searching question and answers,
that a probable cause exists and that there is a
necessity of placing the respondent under
immediate custody in order not to frustrate the
ends of justice.
(c) When warrant of arrest not necessary. A
warrant of arrest shall not issue if the accused
is already under detention pursuant to a
warrant issued by the municipal trial court in
accordance with paragraph (b) of this section,
or if the complaint or information was filed
pursuant to section 7 of this Rule or is for an
offense penalized by fine only. The court shall
then proceed in the exercise of its original
jurisdiction
Section 8. Records. (a) Records supporting
the information or complaint. An information
or complaint filed in court shall be supported by

the affidavits and counter-affidavits of the


parties and their witnesses, together with the
other supporting evidence and the resolution
on the case.
(b) Record of preliminary investigation. The
record of the preliminary investigation, whether
conducted by a judge or a fiscal, shall not form
part of the record of the case. However, the
court, on its own initiative or on motion of any
party, may order the production of the record or
any its part when necessary in the resolution of
the case or any incident therein, or when it is to
be introduced as an evidence in the case by
the requesting party. (8a)

Section 13. Taking Custody of a Child


Without a Warrant. - The law
enforcement officer or a private person
taking into custody a child in conflict with
the law without a warrant shall observe
the provisions in Sections 5, 8 and 9 of
Rule 113 of the Revised Rules of
Criminal Procedure and shall forthwith
deliver the child to the nearest police
station. The child shall be proceeded
against in accordance with Section 7 of
Rule 112 of the Rules of Criminal
Procedure.
B. IN CASES COGNIZIBLE BY MTC
1.
CONDUCTED
BY
PROSECUTOR
Section 1 (PAR 2)
Section 1. Preliminary investigation defined;
when required. Preliminary investigation is
an inquiry or proceeding to determine whether
there is sufficient ground to engender a wellfounded belief that a crime has been
committed and the respondent is probably
guilty thereof, and should be held for trial.
Except as provided in section 7 of this
Rule, a preliminary investigation is
required to be conducted before the
filing of a complaint or information for an
offense where the penalty prescribed by
law is at least four (4) years, two (2)
months and one (1) day without regard
to the fine.
Section 3. Procedure. The preliminary
investigation shall be conducted in the

following manner:
(a) The complaint shall state the address of the
respondent and shall be accompanied by the
affidavits of the complainant and his witnesses,
as well as other supporting documents to
establish probable cause. They shall be in such
number of copies as there are respondents,
plus two (2) copies for the official file. The
affidavits shall be subscribed and sworn to
before any prosecutor or government official
authorized to administer oath, or, in their
absence or unavailability, before a notary
public, each of who must certify that he
personally examined the affiants and that he is
satisfied that they voluntarily executed and
understood their affidavits.
(b) Within ten (10) days after the filing of the
complaint, the investigating officer shall either
dismiss it if he finds no ground to continue with
the investigation, or issue a subpoena to the
respondent attaching to it a copy of the
complaint and its supporting affidavits and
documents.
The respondent shall have the right to examine
the evidence submitted by the complainant
which he may not have been furnished and to
copy them at his expense. If the evidence is
voluminous, the complainant may be required
to specify those which he intends to present
against the respondent, and these shall be
made available for examination or copying by
the respondent at his expense.
Objects as evidence need not be furnished a
party but shall be made available for
examination, copying, or photographing at the
expense of the requesting party.
(c) Within ten (10) days from receipt of the
subpoena with the complaint and supporting
affidavits and documents, the respondent shall
submit his counter-affidavit and that of his
witnesses and other supporting documents
relied upon for his defense. The counteraffidavits shall be subscribed and sworn to and
certified as provided in paragraph (a) of this
section, with copies thereof furnished by him to
the complainant. The respondent shall not be
allowed to file a motion to dismiss in lieu of a
counter-affidavit.
(d) If the respondent cannot be subpoenaed, or
if subpoenaed, does not submit counteraffidavits within the ten (10) day period, the
investigating officer shall resolve the complaint

based on the evidence presented by the


complainant.
(e) The investigating officer may set a hearing
if there are facts and issues to be clarified from
a party or a witness. The parties can be
present at the hearing but without the right to
examine or cross-examine. They may,
however, submit to the investigating officer
questions which may be asked to the party or
witness concerned.
The hearing shall be held within ten (10) days
from submission of the counter-affidavits and
other documents or from the expiration of the
period for their submission. It shall be
terminated within five (5) days.
(f) Within ten (10) days after the investigation,
the investigating officer shall determine
whether or not there is sufficient ground to hold
the respondent for trial. (3a)
Section 4. Resolution of investigating
prosecutor and its review. If the investigating
prosecutor finds cause to hold the respondent
for trial, he shall prepare the resolution and
information. He shall certify under oath in the
information that he, or as shown by the record,
an authorized officer, has personally examined
the complainant and his witnesses; that there is
reasonable ground to believe that a crime has
been committed and that the accused is
probably guilty thereof; that the accused was
informed of the complaint and of the evidence
submitted against him; and that he was given
an opportunity to submit controverting
evidence. Otherwise, he shall recommend the
dismissal of the complaint.
Within five (5) days from his resolution, he shall
forward the record of the case to the provincial
or city prosecutor or chief state prosecutor, or
to the Ombudsman or his deputy in cases of
offenses cognizable by the Sandiganbayan in
the exercise of its original jurisdiction. They
shall act on the resolution within ten (10) days
from their receipt thereof and shall immediately
inform the parties of such action.
No complaint or information may be filed or
dismissed by an investigating prosecutor
without the prior written authority or approval of
the provincial or city prosecutor or chief state
prosecutor or the Ombudsman or his deputy.
Where
the
investigating
prosecutor
recommends the dismissal of the complaint but
his recommendation is disapproved by the

provincial or city prosecutor or chief state


prosecutor or the Ombudsman or his deputy on
the ground that a probable cause exists, the
latter may, by himself, file the information
against the respondent, or direct any other
assistant prosecutor or state prosecutor to do
so without conducting another preliminary
investigation.
If upon petition by a proper party under such
rules as the Department of Justice may
prescribe or motu proprio, the Secretary of
Justice reverses or modifies the resolution of
the provincial or city prosecutor or chief state
prosecutor, he shall direct the prosecutor
concerned either to file the corresponding
information
without
conducting
another
preliminary investigation, or to dismiss or move
for dismissal of the complaint or information
with notice to the parties.
Section 6. When warrant of arrest may issue.
(a) By the Regional Trial Court. Within ten
(10) days from the filing of the complaint or
information, the judge shall personally evaluate
the resolution of the prosecutor and its
supporting evidence. He may immediately
dismiss the case if the evidence on record
clearly fails to establish probable cause. If he
finds probable cause, he shall issue a warrant
of arrest, or a commitment order if the accused
has already been arrested pursuant to a
warrant issued by the judge who conducted the
preliminary investigation or when the complaint
or information was filed pursuant to section 7 of
this Rule. In case of doubt on the existence of
probable cause, the judge may order the
prosecutor to present additional evidence
within five (5) days from notice and the issue
must be resolved by the court within thirty (30)
days from the filing of the complaint of
information.
(b) By the Municipal Trial Court. When
required pursuant to the second paragraph of
section 1 of this Rule, the preliminary
investigation of cases falling under the original
jurisdiction of the Metropolitan Trial Court,
Municipal Trial Court in Cities, Municipal Trial
Court, or Municipal Circuit Trial Court may be
conducted by either the judge or the
prosecutor. When conducted by the prosecutor,
the procedure for the issuance of a warrant or
arrest by the judge shall be governed by
paragraph (a) of this section. When the

investigation is conducted by the judge himself,


he shall follow the procedure provided in
section 3 of this Rule. If the findings and
recommendations are affirmed by the
provincial or city prosecutor, or by the
Ombudsman or his deputy, and the
corresponding information is filed, he shall
issue a warrant of arrest. However, without
waiting for the conclusion of the investigation,
the judge may issue a warrant of arrest if he
finds after an examination in writing and under
oath of the complainant and his witnesses in
the form of searching question and answers,
that a probable cause exists and that there is a
necessity of placing the respondent under
immediate custody in order not to frustrate the
ends of justice.
(c) When warrant of arrest not necessary. A
warrant of arrest shall not issue if the accused
is already under detention pursuant to a
warrant issued by the municipal trial court in
accordance with paragraph (b) of this section,
or if the complaint or information was filed
pursuant to section 7 of this Rule or is for an
offense penalized by fine only. The court shall
then proceed in the exercise of its original
jurisdiction.
Section 8. Records. (a) Records supporting
the information or complaint. An information
or complaint filed in court shall be supported by
the affidavits and counter-affidavits of the
parties and their witnesses, together with the
other supporting evidence and the resolution
on the case.
(b) Record of preliminary investigation. The
record of the preliminary investigation, whether
conducted by a judge or a fiscal, shall not form
part of the record of the case. However, the
court, on its own initiative or on motion of any
party, may order the production of the record or
any its part when necessary in the resolution of
the case or any incident therein, or when it is to
be introduced as an evidence in the case by
the requesting party.

Section 13. Taking Custody of a Child


Without a Warrant. - The law
enforcement officer or a private person
taking into custody a child in conflict with
the law without a warrant shall observe
the provisions in Sections 5, 8 and 9 of
Rule 113 of the Revised Rules of

Criminal Procedure and shall forthwith


deliver the child to the nearest police
station. The child shall be proceeded
against in accordance with Section 7 of
Rule 112 of the Rules of Criminal
Procedure.
C. IN CASE COGNIZIBLE BY
SANDIGANBAYAN
Section 3. Procedure. The preliminary
investigation shall be conducted in the
following manner:
(a) The complaint shall state the address of the
respondent and shall be accompanied by the
affidavits of the complainant and his witnesses,
as well as other supporting documents to
establish probable cause. They shall be in such
number of copies as there are respondents,
plus two (2) copies for the official file. The
affidavits shall be subscribed and sworn to
before any prosecutor or government official
authorized to administer oath, or, in their
absence or unavailability, before a notary
public, each of who must certify that he
personally examined the affiants and that he is
satisfied that they voluntarily executed and
understood their affidavits.
(b) Within ten (10) days after the filing of the
complaint, the investigating officer shall either
dismiss it if he finds no ground to continue with
the investigation, or issue a subpoena to the
respondent attaching to it a copy of the
complaint and its supporting affidavits and
documents.
The respondent shall have the right to examine
the evidence submitted by the complainant
which he may not have been furnished and to
copy them at his expense. If the evidence is
voluminous, the complainant may be required
to specify those which he intends to present
against the respondent, and these shall be
made available for examination or copying by
the respondent at his expense.
Objects as evidence need not be furnished a
party but shall be made available for
examination, copying, or photographing at the
expense of the requesting party.
(c) Within ten (10) days from receipt of the
subpoena with the complaint and supporting
affidavits and documents, the respondent shall
submit his counter-affidavit and that of his

witnesses and other supporting documents


relied upon for his defense. The counteraffidavits shall be subscribed and sworn to and
certified as provided in paragraph (a) of this
section, with copies thereof furnished by him to
the complainant. The respondent shall not be
allowed to file a motion to dismiss in lieu of a
counter-affidavit.
(d) If the respondent cannot be subpoenaed, or
if subpoenaed, does not submit counteraffidavits within the ten (10) day period, the
investigating officer shall resolve the complaint
based on the evidence presented by the
complainant.
(e) The investigating officer may set a hearing
if there are facts and issues to be clarified from
a party or a witness. The parties can be
present at the hearing but without the right to
examine or cross-examine. They may,
however, submit to the investigating officer
questions which may be asked to the party or
witness concerned.
The hearing shall be held within ten (10)
days from submission of the counteraffidavits and other documents or from
the expiration of the period for their
submission. It shall be terminated within
five (5) days.
(f) Within ten (10) days after the
investigation, the investigating officer
shall determine whether or not there is
sufficient ground to hold the respondent
for trial. (3a)
Section 4. Resolution of investigating
prosecutor and its review. If the
investigating prosecutor finds cause to
hold the respondent for trial, he shall
prepare the resolution and information.
He shall certify under oath in the
information that he, or as shown by the
record, an authorized officer, has
personally examined the complainant
and his witnesses; that there is
reasonable ground to believe that a
crime has been committed and that the
accused is probably guilty thereof; that
the accused was informed of the
complaint and of the evidence submitted
against him; and that he was given an
opportunity to submit controverting
evidence.
Otherwise,
he
shall
recommend the dismissal of the

complaint.
Within five (5) days from his resolution,
he shall forward the record of the case
to the provincial or city prosecutor or
chief state prosecutor, or to the
Ombudsman or his deputy in cases of
offenses
cognizable
by
the
Sandiganbayan in the exercise of its
original jurisdiction. They shall act on the
resolution within ten (10) days from their
receipt thereof and shall immediately
inform the parties of such action.
No complaint or information may be filed
or dismissed by an investigating
prosecutor without the prior written
authority or approval of the provincial or
city prosecutor or chief state prosecutor
or the Ombudsman or his deputy.
Where the investigating prosecutor
recommends the dismissal of the
complaint but his recommendation is
disapproved by the provincial or city
prosecutor or chief state prosecutor or
the Ombudsman or his deputy on the
ground that a probable cause exists, the
latter may, by himself, file the
information against the respondent, or
direct any other assistant prosecutor or
state prosecutor to do so without
conducting
another
preliminary
investigation.
If upon petition by a proper party under
such rules as the Department of Justice
may prescribe or motu proprio, the
Secretary of Justice reverses or
modifies the resolution of the provincial
or city prosecutor or chief state
prosecutor, he
shall
direct
the
prosecutor concerned either to file the
corresponding
information
without
conducting
another
preliminary
investigation, or to dismiss or move for
dismissal of the complaint or information
with notice to the parties. The same rule
shall apply in preliminary investigations
conducted by the officers of the Office of
the Ombudsman. (4a)
Section 5. Resolution of investigating
judge and its review. Within ten (10)
days after the preliminary investigation,
the investigating judge shall transmit the
resolution of the case to the provincial or

city prosecutor, or to the Ombudsman or


his deputy in cases of offenses
cognizable by the Sandiganbayan in the
exercise of its original jurisdiction, for
appropriate action. The resolution shall
state the findings of facts and the law
supporting his action, together with the
record of the case which shall include:
(a) the warrant, if the arrest is by virtue
of a warrant; (b) the affidavits, counteraffidavits and other supporting evidence
of the parties; (c) the undertaking or bail
of the accused and the order for his
release; (d) the transcripts of the
proceedings during the preliminary
investigation; and (e) the order of
cancellation of his bail bond, if the
resolution is for the dismissal of the
complaint.
Within thirty (30) days from receipt of the
records, the provincial or city prosecutor,
or the Ombudsman or his deputy, as the
case may be, shall review the resolution
of the investigating judge on the
existence of probable cause. Their ruling
shall expressly and clearly state the
facts and the law on which it is based
and the parties shall be furnished with
copies thereof. They shall order the
release of an accused who is detained if
no probable cause is found against him.
Section 8. Records. (a) Records supporting
the information or complaint. An information
or complaint filed in court shall be supported by
the affidavits and counter-affidavits of the
parties and their witnesses, together with the
other supporting evidence and the resolution
on the case.
(b) Record of preliminary investigation. The
record of the preliminary investigation, whether
conducted by a judge or a fiscal, shall not form
part of the record of the case. However, the
court, on its own initiative or on motion of any
party, may order the production of the record or
any its part when necessary in the resolution of
the case or any incident therein, or when it is to
be introduced as an evidence in the case by
the requesting party.

2.

CONDUCTED
OMBUDSMAN/

BY

RA 6770

SPECIAL
PROSECUTOR

AN ACT PROVIDING FOR THE


FUNCTIONAL AND STRUCTURAL
ORGANIZATION OF THE OFFICE OF
THE OMBUDSMAN, AND FOR OTHER
PURPOSES
Be it enacted by the Senate and House
of Representatives of the Philippines in
Congress assembled::
Section 1. Title. This Act shall be
known as "The Ombudsman Act of
1989".
Section 2. Declaration of Policy.
The State shall maintain honesty and
integrity in the public service and take
positive and effective measures against
graft and corruption.
Public office is a public trust. Public
officers and employees must at all times
be accountable to the people, serve
them with utmost responsibility, integrity,
loyalty, efficiency, act with patriotism and
justice and lead modest lives.
Section 3. Office of the Ombudsman.
The Office of the Ombudsman shall
include the Office of the Overall Deputy,
the Office of the Deputy for Luzon, the
Office of the Deputy for the Visayas, the
Office of the Deputy for Mindanao, the
Office of the Deputy for the Armed
Forces, and the Office of the Special
Prosecutor. The President may appoint
other Deputies as the necessity for it
may arise, as recommended by the
Ombudsman.
Section 4. Appointment. The
Ombudsman and his Deputies, including
the Special Prosecutor, shall be
appointed by the President from a list of
at least twenty-one (21) nominees
prepared by the Judicial and Bar
Council, and from a list of three (3)

nominees for each vacancy thereafter,


which shall be filled within three (3)
months after it occurs, each of which list
shall be published in a newspaper of
general circulation.
In the organization of the Office of the
Ombudsman for filling up of positions
therein, regional, cultural or ethnic
considerations shall be taken into
account to the end that the Office shall
be as much as possible representative
of the regional, ethnic and cultural
make-up of the Filipino nation.
Section 5. Qualifications. The
Ombudsman and his Deputies, including
the Special Prosecutor, shall be naturalborn citizens of the Philippines, at least
forty (40) years old, of recognized
probity and independence, members of
the Philippine Bar, and must not have
been candidates for any elective
national or local office in the immediately
preceding election whether regular or
special. The Ombudsman must have, for
ten (10) years or more, been a judge or
engaged in the practice of law in the
Philippines.
Section 6. Rank and Salary. The
Ombudsman and his Deputies shall
have the same ranks, salaries and
privileges as the Chairman and
members,
respectively,
of
a
Constitutional
Commission.
Their
salaries shall not be decreased during
their term of office.
The members of the prosecution,
investigation and legal staff of the Office
of the Ombudsman shall receive
salaries which shall not be less than
those given to comparable positions in
any office in the Government.
Section 7. Term of Office. The
Ombudsman and his Deputies, including
the Special Prosecutor, shall serve for a

term of seven (7) years without


reappointment.
Section 8. Removal; Filling of
Vacancy.
(1) In accordance with the provisions of
Article XI of the Constitution, the
Ombudsman may be removed from
office on impeachment for, and
conviction of, culpable violation of the
Constitution, treason, bribery, graft and
corruption, other high crimes, or betrayal
of public trust.
(2) A Deputy or the Special Prosecutor,
may be removed from office by the
President for any of the grounds
provided for the removal of the
Ombudsman, and after due process.
(3) In case of vacancy in the Office of
the Ombudsman due to death,
resignation, removal or permanent
disability of the incumbent Ombudsman,
the Overall Deputy shall serve as Acting
Ombudsman in a concurrent capacity
until a new Ombudsman shall have
been appointed for a full term.n case the
Overall Deputy cannot assume the role
of Acting Ombudsman, the President
may designate any of the Deputies, or
the Special Prosecutor, as Acting
Ombudsman.
(4) In case of temporary absence or
disability of the Ombudsman, the Overall
Deputy shall perform the duties of the
Ombudsman until the Ombudsman
returns or is able to perform his duties.
Section
9.
Prohibitions
and
Disqualifications. The Ombudsman,
his Deputies and the Special Prosecutor
shall not, during their tenure, hold any
other office or employment. They shall
not, during said tenure, directly or
indirectly practice any other profession,
participate in any business, or be
financially interested in any contract

with, or in any franchise, or special


privilege granted by the Government or
any
subdivision,
agency
or
instrumentality
thereof,
including
government-owned
or
controlled
corporations or their subsidiaries. They
shall strictly avoid conflict of interest in
the conduct of their office. They shall not
be qualified to run for any office in the
election immediately following their
cessation from office. They shall not be
allowed to appear or practice before the
Ombudsman for two (2) years following
their cessation from office.
No spouse or relative by consanguinity
or affinity within the fourth civil degree
and no law, business or professional
partner or associate of the Ombudsman,
his Deputies or Special Prosecutor
within one (1) year preceding the
appointment may appear as counsel or
agent on any matter pending before the
Office of the Ombudsman or transact
business directly or indirectly therewith.
This disqualification shall apply during
the tenure of the official concerned. This
disqualification likewise extends to the
law, business or professional firm for the
same period.
Section
10.
Disclosure
of
Relationship. It shall be the duty of
the Ombudsman, his Deputies, including
the Special Prosecutor to make under
oath, to the best of their knowledge
and/or information, a public disclosure of
the identities of, and their relationship
with the persons referred to in the
preceding section.
The disclosure shall be filed with the
Office of the President and the Office of
the Ombudsman before the appointee
assumes office and every year
thereafter. The disclosures made
pursuant to this section shall form part of

the public records and shall be available


to any person or entity upon request.
Section 11. Structural Organization.
The authority and responsibility for
the exercise of the mandate of the Office
of the Ombudsman and for the
discharge of its powers and functions
shall be vested in the Ombudsman, who
shall have supervision and control of the
said office.
(1) The Office of the Ombudsman may
organize
such
directorates
for
administration and allied services as
may be necessary for the effective
discharge of its functions. Those
appointed as directors or heads shall
have the rank and salary of line bureau
directors.
(2) The Office of the Overall Deputy
shall oversee and administer the
operations of the different offices under
the Office of Ombudsman.t shall
likewise perform such other functions
and duties assigned to it by the
Ombudsman.
(3) The Office of the Special Prosecutor
shall be composed of the Special
Prosecutor and his prosecution staff.
The Office of the Special Prosecutor
shall be an organic component of the
Office of the Ombudsman and shall be
under the supervision and control of the
Ombudsman.
(4) The Office of the Special Prosecutor
shall, under the supervision and control
and upon the authority of the
Ombudsman,
have
the
following
powers:
(a) To conduct preliminary investigation
and prosecute criminal cases within the
jurisdiction of the Sandiganbayan;
(b) To enter into plea bargaining
agreements; and
(c) To perform such other duties

assigned to it by the Ombudsman.


The Special Prosecutor shall have the
rank and salary of a Deputy
Ombudsman.
(5) The position structure and staffing
pattern of the Office of the Ombudsman,
including the Office of the Special
Prosecutor, shall be approved and
prescribed by the Ombudsman. The
Ombudsman shall appoint all officers
and employees of the Office of the
Ombudsman, including those of the
Office of the Special Prosecutor, in
accordance with the Civil Service Law,
rules and regulations.
Section 12. Official Stations. The
Ombudsman, the Overall Deputy, the
Deputy for Luzon, and the Deputy for
the Armed Forces shall hold office in
Metropolitan Manila; the Deputy for the
Visayas, in Cebu City; and the Deputy
for Mindanao, in Davao City. The
Ombudsman may transfer their stations
within their respective geographical
regions, as public interest may require.
Section 13.
Mandate.
The
Ombudsman and his Deputies, as
protectors of the people, shall act
promptly on complaints filed in any form
or manner against officers or employees
of the Government, or of any
subdivision, agency or instrumentality
thereof, including government-owned or
controlled corporations, and enforce
their administrative, civil and criminal
liability in every case where the
evidence warrants in order to promote
efficient service by the Government to
the people.
Section 14. Restrictions. No writ of
injunction shall be issued by any court to
delay an investigation being conducted
by the Ombudsman under this Act,
unless there is a prima facie evidence

that the subject matter of the


investigation is outside the jurisdiction of
the Office of the Ombudsman.
No court shall hear any appeal or
application for remedy against the
decision or findings of the Ombudsman,
except the Supreme Court, on pure
question of law.
Section 15. Powers, Functions and
Duties. The Office of the
Ombudsman shall have the following
powers, functions and duties:
(1) Investigate and prosecute on its own
or on complaint by any person, any act
or omission of any public officer or
employee, office or agency, when such
act or omission appears to be illegal,
unjust, improper or inefficient.t has
primary
jurisdiction
over
cases
cognizable by the Sandiganbayan and,
in the exercise of this primary
jurisdiction, it may take over, at any
stage, from any investigatory agency of
Government, the investigation of such
cases;
(2) Direct, upon complaint or at its own
instance, any officer or employee of the
Government, or of any subdivision,
agency or instrumentality thereof, as
well as any government-owned or
controlled corporations with original
charter, to perform and expedite any act
or duty required by law, or to stop,
prevent, and correct any abuse or
impropriety in the performance of duties;
(3) Direct the officer concerned to take
appropriate action against a public
officer or employee at fault or who
neglect to perform an act or discharge a
duty required by law, and recommend
his removal, suspension, demotion, fine,
censure, or prosecution, and ensure
compliance therewith; or enforce its
disciplinary authority as provided in

Section 21 of this Act: provided, that the


refusal by any officer without just cause
to comply with an order of the
Ombudsman to remove, suspend,
demote, fine, censure, or prosecute an
officer or employee who is at fault or
who neglects to perform an act or
discharge a duty required by law shall
be a ground for disciplinary action
against said officer;
(4) Direct the officer concerned, in any
appropriate case, and subject to such
limitations as it may provide in its rules
of procedure, to furnish it with copies of
documents relating to contracts or
transactions entered into by his office
involving the disbursement or use of
public funds or properties, and report
any irregularity to the Commission on
Audit for appropriate action;
(5) Request any government agency for
assistance and information necessary in
the discharge of its responsibilities, and
to examine, if necessary, pertinent
records and documents;
(6) Publicize matters covered by its
investigation of the matters mentioned in
paragraphs (1), (2), (3) and (4) hereof,
when circumstances so warrant and with
due prudence: provided, that the
Ombudsman under its rules and
regulations may determine what cases
may not be made public: provided,
further, that any publicity issued by the
Ombudsman shall be balanced, fair and
true;
(7) Determine the causes of inefficiency,
red tape, mismanagement, fraud, and
corruption in the Government, and make
recommendations for their elimination
and the observance of high standards of
ethics and efficiency;
(8) Administer oaths, issue subpoena
and subpoena duces tecum, and take

testimony in any investigation or inquiry,


including the power to examine and
have access to bank accounts and
records;
(9) Punish for contempt in accordance
with the Rules of Court and under the
same procedure and with the same
penalties provided therein;
(10) Delegate to the Deputies, or its
investigators or representatives such
authority or duty as shall ensure the
effective exercise or performance of the
powers, functions, and duties herein or
hereinafter provided;
(11) Investigate and initiate the proper
action for the recovery of ill-gotten
and/or unexplained wealth amassed
after February 25, 1986 and the
prosecution of the parties involved
therein.
The Ombudsman shall give priority to
complaints filed against high ranking
government officials and/or those
occupying
supervisory
positions,
complaints involving grave offenses as
well as complaints involving large sums
of money and/or properties.
Section 16. Applicability. The
provisions of this Act shall apply to all
kinds of malfeasance, misfeasance, and
non-feasance that have been committed
by any officer or employee as mentioned
in Section 13 hereof, during his tenure of
office.
Section 17. Immunities. In all
hearings, inquiries, and proceedings of
the Ombudsman, including preliminary
investigations of offenses, nor person
subpoenaed to testify as a witness shall
be excused from attending and testifying
or from producing books, papers,
correspondence, memoranda and/or
other records on the ground that the
testimony or evidence, documentary or

otherwise, required of him, may tend to


incriminate him or subject him to
prosecution: provided, that no person
shall be prosecuted criminally for or on
account of any matter concerning which
he is compelled, after having claimed
the privilege against self-incrimination,
to testify and produce evidence,
documentary or otherwise.
Under such terms and conditions as it
may determine, taking into account the
pertinent provisions of the Rules of
Court, the Ombudsman may grant
immunity from criminal prosecution to
any person whose testimony or whose
possession
and
production
of
documents or other evidence may be
necessary to determine the truth in any
hearing, inquiry or proceeding being
conducted by the Ombudsman or under
its authority, in the performance or in the
furtherance of its constitutional functions
and statutory objectives. The immunity
granted under this and the immediately
preceding paragraph shall not exempt
the witness from criminal prosecution for
perjury or false testimony nor shall he be
exempt from demotion or removal from
office.
Any refusal to appear or testify pursuant
to the foregoing provisions shall be
subject to punishment for contempt and
removal of the immunity from criminal
prosecution.
Section 18. Rules of Procedure.
(1) The Office of the Ombudsman shall
promulgate its rules of procedure for the
effective exercise or performance of its
powers, functions, and duties.
(2) The rules of procedure shall include
a provision whereby the Rules of Court
are made suppletory.
(3) The rules shall take effect after
fifteen
(15)
days
following
the

completion of their publication in the


Official Gazette or in three (3)
newspapers of general circulation in the
Philippines, one of which is printed in
the national language.
Section
19.
Administrative
Complaints. The Ombudsman shall
act on all complaints relating, but not
limited to acts or omissions which:
(1) Are contrary to law or regulation;
(2) Are unreasonable, unfair, oppressive
or discriminatory;
(3) Are inconsistent with the general
course of an agency's functions, though
in accordance with law;
(4) Proceed from a mistake of law or an
arbitrary ascertainment of facts;
(5) Are in the exercise of discretionary
powers but for an improper purpose; or
(6) Are otherwise irregular, immoral or
devoid of justification.
Section 20. Exceptions. The Office
of the Ombudsman may not conduct the
necessary
investigation
of
any
administrative
act
or
omission
complained of if it believes that:
(1) The complainant has an adequate
remedy in another judicial or quasijudicial body;
(2) The complaint pertains to a matter
outside the jurisdiction of the Office of
the Ombudsman;
(3) The complaint is trivial, frivolous,
vexatious or made in bad faith;
(4) The complainant has no sufficient
personal interest in the subject matter of
the grievance; or
(5) The complaint was filed after one (1)
year from the occurrence of the act or
omission complained of.
Section 21. Official Subject to
Disciplinary Authority; Exceptions.
The Office of the Ombudsman shall
have disciplinary authority over all

elective and appointive officials of the


Government and its subdivisions,
instrumentalities and agencies, including
Members of the Cabinet, local
government,
government-owned
or
controlled
corporations
and
their
subsidiaries, except over officials who
may be removed only by impeachment
or over Members of Congress, and the
Judiciary.
Section 22. Investigatory Power.
The Office of the Ombudsman shall
have the power to investigate any
serious misconduct in office allegedly
committed by officials removable by
impeachment, for the purpose of filing a
verified complaint for impeachment, if
warranted.
In all cases of conspiracy between an
officer or employee of the government
and a private person, the Ombudsman
and his Deputies shall have jurisdiction
to include such private person in the
investigation and proceed against such
private person as the evidence may
warrant. The officer or employee and the
private person shall be tried jointly and
shall be subject to the same penalties
and liabilities.
Section 23. Formal Investigation.
(1)
Administrative
investigations
conducted by the Office of the
Ombudsman shall be in accordance with
its rules of procedure and consistent
with due process.
(2) At its option, the Office of the
Ombudsman
may
refer
certain
complaints to the proper disciplinary
authority for the institution of appropriate
administrative
proceedings
against
erring public officers or employees,
which shall be determined within the
period prescribed in the civil service law.
Any delay without just cause in acting on

any referral made by the Office of the


Ombudsman shall be a ground for
administrative action against the officers
or employees to whom such referrals
are addressed and shall constitute a
graft offense punishable by a fine of not
exceeding
Five
thousand
pesos
(P5,000.00).
(3) In any investigation under this Act
the Ombudsman may: (a) enter and
inspect the premises of any office,
agency, commission or tribunal; (b)
examine and have access to any book,
record, file, document or paper; and (c)
hold private hearings with both the
complaining individual and the official
concerned.
Section 24. Preventives Suspension.
The Ombudsman or his Deputy may
preventively suspend any officer or
employee under his authority pending
an investigation, if in his judgment the
evidence of guilt is strong, and (a) the
charge against such officer or employee
involves dishonesty, oppression or grave
misconduct
or
neglect
in
the
performance of duty; (b) the charges
would warrant removal from the service;
or (c) the respondent's continued stay in
office may prejudice the case filed
against him.
The
preventive
suspension
shall
continue until the case is terminated by
the Office of the Ombudsman but not
more than six (6) months, without pay,
except when the delay in the disposition
of the case by the Office of the
Ombudsman is due to the fault,
negligence or petition of the respondent,
in which case the period of such delay
shall not be counted in computing the
period of suspension herein provided.
Section 25. Penalties.
(1) In administrative proceedings under

Presidential Decree No. 807, the


penalties and rules provided therein
shall be applied.
(2) In other administrative proceedings,
the penalty ranging from suspension
without pay for one (1) year to dismissal
with forfeiture of benefits or a fine
ranging from Five thousand pesos
(P5,000.00) to twice the amount
malversed, illegally taken or lost, or both
at the discretion of the Ombudsman,
taking into consideration circumstances
that mitigate or aggravate the liability of
the officer or employee found guilty of
the complaint or charges.
Section 26. Inquiries.
(1) The Office of the Ombudsman shall
inquire into acts or omissions of a public
officer, employee, office or agency
which, from the reports or complaints it
has received, the Ombudsman or his
Deputies consider to be:
(a) contrary to law or regulation;
(b) unreasonable, unfair, oppressive,
irregular or inconsistent with the general
course of the operations and functions
of a public officer, employee, office or
agency;
(c) an error in the application or
interpretation of law, rules or regulations,
or a gross or palpable error in the
appreciation of facts;
(d) based on improper motives or
corrupt considerations;
(e) unclear or inadequately explained
when reasons should have been
revealed; or
(f) inefficient performed or otherwise
objectionable.
(2) The Officer of the Ombudsman shall
receive complaints from any source in
whatever form concerning an official act
or omission.t shall act on the complaint
immediately and if it finds the same

entirely baseless, it shall dismiss the


same and inform the complainant of
such dismissal citing the reasons
therefor.f it finds a reasonable ground to
investigate further, it shall first furnish
the respondent public officer or
employee with a summary of the
complaint and require him to submit a
written answer within seventy-two (72)
hours from receipt thereof.f the answer
is found satisfactory, it shall dismiss the
case.
(3) When the complaint consists in delay
or refusal to perform a duty required by
law, or when urgent action is necessary
to protect or preserve the rights of the
complainant,
the
Office
of
the
Ombudsman shall take steps or
measures and issue such orders
directing the officer, employee, office or
agency concerned to:
(a) expedite the performance of duty;
(b) cease or desist from the
performance of a prejudicial act;
(c) correct the omission;
(d) explain fully the administrative act in
question; or
(e) take any other steps as may be
necessary under the circumstances to
protect and preserve the rights of the
complainant.
(4) Any delay or refusal to comply with
the referral or directive of the
Ombudsman or any of his Deputies,
shall
constitute
a
ground
for
administrative disciplinary action against
the officer or employee to whom it was
addressed.
Section 27. Effectivity and Finality of
Decisions. (1) All provisionary orders
of the Office of the Ombudsman are
immediately effective and executory.
A motion for reconsideration of any
order, directive or decision of the Office

of the Ombudsman must be filed within


five (5) days after receipt of written
notice and shall be entertained only on
any of the following grounds:
(1) New evidence has been discovered
which materially affects the order,
directive or decision;
(2) Errors of law or irregularities have
been committed prejudicial to the
interest of the movant. The motion for
reconsideration shall be resolved within
three (3) days from filing: provided, that
only one motion for reconsideration shall
be entertained.
Findings of fact by the Officer of the
Ombudsman when supported by
substantial evidence are conclusive. Any
order, directive or decision imposing the
penalty of public censure or reprimand,
suspension of not more than one (1)
month's salary shall be final and
unappealable.
In all administrative disciplinary cases,
orders, directives, or decisions of the
Office of the Ombudsman may be
appealed to the Supreme Court by filing
a petition for certiorari within ten (10)
days from receipt of the written notice of
the order, directive or decision or denial
of the motion for reconsideration in
accordance with Rule 45 of the Rules of
Court.
The above rules may be amended or
modified by the Office of the
Ombudsman as the interest of justice
may require.
Section
28.
Investigation
in
Municipalities, Cities and Provinces.
The Office of the Ombudsman may
establish offices in municipalities, cities
and provinces outside Metropolitan
Manila, under the immediate supervision
of the Deputies for Luzon, Visayas and
Mindanao,
where
necessary
as

determined by the Ombudsman. The


investigation of complaints may be
assigned to the regional or sectoral
deputy concerned or to a special
investigator who shall proceed in
accordance with the rules or special
instructions or directives of the Office of
the Ombudsman. Pending investigation
the deputy or investigator may issue
orders and provisional remedies which
are immediately executory subject to
review by the Ombudsman. Within three
(3)
days
after
concluding
the
investigation, the deputy or investigator
shall transmit, together with the entire
records of the case, his report and
conclusions to the Office of the
Ombudsman. Within five (5) days after
receipt of said report, the Ombudsman
shall render the appropriate order,
directive or decision.
Section 29. Change of Unjust Laws.
If the Ombudsman believes that a law
or regulation is unfair or unjust, he shall
recommend to the President and to
Congress the necessary changes
therein or the repeal thereof.
Section 30. Transmittal/Publication of
Decision. In every case where the
Ombudsman has reached a decision,
conclusion or recommendation adverse
to a public official or agency, he shall
transmit his decision, conclusion,
recommendation or suggestion to the
head of the department, agency or
instrumentality, or of the province, city or
municipality
concerned
for
such
immediate action as may be necessary.
When transmitting his adverse decision,
conclusion or recommendation, he shall,
unless excused by the agency or official
affected, include the substance of any
statement the public agency or official
may have made to him by way of

explaining past difficulties with or


present rejection of the Ombudsman's
proposals.
Section
31.
Designation
of
Investigators and Prosecutors. The
Ombudsman may utilize the personnel
of his office and/or designate or deputize
any fiscal, state prosecutor or lawyer in
the government service to act as special
investigator or prosecutor to assist in the
investigation and prosecution of certain
cases. Those designated or deputized to
assist him herein provided shall be
under his supervision and control.
The Ombudsman and his investigators
and prosecutors, whether regular
members of his staff or designated by
him as herein provided, shall have
authority to administer oaths, to issue
subpoena and subpoena duces tecum,
to summon and compel witnesses to
appear and testify under oath before
them and/or bring books, documents
and other things under their control, and
to secure the attendance or presence of
any absent or recalcitrant witness
through
application
before
the
Sandiganbayan or before any inferior or
superior court having jurisdiction of the
place where the witness or evidence is
found.
Section 32. Rights and Duties of
Witness.
(1) A person required by the
Ombudsman to provide the information
shall be paid the same fees and travel
allowances as are extended to
witnesses whose attendance has been
required in the trial courts. Upon request
of the witness, the Ombudsman shall
also furnish him such security for his
person and his family as may be
warranted by the circumstances. For this
purpose, the Ombudsman may, at its

expense, call upon any police or


constabulary unit to provide the said
security.
(2) A person who, with or without service
or compulsory process, provides oral or
documentary information requested by
the Ombudsman shall be accorded the
same privileges and immunities as are
extended to witnesses in the courts, and
shall likewise be entitled to the
assistance of counsel while being
questioned.
(3) If a person refuses to respond to the
Ombudsman's
or
his
Deputy's
subpoena, or refuses to be examined, or
engages in obstructive conduct, the
Ombudsman or his Deputy shall issue
an order directing the person to appear
before him to show cause why he
should not be punished for contempt.
The contempt proceedings shall be
conducted pursuant to the provisions of
the Rules of Court.
Section
33.
Duty
to
Render
Assistance to the Office of the
Ombudsman. Any officer or
employee of any department, bureau or
office,
subdivision,
agency
or
instrumentality of the Government,
including
government-owned
or
controlled corporations and local
governments, when required by the
Ombudsman, his Deputy or the Special
Prosecutor shall render assistance to
the Office of the Ombudsman.
Section 34. Annual Report. The
Office of the Ombudsman shall render
an annual report of its activities and
performance to the President and to
Congress to be submitted within thirty
(30) days from the start of the regular
session of Congress.
Section 35. Malicious Prosecution.
Any person who, actuated by malice or

gross bad faith, files a completely


unwarranted or false complaint against
any government official or employee
shall be subject to a penalty of one (1)
month and one (1) day to six (6) months
imprisonment and a fine not exceeding
Five thousand pesos (P5,000.00).
Section 36. Penalties for Obstruction.
Any person who willfully obstructs or
hinders the proper exercise of the
functions of the Office of the
Ombudsman or who willfully misleads or
attempts to mislead the Ombudsman,
his Deputies and the Special Prosecutor
in replying to their inquiries shall be
punished by a fine of not exceeding Five
thousand pesos (P5,000.00).
Section 37. Franking Privilege. All
official mail matters and telegrams of the
Ombudsman addressed for delivery
within the Philippines shall be received,
transmitted, and delivered free of
charge: provided, that such mail matters
when addressed to private persons or
nongovernment offices shall not exceed
one hundred and twenty (120) grams. All
mail matters and telegrams sent through
government
telegraph
facilities
containing complaints to the Office of
the Ombudsman shall be transmitted
free of charge, provided that the
telegram shall contain not more than
one hundred fifty (150) words.
Section 38. Fiscal Autonomy. The
Office of the Ombudsman shall enjoy
fiscal autonomy. Appropriations for the
Office of the Ombudsman may not be
reduced below the amount appropriated
for the previous years and, after
approval, shall be automatically and
regularly released.
Section 39. Appropriations. The
appropriation for the Office of the
Special Prosecutor in the current

General Appropriations Act is hereby


transferred to the Office of the
Ombudsman. Thereafter, such sums as
may be necessary shall be included in
the annual General Appropriations Act.
Section 40. Separability Clause. If
any provision of this Act is held
unconstitutional, other provisions not
affected thereby shall remain valid and
binding.
Section 41. Repealing Clause. All
laws, presidential decrees, letters of
instructions, executive orders, rules and
regulations insofar as they are
inconsistent with this Act, are hereby
repealed or amended as the case may
be.
Section 42. Effectivity. This Act
shall take effect after fifteen (15) days
following its publication in the Official
Gazette or in three (3) newspapers of
general circulation in the Philippines.
AO 7

RULE II
PROCEDURE IN CRIMINAL CASES
SECTION 1. GROUNDS. A criminal
complaint may be brought for an offense in
violation of R.A. 3019, as amended, R.A. 1379,
as amended, R.A. 6713, Title VII Chapter II,
Section 2 of the Revised Penal Code, and for
such other offenses committed by public
officers and employees in relation to office.
Sec. 2. EVALUATION. Upon evaluating the
complaint, the investigating officer shall
recommend whether it may be:
a) dismissed outright for want of palpable merit;
b) referred to respondent for comment;
c) indorsed to the proper government office or
agency which has jurisdiction over the case;
d) forwarded to the appropriate office or official
for fact-finding investigation;
e) referred for administrative adjudication; or
f) subjected to a preliminary investigation.
Sec. 3. PRELIMINARY INVESTIGATION;
WHO MAY CONDUCT. Preliminary
investigation may be conducted by any of the
following:

1) Ombudsman Investigators;
2) Special Prosecuting Officers;
3) Deputized Prosecutors;
4) Investigating Officials authorized by law to
conduct preliminary investigations; or
5) Lawyers in the government service, so
designated by the Ombudsman.
Sec. 4. PROCEDURE. Preliminary
investigation of cases falling under the
jurisdiction of the Sandiganbayan and Regional
Trial Courts shall be conducted in the manner
prescribed in Section 3, Rule 112 of the Rules
of Court, subject to the following provisions:
a) If the complaint is not under oath or is based
only on official reports, the investigating officer
shall require the complainant or supporting
witnesses to execute affidavits to substantiate
the complaints.
b) After such affidavits have been secured, the
investigating officer shall issue an order,
attaching thereto a copy of the affidavits and
other supporting documents, directing the
respondent to submit, within ten (10) days from
receipt thereof, his counter-affidavits and
controverting evidence with proof of service
thereof on the complainant. The complainant
may file reply affidavits within ten (10) days
after service of the counter-affidavits.
c) If the respondent does not file a counteraffidavit, the investigating officer may consider
the comment filed by him, if any, as his answer
to the complaint. In any event, the respondent
shall have access to the evidence on record.
d) No motion to dismiss shall be allowed
except for lack of jurisdiction. Neither may be
motion for a bill of particulars be entertained. If
respondent desires any matter in the
complainants affidavit to be clarified, the
particularization thereof may be done at the
time of clarificatory questioning in the manner
provided in paragraph (f) of this section.
e) If the respondent cannot be served with the
order mentioned in paragraph 6 hereof, or
having been served, does not comply
therewith, the complaint shall be deemed
submitted for resolution on the basis of the
evidence on record.
f) If, after the filing of the requisite affidavits and
their supporting evidences, there are facts
material to the case which the investigating
officer may need to be clarified on, he may
conduct a clarificatory hearing during which the

parties shall be afforded the opportunity to be


present but without the right to examine or
cross-examine the witness being questioned.
Where the appearance of the parties or
witnesses is impracticable, the clarificatory
questioning may be conducted in writing,
whereby the questions desired to be asked by
the investigating officer or a party shall be
reduced into writing and served on the witness
concerned who shall be required to answer the
same in writing and under oath.
g) Upon the termination of the preliminary
investigation, the investigating officer shall
forward the records of the case together with
his resolution to the designated authorities for
their appropriate action thereon.
No information may be filed and no complaint
may be dismissed without the written authority
or approval of the Ombudsman in cases falling
within the jurisdiction of the Sandiganbayan, or
of the proper Deputy Ombudsman in all other
cases.
Sec. 5. CASES FALLING UNDER THE
JURISDICTION OF MUNICIPAL TRIAL
COURTS. Cases falling under the
jurisdiction of the Office of the Ombudsman
which are cognizable by municipal trial courts,
including those subject to the Rule of Summary
Procedure may only be filed in court by
information approved by the Ombudsman or
the proper Deputy Ombudsman.
Sec. 6. NOTICE TO PARTIES. The parties
shall be served with a copy of the resolution as
finally approved by the Ombudsman or by the
proper Deputy Ombudsman.
Sec. 7. MOTION FOR RECONSIDERATION.
a) Only one (1) motion for reconsideration or
reinvestigation of an approved order or
resolution shall be allowed, the same to be filed
within fifteen (15) days from notice thereof with
the Office of the Ombudsman, or the Deputy
Ombudsman as the case may be.
b) No motion for reconsideration or
reinvestigation shall be entertained after the
information shall have been filed in court,
except upon order of the court wherein the
case was filed.
III.

PROCEDURE IN
REQUIRING
INVESTIGATION

CASES NOT
PRELIMINARY

A. MTC CASES OR THOSE COVERED BY


SUMMARY PROCEDURE
RULE 112 Section 9. Cases not requiring a

preliminary investigation nor covered by the


Rule on Summary Procedure.
(a) If filed with the prosecutor. If the
complaint is filed directly with the prosecutor
involving an offense punishable by
imprisonment of less four (4) years, two (2)
months and one (1) day, the procedure outlined
in section 3(a) of this Rule shall be observed.
The prosecutor shall act on the complaint
based on the affidavits and other supporting
documents submitted by the complainant within
ten (10) days from its filing.
(b) If filed with the Municipal Trial Court. If
the complaint or information is filed directly with
the Municipal Trial Court or Municipal Circuit
Trial Court for an offense covered by this
section, the procedure in section 3(a) of this
Rule shall be observed. If within ten (10) days
after the filing of the complaint or information,
the judge finds no probable cause after
personally evaluating the evidence, or after
personally examining in writing and under oath
the complainant and his witnesses in the form
of searching question and answers, he shall
dismiss the same. He may, however, require
the submission of additional evidence, within
ten (10) days from notice, to determine further
the existence of probable cause. If the judge
still finds no probable cause despite the
additional evidence, he shall, within ten (10)
days from its submission or expiration of said
period, dismiss the case. When he finds
probable cause, he shall issue a warrant of
arrest, or a commitment order if the accused
had already been arrested, and hold him for
trial. However, if the judge is satisfied that there
is no necessity for placing the accused under
custody, he may issue summons instead of a
warrant of arrest. (9a)
B. WHEN
PERSON
LAWFULLY
ARRESTED WITHOUT WARRANT
Section 7. When accused lawfully
arrested without warrant. When a person is
lawfully arrested without a warrant involving an
offense
which
requires
a
preliminary
investigation, the complaint or information may
be filed by a prosecutor without need of such
RULE 112

investigation provided an inquest has been


conducted in accordance with existing rules. In
the absence or unavailability of an inquest
prosecutor, the complaint may be filed by the
offended party or a peace office directly with
the proper court on the basis of the affidavit of
the offended party or arresting officer or
person.
Before the complaint or information is filed, the
person arrested may ask for a preliminary
investigation in accordance with this Rule, but
he must sign a waiver of the provisions of
Article 125 of the Revised Penal Code, as
amended, in the presence of his counsel.
Notwithstanding the waiver, he may apply for
bail and the investigation must be terminated
within fifteen (15) days from its inception.
After the filing of the complaint or information in
court without a preliminary investigation, the
accused may, within five (5) days from the time
he learns of its filing, ask for a preliminary
investigation with the same right to adduce
evidence in his defense as provided in this
Rule.
Inquest Procedures (DOJ Circular No. 61, 1993)
Evidence Needed for an Inquest Proceedings
SECTION 1.Concept .Inquest is an informal and
summary investigation con-ducted by a public
prosecutor in criminal cases involving persons
arrested and detained without the benefit of a
warrant of arrest issued by the court for the purpose
of deter-mining whether or not said persons should
remain under custody and correspondingly be
charged in court.
SEC. 2.Designationof Inquest Officers.The
City or Provincial Prosecutor shall designate the
Prosecutors assigned to inquest duties and shall
furnish the Philippine National Police (PNP) a list
of their names and their schedule of assignments. If,
however, there is only one Prosecutor in the area, all
inquest cases shall be referred to him for
appropriate action. Unless otherwise directed by the
City or Provincial Prosecutor, those assigned to
inquest duties shall discharge their functions during
the hours of their designated assignments and only
at the police stations/headquarters of the PNP in
order to expedite and facilitate the disposition of
inquest cases.

SEC. 3.Commencement and Termination of


Inquest.The inquest proceedings shall be
considered commenced upon receipt by the Inquest
Officer from the law enforcement authorities of the
complaint/referral documents which should include:
a. the affidavit of arrest;
b. the investigation report;
c. the statement of the complainant and
witnesses; and
d. other supporting evidence gathered by the
police in the course of the latters
investigation of the criminal incident
involving the arrested or detained person.
The inquest Officer shall, as far as
practicable, cause the affidavit of arrest and
statements/affidavits of the complainant and
the witnesses to be subscribed and sworn to
before him by the arresting officer and the
affiants. The inquest proceedings must be
terminated within the period prescribed
under the provisions of Article 125 of the
Revised Penal Code, as amended. *
SEC. 4.Particular Documents Required in
Specific Cases.The submission, presentation of
the documents listed herein below should as far as
practicable, be required in the following cases by
the Inquest Officer.
Violation of the Anti-Fencing Law (PD 1612)
a. a list/inventory of the articles and items
subject of the offense; and
b. statement of their respective value
Illegal Possession of Explosives (PD 1866)
a. chemistry report duly signed by the
forensic chemist and
b. photograph of the explosives, if readily
available.
Violation of the Fisheries Law (PD 704)(now RA
8550)
a. photograph of the confiscated fish, if
readily available; and
b. certification of the Bureau of Fisheries
and Aquatic Resources;
Violation of the Forestry Law (PD 705)
a. scale sheets containing the volume and
species of the forest products confiscated,
number of pieces and other important details

such as estimated value of the products


confiscated;
b.
certification
of
Department
of
Environment and Natural Resources/Bureau
of Forest Management; and
c. seizure receipt. The submission of the
foregoing documents shall no absolutely be
required if there are other forms of evidence
submitted which will sufficiently establish
the facts sought to be proved by the
foregoing documents.
SEC. 5.Incomplete documents.When the
documents presented are not complete to establish
probable cause, the Inquest Officer shall direct the
law enforcement agency to submit the required
evidence within the period prescribed under the
provisions of Article 125 of the Revised Penal
Code, as amended; otherwise, the Inquest Officer
shall order the release of the detained person and,
where the inquest is conducted outside of office
hours, direct the law enforcement agency concerned
to file the case with the City or Provincial
Prosecutor for appropriate action.
SEC. 6.Presence of the detained person.The
presence of the detained person who is under
custody shall be ensured during the proceedings.
However, the production of the detained person
before the Inquest Officer may be dispensed with in
the following cases:
a. if he is confined in a hospital;
b. if he is detained in a place under
maximum security;
c. if production of the detained person
involve security risks; or
d. if the presence of the detained person is
not feasible by reason of age, health, sex and
other similar factors.
The absence of the detained person by reason of any
of the foregoing factors must be noted by the
Inquest Officer and reflected in the record of the
case.
SEC. 7.Charges and counter-charges.All
charges and counter-charges arising from the same
incident shall, as far as practicable, be consolidated
and inquested jointly to avoid contradictory or
inconsistent dispositions.

SEC. 8.Initial duty of the inquest officer .The


Inquest Officer must first deter-mine if the arrest of
the detained person was made in accordance with
the provisions of paragraphs (a) and (b) of Section
5, Rule 113 of the 1985 Rules on Criminal
Procedure, as amended, which provide that arrests
without a warrant may be effected:
a. when, in the presence of the arresting
officer, the person to be arrested has
committed, is actually committing, or is
attempting to commit an offense; or
b. when an offense has in fact just been
committed, and the arresting officer has
personal knowledge of facts indicating that
the person to be arrested has committed it.
For this purpose, the Inquest Officer may
summarily examine the arresting officers on
the circumstances surrounding the arrest or
apprehension of the detained per-son.
SEC. 9.Where arrest not properly effected.
Should the Inquest Officer find that the arrest was
not made in accordance with the Rules, he shall:
a. recommend the release of the person
arrested or detained;
b. note down the disposition of the referral
document;
c. prepare a brief memorandum indicating
the reasons for the action taken; and
d. forward the same, together with the record
of the case, to the City or Provincial
Prosecutor for appropriate action.
Where the recommendation for the release of the
detained person is approved by the City or
Provincial Prosecutor but the evidence on hand
warrant the conduct of a regular preliminary
investigation, the order of release shall be served on
the officer having custody of said detainee and shall
direct the said officer to serve upon the detainee the
subpoena or notice of preliminary investigation,
together with the copies of the charge sheet or
complaint, affidavits or sworn statements of the
complainant and his witnesses and other supporting
evidence.
SEC. 10.Where the arrest property effected.
Should the Inquest Officer find that the arrest was
properly effected, the detained person should be
asked if he desires to avail himself of a preliminary

investigation, if he does, he shall be made to


execute a waiver of the provisions of Article 125 of
the Revised Penal Code, as amended, with the
assistance of a lawyer and, in case of nonavailability of a lawyer, a responsible person of his
choice. The preliminary investigation may be
conducted by the Inquest Officer himself or by any
other Assistant Prosecutor to whom the case may be
assigned by the City or Provincial Prosecutor, which
investigation shall be terminated within fifteen(15)
days from its inception.
SEC. 11.Inquest proper .Where the detained
person does not opt for a preliminary investigation
or otherwise refuses to execute the required waiver,
the Inquest Officer shall proceed with the inquest by
examining the sworn statements/affidavits of the
complainant and the witnesses and other supporting
evidence submitted to him. If necessary, the Inquest
Officer may require the presence of the complainant
and witnesses and subject them to an informal and
summary investigation or examination for purposes
of determining the existence of probable cause.
SEC. 12.Meaning of probable cause.Probable
cause exists when the evidence submitted to the
Inquest Officer engenders a well-founded belief that
a crime has been committed and that the arrested or
detained person is probably guilty thereof.
SEC. 13.Presence of probable cause.If the
Inquest Officer finds that probable cause exists, he
shall forthwith prepare the corresponding
complaint/information with the recommendation
that the same be filed in court. The
complaint/information shall indicate the offense
committed and the amount of bail recommended, if
bailable. Thereafter, the record of the case, together
with the prepared com-plaint/information, shall be
forwarded to the City or Provincial Prosecutor for
appropriate action. The complaint/information may
be filed by the Inquest Officer himself or by any
other Assistant Prosecutor to whom the case may be
assigned by the City or Provincial Prosecutor.
SEC.
14.Contents
of
information.The
information shall, among others, contain:
a. a certification by the filing Prosecutor that
he is filing the same in accordance with the
provisions of Section 7, Rule 112 of the
1985 Rules on Criminal Procedure, as

amended, in cases cognizable by the


Regional Trial Court;
b. the full name and alias, if any, and address
of the accused;
c. the place where the accused is actually detained;
d. the full names and addresses of the
complainant and witnesses;
e. a detailed description of the recovered
item, if any;
f. the full name and address of the evidence
custodian;
g. the age and date of birth of the
complainant or the accused, if eighteen
(19)years of age or below; and
h. the full names and addresses of the
parents, custodians or guardians of the minor
complainant or accused, as the case may be.
SEC. 15.Absence of probable cause.If the
Inquest Officer finds no probable cause, he shall:
a. recommend the release of the arrested or
detained person;
b. note down his disposition on the referral
document;
c. prepare a brief memorandum indicating
the reasons for the action taken; and
d. forthwith forward the record of the case to
the City or Provincial Prosecutor
for
appropriate action.
If the recommendation of the Inquest Officer for the
release of the arrested or detained person is
approved, the order of release shall be served on the
officer having custody of the said detainee. Should
the City or Provincial Prosecutor disapprove the
recommendation of release, the arrested or detained
person shall remain under custody, and the
correspond-ing complaint/information shall be filed
by the City or Provincial Prosecutor or by any
Assistant Prosecutor to whom the case may be
assigned.
SEC. 16.Presence at the crime scene.Whenever
a dead body is found and there is reason to believe
that the death resulted from foul play, or from the
unlawful acts or omissions of other persons and
such fact has been brought to his attention, the
Inquest Officer shall:
a. forthwith proceed to the crime scene or
place of discovery of the dead person;

b. cause an immediate autopsy to be


conducted by the appropriate medico-legal
officer in the locality or the PNP medicolegal division or the NBI medico-legal
office, as the case may be;
c. direct the police investigator to cause the
taking of photographs of the crime scene or
place of discovery of the dead body;
d. supervise the investigation to be
conducted by the police authorities as well
as the recovery of all articles and pieces of
evidence found thereat and see to it that the
same are safeguarded and the chain of the
custody thereof properly recorded; and
e. submit a written report of his finding to
the City or Provincial Prosecutor for
appropriate action.
SEC. 17.Sandiganbayan cases.Should any
complaint cognizable by the Sandiganbayan be
referred to an Inquest Officer for investigation, the
latter shall, after conducting the corresponding
inquest proceeding, forthwith forward the complete
record to the City or Provincial Prosecutor for
appropriate action.

Art. 125. Delay in the


delivery
of
detained
persons
to
the
proper
judicial authorities. The
penalties provided in the
next preceding article shall
be imposed upon the public
officer or employee who
shall detain any person for
some legal ground and
shall fail to deliver such
person
to
the
proper
judicial authorities within
the period of; twelve (12)
hours,
for
crimes
or
offenses
punishable
by
light penalties, or their
equivalent; eighteen (18)

hours,
for
crimes
or
offenses
punishable
by
correctional penalties, or
their equivalent and thirtysix (36) hours, for crimes,
or offenses punishable by
afflictive
or
capital
penalties,
or
their
equivalent.
In every case, the person
detained shall be informed
of
the
cause
of
his
detention and shall be
allowed upon his request,
to communicate and confer
at any time with his
attorney or counsel.
chanrobles virtual law library

RJCL

Section 8. Procedure for


Handling Children Exempted from
Criminal Liability. - If it is determined at
the initial contact that the child is 15
years of age or below, the procedure
provided in Section 20, Republic Act No.
9344 shall be observed as follows:
(a) The authority who had the initial
contact with the child shall immediately
release the child to the custody of the
mother or father, or the appropriate
guardian or custodian, or in their
absence, the nearest relative.
(b) The authority shall immediately notify
the local social welfare and development
officer of the taking of the child into
custody.
(c) The local social welfare and
development officer shall, with the
consent of the child and the person
having custody over the child, determine
the appropriate intervention programs

for the child.


(d) If the child's parents, guardians or
nearest relatives cannot be located, or if
they refuse to take custody, the child
may be released to any of the following:
a duly registered nongovernmental or
religious organization; a barangay
official or a member of the Barangay
Council for the Protection of Children; a
local social welfare and development
officer; or, when and where appropriate,
the Department of Social Welfare and
Development.
(e) If the child has been found by the
local social welfare and development
office to be abandoned, neglected or
abused by the parents, or if the parents
and the child do not consent to or do not
comply with the prevention program, the
Department of Social Welfare and
Development or the Local Social
Welfare and Development Office shall
file before the court a petition for
involuntary commitment pursuant to
Presidential Decree No. 603, otherwise
known as "The Child and Youth Welfare
Code." (a)
IV.

REMEDIES
FOR
INVESTIGATION
A. APPEAL

PRELIMINARY

DEPARTMENT CIRCULAR NO. 70

SECTION 2. Where to appeal. An appeal may be


brought to the Secretary of Justice within the period
and in the manner herein provided.
SECTION 3. Period to appeal. The appeal shall be
taken within fifteen (15) days from receipt of the
resolution, or of the denial of the motion for
reconsideration/reinvestigation if one has been filed
within fifteen (15) days from receipt of the assailed
resolution. Only one motion for reconsideration
shall be allowed.
SECTION 4. How appeal taken. An aggrieved party
may appeal by filing a verified petition for review
with the Office of the Secretary, Department of
Justice, and by furnishing copies thereof to the
adverse party and the Prosecution Office issuing
the appealed resolution.
SECTION 5. Contents of petition. - The petition
shall contain or state: (a) the names and addresses
of the parties; (b) the Investigation Slip number (I.S.
No.) and criminal case number, if any, and title of
the case, including the offense charged in the
complaint; (c) the venue of the preliminary
investigation; (d) the specific material dates
showing that it was filed on time; (e) a clear and
concise statement of the facts, the assignment of
errors, and the reasons or arguments relied upon
for the allowance of the appeal; and (f) proof of
service of a copy of the petition to the adverse party
and the Prosecution Office concerned.
The petition shall be accompanied by legible
duplicate original or certified true copy of the
resolution appealed from together with legible true
copies of the complaint, affidavits/sworn statements
and other evidence submitted by the parties during
the preliminary investigation/ reinvestigation.

In the interest of expeditious and efficient


administration of justice and in line with recent
jurisprudence, the following Rule governing appeals
from resolutions of prosecutors in the National
Prosecution Service, to be known as the 2000 NPS
Rule on Appeal, is hereby adopted.

If an information has been filed in court pursuant to


the appealed resolution, a copy of the motion to
defer proceedings filed in court must also
accompany the petition. The
investigating/reviewing/approving prosecutor shall
not be impleaded as party respondent in the
petition. The party taking the appeal shall be
referred to in the petition as either "ComplainantAppellant" or "Respondent- Appellant".

SECTION 1. Scope. - This Rule shall apply to


appeals from resolutions of the Chief State
Prosecutor, Regional State Prosecutors and
Provincial/City Prosecutors in cases subject of
preliminary investigation/ reinvestigation.

SECTION 6. Effect of failure to comply with


requirements. The failure of the petitioner to comply
with any of the foregoing requirements shall
constitute sufficient ground for the dismissal of the
petition.

SUBJECT : 2000 NPS RULE ON APPEAL

SECTION 7. Action on the petition. The Secretary


of Justice may dismiss the petition outright if he
finds the same to be patently without merit or
manifestly intended for delay, or when the issues
raised therein are too unsubstantial to require
consideration. If an information has been filed in
court pursuant to the appealed resolution, the
petition shall not be given due course if the
accused had already been arraigned. Any
arraignment made after the filing of
the petition shall not bar the Secretary of Justice
from exercising his power of review.
SECTION 8. Comment. Within a non-extendible
period of fifteen (15) days from receipt of a copy of
the petition, the adverse party may file a verified
comment, indicating therein the date of such receipt
and submitting proof of service of his comment to
the petitioner and the Prosecution Office
concerned. Except when directed by the Secretary
of Justice, the investigating/reviewing/approving
prosecutor need not submit any comment.
If no comment is filed within the prescribed period,
the appeal shall be resolved on the basis of the
petition.
SECTION 9. Effect of the appeal. Unless the
Secretary of Justice directs otherwise, the appeal
shall not hold the filing of the corresponding
information in court on the basis of the finding of
probable cause in the appealed resolution.
The appellant and the trial prosecutor shall see to it
that, pending resolution of the appeal, the
proceedings in court are held in abeyance.
SECTION 10. Withdrawal of appeal.
Notwithstanding the perfection of the appeal, the
petitioner may withdraw the same at any time
before it is finally resolved, in which case the
appealed resolution shall stand as though no
appeal has been taken.
SECTION 11. Reinvestigation. If the Secretary of
Justice finds it necessary to reinvestigate the case,
the reinvestigation shall be held by the investigating
prosecutor, unless, for compelling reasons, another
prosecutor is designated to conduct the same.
SECTION 12. Disposition of the appeal. The
Secretary may reverse, affirm or modify the
appealed resolution. He may, motu proprio or upon
motion, dismiss the petition for review on any of the
following grounds:

That the petition was filed beyond the period


prescribed in Section 3 hereof;

That the procedure or any of the


requirements herein provided has not been
complied with;

That there is no showing of any reversible


error;

That the appealed resolution is interlocutory


in nature, except when it suspends the
proceedings based on the alleged existence
of a prejudicial question;

That the accused had already been


arraigned when the appeal was taken;

That the offense has already prescribed;


and

That other legal or factual grounds exist to


warrant a dismissal.
SECTION 13. Motion for reconsideration.
The aggrieved party may file a motion for
reconsideration within a non-extendible
period of ten (10) days from receipt of the
resolution on appeal, furnishing the adverse
party and the Prosecution Office concerned
with copies thereof and submitting proof of
such service. No second or further motion
for reconsideration shall be entertained.
SECTION 14. Repealing clause. This
Circular supersedes Department Order No.
223 dated June 30, 1993 and all other
Department issuances inconsistent
herewith.
SECTION 15. Effectivity. This Circular shall
be published once in two (2) newspapers of
general circulation, after which it shall take
effect on September 1, 2000.

DIMATULAC V VILLON
Facts:
SP03 Virgilio Dimatulac was shot dead at his
residence in Pampanga. A complaint for
murder was filed in the MTC and after
preliminary investigation, Judge Designate
David issued warrants of arrest against the
accused.
Only David, Mandap, Magat, and Yambao were

arrested and it was only Yambao who


submitted his counter-affidavit. Judge
David then issued a resolution finding
reasonable ground that the crime of
murder has been committed and that the
accused is probably guilty thereof.
Though it was not clear whether Pampanga
Assistant Provincial
Prosecutor
Sylvia
Alfonso-Flores acted motu proprio, or upon
motion of the private respondents, she
conducted a reinvestigation and resolved
that the Yabuts and Danny were in
conspiracy, along with the other accused,
and committed homicide.
Before the information for homicide was filed,
the Petitioner appealed the resolution of
Alfonso-Flores to the Secretary of Justice.
However, Provincial Proseutor Maranag
ordered for the release of David, Mandap,
Magat, and Naguit. An information for
homicide was also filed before the
Regional Trial Court.
Judge Raura approved the cash bonds of the
Yabuts and recalled the warrants of arrest
against them.
Private Prosecutor Amado Valdez then filed a
Motion to issue hold departure order and
Urgent Motion to defer proceedings. Judge
Roura deferred the resolution of the first
Motion and denied the second. He also set
the arraignment of the accused.
The petitioners filed a Motion to inhibit Judge
Roura for hastily setting the date for
arraignment pending the appeal in the DOJ
and for prejudging the matter. They also
filed a Petition for prohibition with the
Court of Appeals.
Public Prosecutor Datu filed a Manifestation
and Comment with the trial court and
opposed the inhibition of Roura. He also
stated that he will no longer allow the
private prosecutor to participate. Judge
Roura voluntarily inhibited himself and
was replaced by Judge Villon.
The Petitioners filed with the RTC a
Manifestation submitting documentary
evidence to support their contention that
the offense committed was murder.
Judge Villon ordered for the resetting of the
arraignment. The Yabuts entered a plea of
not guilty. The petitioners then filed a
Urgent Motion to set aside arraignment.
Secretary Guingona of the DOJ resolved the
appeal in favor of the petitioners. He also
ruled that treachery was present.
The Yabuts opposed the Manifestation
because they have already been arraigned
and they would be put under double
jeopardy.
The Secretary of Justice then set aside his
order and the appeal was held not and

academic due to the previous arraignment


of the accused for homicide.
Judge Villon denied the Motion to set aside
arraignment.
The
motion
for
reconsideration was also denied. Hence,
this petition for certiorari/prohibition and
mandamus.
Issues:
Whether the Office of the Provincial
Prosecutor committed grave abuse of
discretion in reinvestigating the case
without having the respondents within the
custody of the law and for filing the
information pending the appeal of the
resolution with the DOJ.
Whether Hon. Villon acted with grave abuse of
discretion
in
proceeding
with
the
arraignment and for denying the Motions
to set aside the arraignment.
Whether the Secretary of Justice committed
grave abuse of discretion in reconsidering
his order.
Decision:
Petition is GRANTED.
Alfonso-Reyes was guilty of having acted
with grave abuse of discretion for conducting a
reinvestigation despite the fact that the Yabuts
were still at large. Though Sec. 5, Rule 112 states
that the prosecutor is not bound by the findings
of the judge who conducted the investigation, the
resolution should be based on the review of the
record and evidence transmitted. Hence, she
should have sustained the recommendation since
all the accused, except Yambao, failed to file their
counter-affidavits. It is impossible for AlfonsoReyes to not have known the appeal filed with the
DOJ. The filing of an appeal is provided in Sec. 4,
Rule 112 of the Rules of Court. There is nothing in
the law which prohibits the filing of an appeal
once an information is filed.
Judge Roura acted with grave abuse of
discretion for deferring the resolution to the
motion for a hold departure order. Since the
accused were out on bail, the Motion should have
been granted since they could have easily fled.
Though he is not bound to the resolution of the
DOJ, he should have perused the documents
submitted.
The DOJ was also in grave abuse of its
discretion for setting aside its order. In doing so, it
has relinquished its power of control and
supervision of the Public Prosecutor. The state
has been deprived of due process. Hence, the
dismissal of the case is null and void and double

jeopardy cannot be invoked by the accused.


B.
REINVISTIGATION/PRELIMINARY
INVESTIGATION
CRESPO VS MOGUL
NATURE
Petition to review the decision of the Circuit Criminal Court of Lucena City (petitioner
prays that respondent judge be perpetually enjoined from enforcing his threat to
proceed with the arraignment and trail of petitioner, ordering respondent Judge to
dismiss the said case, and declaring the obligation of petitioner as purely civil.)
FACTS
- Assistant Fiscal Proceso de Gala filed an information for estafa against Mario Crespo
in Circuit Criminal Court of Lucena City. When the case was set for arraignment, the
accused filed a motion to defer arraignment on the ground that there was a pending
petition for review filed with the Secretary of Justice of the resolution of the Office of
the Provincial Fiscal for the filing of the information. The presiding judge (leodegario
Mogul) denied the motion through his order.
- The accused filed a petition for certiorari and prohibition with prayer for a preliminary
writ of injunction. In an order (Aug 17 1977), the CA restrained Judge Mogul from
proceeding with the arraignment of the accused until further orders from the Court
- On May 15 1978, a decision was made by the CA granting the writ and perpetually
restraining the judge from enforcing his threat to compel the arraignment of the
accused in the case until the Dept of Justice shall have finally resolved the petition for
review.
- On March 22, 1978, The Undersecretary of Justice Hon Catalino Macaraig Jr,
resolving the petition for review, reversed the resolution of the Office of the Provincial
Fiscal and directed the fiscal to move for immediate dismissal of the information filed
against the accused. The Provincial Fiscal filed a motion to dismiss for insufficiency of
evidence on April 10, 1978. On November 24 1978, The Judge denied the motion and
set the arraignment
- The accused filed a petition for certiorari, prohibition, and mandamus with petition for
the issuance of preliminary writ of prohibition and/or temporary restraining order in the
CA. On January 23 1979, a restraining order was issued by the CA against the
threatened act of arraignment of the accused. However, in a decision of October 25
1979, the CA dismissed the petition and lifted the restraining order of Jan 23,1979. The
motion for reconsideration of the accused was denied in a resolution.
ISSUE
WON the trial court acting on a motion to dismiss a criminal case filed by the Provincial
Fiscal upon instructions of the Secretary of Justice to whom the case was elevated for
review, may refuse to grant the motion and insist on the arraignment and trial on the
merits
HELD
YES
Ratio Once an information is filed in court, the courts prior permission must be
secured if fiscal wants to reinvestigate the case. While it is true that the fiscal has the
quasi judicial discretion to determine whether or not a criminal case should be filed in
court or not, once the case had already been brought to Court, whatever disposition
the fiscal may feel should be proper in the case thereafter should be addressed for the
consideration of the Court.
DISPOSITION Petition dismissed

ROBERTS VS CA
FACTS
- Several thousand holders of 349 Pepsi crowns in connection with the Number
Fever Promotion filed with the Office of the City Prosecutor of Quezon City complaints
against the petitioner officials of PEPSI.
- The petitioners filed with the Office of the City Prosecutor a motion for the
reconsideration of the Joint Resolution and with the DOJ a Petition for Review. The
petitioners also Motions to Suspend Proceedings and to hold in Abeyance Issuance of
Warrants of Arrest on the ground that they had filed the aforesaid Petition for Review.
- Respondent Judge Asuncion issued the challenged order (1) denying the petitioners
Motion to Suspend Proceedings and to Hold In Abeyance Issuance of Warrants of
Arrest and the public prosecutors Motion to Defer Arraignment and (2) directing the
issuance of the warrants of arrest after and setting the arraignment on 28 June 1993.
- The petitioners filed with the Court of Appeals a special civil action for certiorari and
prohibition with application for a temporary restraining order. They contended therein
that respondent Judge Asuncion had acted without or in excess of jurisdiction or with
grave abuse of discretion in issuing the aforementioned order.
- The Court of Appeals then issued a resolution denying the application for a writ of
preliminary injunction.

ISSUE
WON public respondent Judge Asuncion committed grave abuse of discretion
in ordering the issuance of warrants of arrest without examining the records of
the preliminary investigation.
HELD
YES.
- Section 2, Article III of the present Constitution provides that no search warrant or
warrant of arrest shall issue except upon probable cause to be determined personally
by the judge after examination under oath or affirmation of the complainant and the
witnesses he may produce.
- The determination of probable cause is a function of the Judge. It is not for the
Provincial Fiscal or Prosecutor nor the Election Supervisor to ascertain. Only the
Judge and the Judge alone makes this determination.
- The preliminary inquiry made by a Prosecutor does not bind the Judge. It merely
assists him to make the determination of probable cause. The Judge does not have to
follow what the Prosecutor presents to him. By itself, the Prosecutors certification of
probable cause is ineffectual. It is the report, the affidavits, the transcripts of
stenographic notes (if any), and all other supporting documents behind the
Prosecutors certification which are material in assisting the Judge to make his
determination.
- The teachings of the cases of Soliven4, Inting5, Lim6, Allado, and Webb reject the
proposition that the investigating prosecutors certification in an information or his
resolution which is made the basis for the filing of the information, or both, would
suffice in the judicial determination of probable cause for the issuance of a warrant of
arrest.
- In the present case, nothing accompanied the information upon its filing with the trial
court. Clearly, when respondent Judge Asuncion issued the assailed order directing,
among other things, the issuance of warrants of arrest, he had only the information,
amended information, and Joint Resolution as bases thereof. He did not have the
records or evidence supporting the prosecutors finding of probable cause. And
strangely enough, he made no specific finding of probable cause; he merely directed
the issuance of warrants of arrest. It may, however, be argued that the directive
presupposes a finding of probable cause. But then compliance with a constitutional
requirement for the protection of individual liberty cannot be left to presupposition,
conjecture, or even convincing logic.

DUNGOG V CA

The Case
This petition for review on
certiorari
assails the
Decision
dated 14 May
1999 of the Court of Appeals in CAG.R. SP No. 48788, as well as the
Resolution dated 24 August 1999
denying the motion for
reconsideration. The Court of
Appeals dismissed outright the
petition for certiorari, prohibition
and mandamus filed by petitioner
Felipe Sy Dungog (Felipe) against
[if !supportFootnotes][1][endif]

[if !supportFootnotes][2][endif]

4 The Judge does not have to personally examine the complainant and his witnesses. The Prosecutor can
perform the same functions as a commissioner for the taking of the evidence. However, there should be a report
and necessary documents supporting the Fiscals bare certification. All of these should be before the Judge.

5 The supporting documents may consist of, viz., the affidavits, the transcripts of stenographic notes (if any), and
all other supporting documents behind the Prosecutors certification which are material in assisting the Judge to
make his determination of probable cause

6 The issuance of the warrants of arrest by a judge solely on the basis of the prosecutors certification in the
information that there existed probable cause, without having before him any other basis for his personal
determination of the existence of a probable cause, is null and void.

respondents. The petition


questioned the propriety of the
Order
dated 14 August
1998 (Order) and the writ of
preliminary injunction (Writ) dated
18 August 1998 issued by the
Regional Trial Court of Cebu, LapuLapu City, Branch 53 (trial court) in
Civil Case No. 5020-L.
The Antecedents
Tracing the roots of this
controversy, Felipe alleges
that he and his sister, Fortune,
agreed to sell their lots in Canjulao,
Cebu, through their parents, Juan
L. Dungog and Emma S. Dungog
(Spouses Dungog). The Spouses
Dungog convinced other lot owners
in Canjulao to sell their lots either
directly to them or to Felipe and his
sister. On 31 December 1996, the
Spouses Dungog entered into a
Contract to Sell (Contract) with
private respondent Carlos A.
Gothong Lines, Inc. (Gothong
Lines) covering several lots in
Canjulao. The lots which the
Spouses Dungog contracted to sell
to Gothong Lines belonged to
various individuals as listed in the
Contracts Annex A
which specified the corresponding
approximate land areas of each lot.
Among these was Lot 1031-F
registered in the name of Felipe
and covered by Transfer Certificate
of Title No. 10359 of the Register of
Deeds of Lapu-Lapu City. Under
the Contract, Gothong Lines was to
[if !supportFootnotes][3][endif]

[if !supportFootnotes]

[4][endif]

[if !supportFootnotes][5][endif]

pay on installment basis the


purchase price of P65,520,475.00
computed at P500 per square
meter. Thus, Gothong Lines paid a
down payment of P12,000,000.00.
For the balance of P53,520,475.00,
Gothong Lines issued
15 postdated checks of
P3,568,031.00 each beginning on
31 January 1997 as payment for 15
equal monthly installments.
Gothong Lines made good all the
checks, except the last 4 checks
dated 30 December 1997, 31
January 1998, 28 February 1998
and 30 March 1998, which
bounced due to Gothong Lines
stop payment order.
Felipe alleges further that as of 31
December 1997, his parents had
delivered 66 parcels of land to
Gothong Lines with a total area of
101,104.20 square meters valued
at P50,552,100.00. Felipe also
states that as of the same date,
Gothong Lines had paid
P51,248,345.00 in encashed
checks plus the initial down
payment of P12,000,000.00. This
left an overpayment of
P696,245.00 in the hands of the
Spouses Dungog. Felipe claims,
however, that despite Gothong
Lines stop payment order of its last
four checks, the Spouses Dungog
still delivered in February 1998, 8
parcels of land with a total land
area of 11,590 square meters
valued at P5,795,000.00. Among
[if !supportFootnotes][6][endif]

those delivered was Lot 1031-F.


The Spouses Dungog demanded
payment for these 8 parcels of
land, but Gothong Lines refused to
pay. The Spouses Dungog became
frustrated with Gothong Lines
complete silence on their demands
for payment, as well as the earlier
stop payment order on the last 4
checks. Thus, the Spouses
Dungog informed Gothong Lines in
a letter dated 18 June 1998 that
they would no longer push through
with their offer to sell the remaining
lots.
On 6 July 1998, Gothong Lines
filed a complaint for Specific
Performance, Damages with Writ
of Preliminary Mandatory
Injunction against the Spouses
Dungog to enforce the Contract.
Gothong Lines faulted the Spouses
Dungog for non-delivery of some of
the parcels of land in breach of the
Contract. Gothong Lines alleged
that while the total amount of
P51,248,348.26 paid to the
Spouses Dungog corresponds to
102,496.69 square meters, the
Spouses Dungog actually delivered
to Gothong Lines only 100,613.69
square meters. Gothong Lines
claimed that it paid an excess of
P941,848.00
corresponding to 1,883 square
meters. To protect its interest,
Gothong Lines ordered the bank to
stop payment on the remaining
postdated checks. Gothong Lines
[if !supportFootnotes][7][endif]

asked the trial court to issue a writ


of preliminary injunction to restrain
the Spouses Dungog from
canceling the Contract and from
preventing its representatives and
vehicles from passing through the
properties subject of the Contract.
Gothong Lines offered to post a
bond of P500,000.00 and
consigned the P4,048,950.00
representing the balance of the
purchase price.
Traversing Gothong Lines
allegations, the Spouses Dungog
contended that it was Gothong
Lines which breached the Contract
by stopping payment on the last 4
checks. The Spouses Dungog also
charged Gothong Lines with
competing with them in acquiring
one of the lots subject of the
Contract. They further countered
that Gothong Lines violated a
verbal agreement between them
not to develop the roads until after
30 June 1998, the last day for the
Spouses Dungog to deliver and
turn over the lots. The Spouses
Dungog opposed Gothong Lines
application for a writ of preliminary
injunction on the ground that
Gothong Lines violated the terms
of the Contract and the other
contemporaneous agreements
between them.
Based on the pleadings and
affidavits presented by the parties,
the trial court granted on 14 August
1998 Gothong Lines prayer for

injunction. The dispositive portion


of the Order reads:
WHEREFORE,inthelightofthe
foregoingconsiderations,plaintiffs
applicationfortheissuanceofawritof
preliminaryinjunctionisGRANTED.
Consequently,afterthefilingand
approvalofabondintheamountof
ThreeHundredThousandPesos
(P300,000.00),letawritof
preliminaryinjunctionissue,enjoining
defendants,theirrepresentatives,or
anyoneactingintheirbehalf;(a)from
cancelingthecontracttoselldated
December31,1996;and(b)from
disallowingorpreventingtheentryand
exitofplaintiffsvehiclesandthoseof
itsrepresentativesthroughLot1031F
andotherundeliveredlotsconcerned.
[if!

supportFootnotes][8][endif]

Based on this Order, the trial court


issued the Writ on 18 August 1998
which the sheriff served on the
same date.
Felipe assailed the Order and the
Writ in a special civil action for
certiorari before the Court of
Appeals. The appellate court,
however, dismissed outright
Felipes petition. The appellate
court also denied on 24 August
1999 Felipes motion for
reconsideration. Thus, Felipe filed
the instant petition questioning the
propriety of the writ of preliminary
injunction issued by the trial court.
The Rulings of the Trial Court
and the Court of Appeals

In granting the Writ, the trial court


stated Thereisnodisputethatplaintiffhas
alreadypaiddefendantstheamountof
P51,248,348.26outofthetotal
considerationofP65,520,475.00.
Plaintiffhasalsodepositedwiththe
OfficeoftheClerkofCourtthe
amountofP4,048,950.00,leavinga
balanceofP10,223,176.74.
Plaintiffhadalreadystartedtheroad
developmentintheproperties
deliveredtoit.Inotherwords,ithas
alreadyspentmuchtodevelopthe
propertieswhichformthebulkofthe
parcelsoflandsubjectofthecontract.
Ingresstoandegressfromplaintiffs
developmentactivitieslieonan
undeliveredparcelofland.Throughit
passthevehicles,equipment,supplies
andmaterials,aswellastheworkers,
requiredbytheproject.Theclosureof
thispassagehasapparentlystymiedthe
developmentinthearea.
About78%ofthepropertiesareinthe
handsofplaintiff.Accesstothese
propertiesisunderthecontrolof
defendants,theentrancebeinglocated
inLot1031F,oneoftheremaining
undeliveredlots.Sincetheentrance
gatehasbeenclosedbydefendants,it
strikesthemindofthecourtthatLot
1031Fandtheotherundeliveredlots
havenow,inamannerofspeaking,
imprisonedthedeliveredproperties.
Itisnotthereforehardtoseethatthe
closureoftheentrancegatehas

workedtotheprejudiceofplaintiffand
willcertainlyjeopardizethe
developmentworkinthedelivered
properties.Elementaryjusticeandthe
spiritoffairplaythusdictatethatthe
statusquoante,whichisthesituation
beforetheclosurewhenplaintiffs
representativeswereabletopass
throughLot1031F,berestored.
Insofarasdefendantsthreatened
cancellationofthecontracttosell,the
Courthasseenthatoutofthetotalarea
of131,040.95squaremeterscovered
bythecontract,plaintiffhadalready
paidfor102,496.69squaremeters,and
thatithaddepositedP4,048,950.00to
payforsomeoftheundelivered
parcels.Itisbutfairthatsuchamove
be,inthemeantime,disallowed.
[if!

supportFootnotes][9][endif]

In dismissing outright Felipes


petition for certiorari, prohibition
and mandamus assailing the trial
courts Order and the Writ, the
Court of Appeals stated Thepetitionshouldbedismissed
outright,thepetitionerhasnostanding
here.Hemaybetheownerofthelotin
questionbutheisnotapartylitigantin
thecaseaquo.Hisbeingasonof
defendantspouseinthelowercourt
doesnotgivehimthecapacitytosue.
Ofcourse,heisnotwithoutlegal
remedytoprotecthisinterest.
[if!supportFootnotes]

[10][endif]

The Issue
In his Memorandum, Felipe
narrows the inquiry to -

MAYPETITIONERBEDEPRIVED
OFHISPROPERTYWITHOUTDUE
PROCESSOFLAWAND
PAYMENTOFJUST
COMPENSATIONFORTHE
BENEFITOFPRIVATE
RESPONDENT?
Felipe laments that the dismissal of
his petition resulted in the outright
confiscation of his property for the
private use of Gothong Lines,
without due process of law and just
compensation. Felipe claims that in
dismissing his petition, the Court of
Appeals effectively sustained the
trial courts Order divesting him of
his rights over Lot 1031-F.
The question of whether Gothong
Lines may demand the turn over of
the parcels of land listed in Annex
A of the Contract is not our concern
here. The issue in this petition is
whether the Court of Appeals erred
in dismissing Felipes petition.
The Courts Ruling
The petition is bereft of merit.
Dismissal by the Court of
Appeals of
Felipes petition was proper.
Felipe committed a procedural
blunder in filing a special civil
action for certiorari to assail the
Order and the Writ. Felipe was not
a party in Civil Case No. 5020-L.
He could not, therefore, assail the
writ of preliminary injunction
through a petition for certiorari
before the Court of Appeals. As
[if!supportFootnotes][11][endif]

correctly pointed out by the Court


of Appeals, Felipe does not
possess the requisite standing to
file such suit.
In Ciudad Real v. Court of
Appeals,
this Court
ruled that there is grave abuse of
discretion if the appellate court
recognizes the standing of a party,
not a litigant in the trial court
proceedings, to join a petition for
certiorari. The Court explained:
Worsewastherulingoftherespondent
appellatecourtsanctioningthe
standingofMagdiwangRealty
Corporationtojoinsaidpetitionfor
certiorari.Astherecordsshow,
MagdiwangfiledaMotionfor
InterventiononJuly18,1989invoking
itsallegedMemorandumof
AgreementwithDoaJuana
DevelopmentCorporationdatedJuly
15,1982.Thetrialcourt,however,
deniedthismotionandMagdiwangdid
notquestiontherulingintheappellate
court.Therulingthus,becamefinal.
Afterabouttwo(2)yearsoronAugust
27,1991,Magdiwangagainfileda
MotiontoSubstituteand/orJoinas
Party/Plaintiffrelyingonthesame
MemorandumofAgreement.Thetrial
courtsimilarlydeniedthemotion,and
thedenialalsoattainedfinalityas
Magdiwangdidnotfurtherchallenge
itscorrectness.Despitethefinalityof
theorderdenyingMagdiwangs
interventionwaybackin1989,the
[if !supportFootnotes][12][endif]

respondentcourtinitsDecisionof
August20,1992recognizedthe
standingofMagdiwangtoassailinthe
appellatecourttheCompromise
Agreement.Again,thisruling
constitutesgraveabuseofdiscretion
forMagdiwangwasnotapartyin
interestinCivilCaseNo.Q35393.
The wisdom of this ruling is all too
apparent. If a person not a party to
an action is allowed to file a
certiorari petition assailing an
interlocutory order of the trial court,
such as an injunctive order and
writ, proceedings will become
unnecessarily complicated,
expensive and interminable.
Eventually, this will defeat the
policy of our remedial laws to
secure party-litigants a speedy and
inexpensive disposition of every
action.
Felipe could have simply
intervened
in the trial
court proceedings to enable him to
protect or preserve a right or
interest which may be affected by
such proceedings. A motion to
intervene may be filed at any time
before rendition of judgment by the
trial court.
The purpose
of intervention is not to obstruct or
unnecessarily delay the placid
operation of the machinery of trial.
The purpose is merely to afford
one, not an original party but
possessing a certain right or
interest in the pending case, the
[if !supportFootnotes][13][endif]

[if !supportFootnotes][14][endif]

opportunity to appear and be joined


so he could assert or protect such
right or interest.
Indeed, Felipe could have easily
joined his parents as defendants in
resisting the claim of Gothong
Lines.
A resolution affirming the Court of
Appeals outright dismissal of
Felipes petition for these reasons
would have been sufficient.
Nevertheless, we deem it best to
address the propriety of the
issuance by the trial court of the
writ of preliminary injunction before
writing finis to this petition.
Issuance of writ of preliminary
injunction
was also proper.
Preliminary injunction is an order
granted at any stage of an action,
prior to the judgment or final order,
requiring a party, court, agency or
person to perform or to refrain from
performing a particular act or acts.
A preliminary injunction,
as the term itself suggests, is
merely temporary, subject to the
final disposition of the principal
action. Its purpose is to preserve
the status quo of the matter subject
of the action to protect the rights of
the plaintiff during the pendency of
the suit. Otherwise, if no
preliminary injunction is issued, the
defendant may, before final
judgment, do the act which the
plaintiff is seeking the court to
restrain. This will make ineffectual
[if !supportFootnotes][15][endif]

[if !

supportFootnotes][16][endif]

the final judgment that the court


may afterwards render in granting
relief to the plaintiff.
The issuance of a writ of
preliminary injunction rests entirely
within the discretion of the court
and is generally not interfered with
except in cases of manifest abuse.
The assessment and
evaluation of evidence in the
issuance of the writ of preliminary
injunction involve findings of facts
ordinarily left to the trial court for its
conclusive determination.
[if !supportFootnotes][17][endif]

[if

!supportFootnotes][18][endif]

[if !supportFootnotes][19]

[endif]

We find that there was adequate


justification for the issuance of the
assailed writ of preliminary
injunction. There is no dispute that
the Spouses Dungog entered into
the Contract with Gothong Lines
which included Lot 1031-F owned
by Felipe. Felipe admitted that he
authorized his parents to sell this
lot. He also admitted that his
parents had delivered to Gothong
Lines Lot 1031-F along with other
parcels of land. However, the
Spouses Dungog threatened to
cancel the Contract and to deny
Gothong Lines passage through
Lot 1031-F allegedly due to nonpayment of the subsequent
installments.
In applying for the Writ, Gothong
Lines sought to restrain in the
meantime the Spouses Dungog
from canceling the Contract in
order not to render the judgment

ineffectual. Gothong Lines also


sought to preserve its right of way
through Lot 1031-F to maintain
access to the other parcels of land
previously delivered by the
Spouses Dungog to Gothong
Lines.
A careful reading of the trial courts
assailed Order discloses that the
Writ enjoined the cancelation of the
Contract on the basis of Gothong
Lines substantial performance of
the Contract. The trial court also
enjoined the closure of the
entrance gate in Lot 1031-F to
preserve the status quo ante.
Under Section 3, Rule 58
of the 1997 Rules on Civil
Procedure, a preliminary injunction
is proper when the plaintiff appears
entitled to the relief demanded in
the complaint. The trial court found
that Gothong Lines had already
paid P51,248,348.26 out of the
total consideration of
P65,520,475.00. Gothong Lines
also consigned with the court an
additional P4,048,950.00 leaving a
balance of P10,223,176.74. The
trial court likewise found that 78%
of the properties were already in
the possession of Gothong Lines.
Moreover, the status quo, which is
the last actual peaceable
uncontested status that preceded
the controversy,
was
that Gothong Lines had access to
the lots subject of the Contract
through the entrance gate in Lot
[if !supportFootnotes][20]

[endif]

[if !supportFootnotes][21][endif]

1031-F. That is why Gothong Lines


commenced construction of its pier
and the development of the roads
within the parcels of land covered
by the Contract. The issuance of
the Writ would no doubt preserve
the status quo between the
Spouses Dungog and Gothong
Lines that existed prior to the filing
of the case. We agree with the trial
court that the status quo should be
maintained until the issue on the
parties respective rights and
obligations under the Contract is
determined after the trial.
Clearly, in issuing the Writ, the trial
court did not forthwith deprive
Felipe of his ownership of Lot
1031-F. Neither did the Writ have
the effect of ousting Felipe from
possession of the lot. The trial
court did not rule on the merits of
the case so as to amount to a
deprivation or confiscation of
property without due process of law
or just compensation. There was
no adjudication on the rightful
possession or ownership of the
contested parcels of land subject of
the Contract. The trial court issued
the injunction only as a preventive
remedy to protect during the
pendency of the action Gothong
Lines right to a final and effective
relief.
WHEREFORE, the petition is
DENIED for lack of merit.
VELASQUEZ VS UNDERSECRETARY

Petition for certiorari to annul and/or set


aside the resolution/ letter dated
January 4, 1989 of the public
respondent, Undersecretary of Justice
Artemio G. Tuquero ordering a
reinvestigation of I.S. No. 86-28751.
Respondent Edgardo Avila was a Cash
and Business Development Consultant
of the Techtrade Management
International Corporation, authorized to
follow-up business transactions,
including loan applications submitted to
the company.
On September 29, 1986, Avila informed
the company that he had a borrower
(whom he did not identify) for P200,000
with interest of 3%/month for a 30-day
term from September 29 to October 29,
1988. This was approved by the
company which issued to him a pay-tocash check for P194,000 after deducting
the 3% interest of 6,000. Instead of
returning the borrowed amount on due
date or giving a satisfactory explanation
for the supposed borrower's failure to
pay the loan despite written demands,
Avila resigned from the company on
December 17, 1986 promising that: "... I
shall set aside the P200,000 upon its
subsequent collection (subject of Atty.
Caacbay's letter of 12/10/86) to answer
for the P100,000 portion of Tony's
P700,000 loan to you; please treat the
P100,000 balance, less my unpaid
professional fee and gas expenses from
November 16 to December 15, 1986, as
my separation and compulsory benefit"
(p. 6, Rollo).
On December 23, 1986, petitioner Felix
A. Velasquez, as Executive VicePresident/Managing Director of
Techtrade, filed a complaint for estafa
against Avila in the Manila City Fiscal's
Office, where it was docketed as I.S. No.

86-28751. Assistant Fiscal Romulo


Lopez dismissed the complaint.
However, upon review by the Chief,
Investigation Division of the City Fiscal's
Office, the latter set aside Fiscal Lopez'
resolution and ordered the filing of an
information for estafa against Avila in the
Regional Trial Court.
Avila twice sought a reconsideration of
that resolution, but both motions were
denied by the City Fiscal (Annexes F &
H).
Before arraignment, Avila filed on June
29, 1987 in the Department of Justice a
petition for review (Annex I) which the
petitioner opposed (Annex J). On
February 15, 1988, Justice
Undersecretary Silvestre Bello III denied
the petition for review (Annex L). A
motion for reconsideration (Annex M) of
the denial did not prosper (Annex O).
On October 14, 1988, Avila filed a
second motion for reconsideration which
the Undersecretary of Justice,
Honorable Artemio Tuquero granted on
January 4, 1989 (Annex A, Petition). He
directed the City Fiscal:
... to conduct a reinvestigation of this
case to afford respondent to properly
present evidence that he was duly
authorized to pay the subject creditors
and for complainant to rebut the same
with controverting evidence, and
thereafter to resolve the case anew on
the basis of all the evidence adduced.
(p. 15, Rollo.)
The complainant filed a motion for
reconsideration (Annex C) of that
resolution but it was denied on May 15,
1989 (Annex B, Petition). Hence, this
petition for certiorari.
The petition is meritorious. This case is
governed by our decision in Crespo vs.
Mogul, 151 SCRA 462, where we ruled

that once the information is filed in court,


the court acquires complete jurisdiction
over it. A motion for reinvestigation
should, after the court had acquired
jurisdiction over the case, be addressed
to the trial judge and to him alone.
Neither the Secretary of Justice, the
State Prosecutor, nor the Fiscal may
interfere with the judge's disposition of
the case, much less impose upon the
court their opinion regarding the guilt or
innocence of the accused, for the court
is the sole judge of that.
The rule therefore in this jurisdiction is
that once a complaint or information is
filed in Court any disposition of the case
as its dismissal or the conviction or
acquittal of the accused rests in the
sound discretion of the Court. Although
the fiscal retains the direction and
control of the prosecution of criminal
cases even while the case is already in
Court he cannot impose his opinion on
the trial court. The Court is the best and
sole judge on what to do with the case
before it. The determination of the case
is within its exclusive jurisdiction and
competence. A motion to dismiss the
case filed by the fiscal should be
addressed to the Court who has the
option to grant or deny the same. It does
not matter if this is done before or after
the arraignment of the accused or that
the motion was filed after a
reinvestigation or upon instructions of
the Secretary of Justice who reviewed
the records of the investigation.
In order therefor[e] to avoid such a
situation whereby the opinion of the
Secretary of Justice who reviewed the
action of the fiscal may be disregarded
by the trial court, the Secretary of
Justice should, as far as practicable,
refrain from entertaining a petition for

review or appeal from the action of the


fiscal, when the complaint or information
has already been filed in Court. The
matter should be left entirely for the
determination of the Court. (Crespo vs.
Mogul, 151 SCRA 462, 471 & 472.)
Crespo vs. Mogul was reiterated in
Marquez vs. Alejo, 154 SCRA 302; Sta.
Rosa Mining Co. vs. Asst. Provincial
Fiscal Augusta Zabala, 153 SCRA 367;
Republic vs. Judge Sunga, G.R. No.
38634, June 20, 1988; Peralta vs. CFI
of La Union, 157 SCRA 476 and
Almazar vs. Judge Cenzon, 161 SCRA
454.
The Undersecretary of Justice gravely
abused his discretion in ordering the reinvestigation of the criminal case against
Avila after it had been filed in court. The
avowed purpose of the reinvestigation
"to give an opportunity to the private
respondent to present an authentic copy
of the board resolution of the offended
party
(Techtrade
Management
International
Corporation)
which
[allegedly] had authorized him to deal
and otherwise dispose of the funds of
the corporation" (p. 72, Rollo), can also
be achieved at the trial in the lower court
where that piece of evidence may be
presented by the accused as part of his
defense.
WHEREFORE, the petition for certiorari
is granted. The order dated January 4,
1989 of the public respondent (Annex A,
Petition) is hereby annulled and set
aside, with costs against the petitioner.

PEOPLE VS BERIALES

Facts: A case of three men who were


charged for the murder of Saturnina on
Sept. 13, 1974. During the hearing on
Nov. 26, 1974, upon motion of the

defense the Court ordered the reinvestigation of the case pending


submission of the Fiscal of its reports.
Couple of postponements was made
until Dec. 13, 1974 hearing when the
Court proceeded with the arraignment
and trial in the absence of the Fiscal
and its report on re-investigation, and
over the disagreement of the defense.
The CFI of Leyte relied on the private
prosecutor being authorized by the
Fiscal to present evidence and the
defense presumed to have waived its
right over its disagreement. Trial then
proceeded and the 3 found guilty of he
offense. Thus, this appeal on the
constitutional requirement of due
process.
Issue: Whether or not due process of
law had been observed.
[endif]
Held: Constitutional due process was
violated, thus, case remanded to CFI
for arraignment and trial. Court should
have held in abeyance the trial while
the report on e-investigation was still
pending. Consistent disregard of the
defense objection on the arraignment,
trial,
presentation
of
private
prosecutors evidence, and rendition of
judgment violates due process.
Prosecutor or Fiscal entrusted with the
investigation is duty bound to take
charge until final termination. They
shall have direction and control of the
criminal prosecution over private
prosecutors.

C. INJUCNTION AND WRITS OF RESTRAINT IN


PROPER CASES
PRIMICIAS VS PANGASINAN

Facts:
On February 8, 1965, Primicia was
driving his car within the jurisdiction of
Urdaneta when he was found
violating Municipal Order 3, Series of
1964 for overtaking a truck. The
Courts of First Instance decided that
from the action initiated by Primicias,
the Municipal Order was null and void
and had been repealed by Republic
Act 4136, the Land Transportation
and Traffic Code
Issues:
1. Whether or not Municipal Order 3
of Urdaneta is null and void
2. Whether or not the Municipal Order
is not definite in its terms or
ambiguous.
Held:
1. Municipal Order 3 is null and void
as there is an explicit repeal in RA
4136 and as per general rule, the
later law prevails over an earlier law
and any conflict between a municipal
order and a national law must be
ruled in favor of the statute.
2. Yes, the terms of Municipal Order 3
was ambiguous and not definite.
Vehicular Traffic is not defined and
no distinctions were made between
cars, trucks, buses, etc.
HERNANDEZ VS ALBANO

SHORT VERSION:
Hernandez sought to enjoin the fiscals

investigation of charges filed against him.


The courts only do so in extreme cases;
Hernandez did not prove that his belonged to
those exceptions.
FACTS:
Isabela Rep. Delfin Albano
(respondent-appellee) filed a
complaint with the Manila city fiscal
against Finance Secretary & Central
Bank Monetary Board Presiding Officer
Jaime Hernandez (petitioner-appellant)
for violating RPC Art. 216 (possession
of prohibited interest by a public
officer), Commonwealth Act 626
*which provides for the penalty for
violations of Article VII, Section 11,
subsection (2) of the Constitution) or
RA 265 (Central Bank Act).
o The complaint involved
Hernandezs alleged
shareholdings in University of
the East, Bicol Electric Co., Rural
Bank of Nueva Caceres, DMG
inc., and University of Nueva
Caceres and the claim that said
corporations obtained dollar
allocations from the Central
Bank, through the Monetary
Board, during Hernandezs
incumbency as presiding officer
thereof.
o In total, there were five charges
docketed in the fiscals office.
After joint investigation of the charges
before Second Assistant City Fiscal of
Manila Carlos Gonzales (respondent),
Albano moved to exclude the alleged
violation of RP Art 216 as the
applicability of the statute was
pending before the SC in Solidum v
Hernandez (it had since been resolved
adversely against Hernandez). The
fiscal granted the motion.
o Hernandez sought the dismissal
of the remaining charges on the
grounds that (a) violation of
Article VII, Section 11,
subsection (2) of the
Constitution, punishable under
Commonwealth Act 626, should
be prosecuted at the domicile of
the private enterprises affected
there by; and that (b) violation
of Section 13 of Republic Act

265 is not criminal in nature.


Dismissal and reconsideration
denied.
Hernandez went to the Court of First
Instance Manila on certiorari and
prohibition praying for preliminary
injunction to restrain the fiscals office
from continuing the investigation.
o The CFI dismissed the petition.

ISSUE:
could the Manila fiscal be restrained
from proceeding with the investigation
of the charges against Hernandez?
REASONING:
By statute, the prosecuting officer of
the City of Manila and his assistants
are empowered to investigate crimes
committed within the city's territorial
jurisdiction. Not a mere privilege, it is
the sworn duty of a Fiscal to conduct
an investigation of a criminal charge
filed with his office. The power to
investigate postulates the other
obligation on the part of the Fiscal to
investigate promptly and file the case
of as speedily.
o A rule was formulated that
ordinarily criminal prosecution
may not be blocked by court
prohibition or injunction.
o However, in extreme cases, a
relief in equity could be availed
of to stop a purported
enforcement of a criminal law
where it was necessary: (a) for
the orderly administration of
justice; (b) to prevent the use of
the strong arm of the law in an
oppressive and vindictive
manner; (c) to avoid multiplicity
of actions; (d) to afford
adequate protection to
constitutional rights; and (e) in
proper cases, because the
statute relied upon is
unconstitutional, or was "held
invalid."
Commonwealth Act 626 provides the
penal sanction for a violation of
Constitution Art VII sec. 11(2): a fine of
not than P5000 or imprisonment of not
more than 2 years, or both.

The legal mandate in Section


14, Rule 110 of the Rules of the
Court is that "[i]n all criminal
prosecutions the action shall be
instituted and tried in the court
of the municipality or province
wherein the offense was
committed or any one of the
essential ingredients thereof
took place. Where an offense is
wholly committed outside the
territorial limits wherein the
court operates, said court is
powerless to try the case.

Similarly, the Manila fiscal could


not investigate a crime
committed within the exclusive
confines of another province.
Otherwise, they would be
overreaching the territorial
limits of their jurisdiction, and
unlawfully encroach upon
powers and prerogatives of
fiscals of the province.

Based on the facts of the case,


Possession of prohibited
interests is but one of the
essential components of the
offense. As necessary an
ingredient thereof is the fact

that petitioner was head of a


department: Secretary of
Finance. So also, the fact that
while head of department and
chairman of the Monetary Board
he allegedly was financially
interested in the corporations
aforesaid which so the dollar
allocations, and that he had to
act officially, in his dual
capacity, not in Camarines Sur,
but in Manila where he held his
office.
o Since criminal action must be
instituted and tried in the place
where the crime or an essential
ingredient thereof, took place, it
stands to reason to say that the
Manila under the facts obtained
here, have jurisdiction to
investigate the violation
complained of.
Violation of RA 265 sec. 13 was
criminal in nature, as the law clearly
provided the penal sanction for
violating its provisions.

RULING: CFI judgment affirmed


GUINGONA JR

Das könnte Ihnen auch gefallen